★東大入試作問者になったつもりのスレ★ 第八問

このエントリーをはてなブックマークに追加
1132人目の素数さん
理系で数学が得意な高校生が25〜50分で
解ける問題を考えてうぷするスレ。
これ以上の難易度の問題はスレ違いとなります。
関連スレへどうぞ

 過去ログ
★東大入試作問者になったつもりのスレ★
http://science.2ch.net/test/read.cgi/math/1000592003/
★東大京大入試作問者になったつもりのスレ★
http://science.2ch.net/test/read.cgi/math/1046165076/
★東大入試作問者になったつもりのスレ★ 第三問
http://science3.2ch.net/test/read.cgi/math/1069171672/
★東大入試作問者になったつもりのスレ★ 第4問
http://science3.2ch.net/test/read.cgi/math/1099493043/
★東大入試作問者になったつもりのスレ★ 第五問
http://science4.2ch.net/test/read.cgi/math/1116752400/
★東大入試作問者になったつもりのスレ★ 第六問
http://science4.2ch.net/test/read.cgi/math/1134000000/
★東大入試作問者になったつもりのスレ★ 第七問
http://science4.2ch.net/test/read.cgi/math/1148569109/
2132人目の素数さん:2006/12/24(日) 05:04:00

3132人目の素数さん:2006/12/24(日) 05:25:41
king愛してる
4132人目の素数さん:2006/12/24(日) 06:14:04
「勉強は誰でもやればできる」と言ってる人達に聞きたい。
いったい何をどうやれば誰でもできるようになるのか?
例えば、
「任意の実数Xに対して、不等式1+KX^2≦CosXが成り立つような定数Kの範囲を定めよ(制限時間20分)。」
という問題(入試問題としては「やや難」程度か?)を題材に、試験場での思考手順を実況中継風に、
なおかつその手順がどうして誰にでも身に付くのかを詰め将棋の解説風に参照文献も交えつつ、具体的かつ詳細に教えてほしい。
「ターゲットを暗記して、英文解釈教室を読めば受かる」とか「駿台行って、Z会やれば受かる」とかの
無意味な抽象的方法論やら「親がどうしたこうした」とかの根性論は一切不要。
誰でもできるようになるプログラムがあるなら、それで一儲けさせてほしいんだが。
5132人目の素数さん:2006/12/24(日) 07:36:40
スレ違い
6KingOfUniverse ◆667la1PjK2 :2006/12/24(日) 09:07:17
talk:>>3 私と■■するのか?
7132人目の素数さん:2006/12/24(日) 09:09:01
a^2+b^2+c^2=abc.
(bc-a)^2+b^2+c^2=(bc-a)bc.
(a,b,c)->(bc-a,b,c).

3^2+3^2+3^2=3x3x3.
(3,3,3)->(6,3,3).
(6,3,3)->(6,15,3).
(6,15,3)->(39,15,3).
(6,15,3)->(6,15,87).
8132人目の素数さん:2006/12/24(日) 10:27:26
6 名前:KingOfUniverse ◆667la1PjK2 [] 投稿日:2006/12/24(日) 09:07:17
talk:>>3 私と囲碁するのか?
9132人目の素数さん:2006/12/24(日) 11:44:49
1+kx^2-cosx<=0
k<=(cosx-1)/x^2 0->-2(-π)^-2
10132人目の素数さん:2006/12/24(日) 11:48:32
1億寄付するといえばOAで入れてくれるだろう。
11132人目の素数さん:2006/12/24(日) 11:49:32
100億寄付すれば名誉学位が授与される。
100億あれば学校なんかいく必要もない。
12132人目の素数さん :2006/12/24(日) 12:18:34
>>11 名誉学位を何百もかき集めている I.氏なんてのがいる。
13132人目の素数さん:2006/12/24(日) 19:07:51
>>4
マクローリン展開を考えると-1/2が怪しいから、答えにめどをつけてから
あとは確認のつもりで解く。
14132人目の素数さん:2006/12/24(日) 21:47:44
C****C(0)は原点を中心とする半径1の円板である
C(n)とx軸との交点を中心として,C(n)の半径を1/2倍した半径を持つ円板C(n+1)を描いていき
x軸を軸に回転した立体の体積をV(n)としたとき,lim[n→∞]V(n)を求めよ
15132人目の素数さん:2006/12/25(月) 00:13:35
とりあえず前スレの未解決問題。
1.(1)eを無理数とする。
このときaを0でない有理数として、aeは必ず無理数であることを証明せよ。
(2)eを無理数とする。
e^eが有理数でかつe^(1/e)が無理数である。
このようなeは存在するか。
(3)eを無理数とする。
e^e^eが有理数でe^eが無理数であるようなeは存在するか。
(4)eを自然対数の底とする。eの定義を述べた上で、eは2.4より大きいことを証明せよ。
※(1)のみ解決。

2.素数pについて次のことが成り立つか。成り立つなら証明し、成り立たないなら
反例をひとつ挙げよ。
(1)pが素数ならばp^2+1も素数とは限らない。
(2)pが素数でないならばp^2+1は素数かもしれない。
(3)pが素数でないとすればp^2-1は素数にならないかもしれない。
(4)pとp^3+2p^2+3p+1が共に素数になることはない。

3.tan1°+tan2°は有理数か。

4.問題(D****)
平面Hは3点A,B,Cを通る。
Dは、AD^2+BD^2+CD^2を最小にする点である。
A(1,0,0)B(1,0,1)C(0,0,1)とするとき、Dの座標を求めよ。

16132人目の素数さん:2006/12/25(月) 00:14:56

5.不等式
(a^2+b^2+c^2)(a^4+b^4+c^4)≧a^6+b^6+c^6
を常に成り立たせるような(a,b,c)の条件のうち、最も広いものを答えよ。


6.α、β、γ、δを4次方程式f(x)=0の解とする。またα、β、γ、δはすべて素数である。
f(x)=ax^4+bx^3+cx^2+dx+eとする。
(1)a+b+c+d+eの最小値はいくらか。
(2)abcdeの最小値はいくらか。
(3)a^5+b^5+c^5+d^5+e^5とα^4+β^4+γ^4+δ^4の大小を比較せよ。

7.コインを投げて最初から表がk回連続で出たら2^k円もらえるゲームを考える。
このゲームは何度でもチャレンジできる。但し、1回やるたびごとに参加料金として
100,000,000円払う必要がある。このゲームには参加すべきか。期待値で
100,000,000円より多いなら参加すべきと答えよ。そうでないなら参加すべきでないと答えよ。

8.f(x)=xe^xとする。
(1)f'(x)を求めよ。
(2)I(a,b)=∫[a,b]{f'(x)}^2dxとおく。I(-1,1)を求めよ。
(3)J=I^2+2aI+bとする。Iの2次方程式J=0が相異なる2つの実数解を持つような
(a,b)の範囲をab平面上に図示せよ。
17132人目の素数さん:2006/12/25(月) 00:17:28
a,b,cは次の3つの条件を満たしている。
条件1 a^2+b^2+c^2=3
条件2 2a+b+4c=4
条件3 (a+b)(b+c)(c+a)=1/64
このとき、次の問いに答えよ。
(1)a,b,cがすべて虚数で条件1〜条件3を満足するものが存在することを示せ。
(2)aのみが虚数で、b,cが実数であるような(a,b,c)で条件1〜条件3を満足する
ものは存在しないことを示せ。

10.f(x)=x^3-3xとする。
f(f(f(x)))をf(f(x))で割った余りをg(x)、
f(f(x))をf(x)で割った余りをh(x)、
g(h(x))をh(g(x))で割った余りをi(x)とする。
このとき、
{i(x)}^2+{f(x)}^2={g(x)}^2+{f(x)}^2
は常には成り立たないことを示せ。

11.f(x)、g(x)、h(x)は3次関数で
f(1)=g(2)=h(3)=1
f(2)=g(3)=h(1)=2
f(3)=g(1)=h(2)=3
が成り立っている。このとき、f(x)+g(x)+h(x)は最大値も最小値も持たないことを
示せ。

12.次の問いに答えよ。
(1)y=1/xとy=1/x^2(x>0)とで囲まれた部分の面積を求めよ。
(2)y=1/x^2とy=1/x^3(x>0)とで囲まれた部分の面積を求めよ。
(3)y=1/x^nとy=1/x^[n+1](x>0)とで囲まれた部分の面積を求めよ(nは自然数)。
(4)AV=SEXで、A=1/a V=1/v、S=1/s E=1/e X=1/xのときavをs e xで表せ。
(5)以上によりeの値を求めよ。
※(4)のみ解決。
18132人目の素数さん:2006/12/25(月) 00:20:52

13a,b,c,x,y.zはすべて自然数で、
a^x+b^y=c^zを満たすとする。このとき、自然数N_1,N_2,N_3の最大公約数を
G(N_1 N_2 N_3)と書くことにすると、
G(a b c)≧G(x y z)
であることを示せ。

14.
500円玉がn枚、100円玉がm枚ある。(n,mは自然数)
k個取り出すとき、500円のほうが100円玉より多く取り出せる確率をp_kとする。
任意のp_kにたいし、p_k>1/2となるような、n,mの条件を求めよ。


15.xyz空間上において、点(3,5,7)に光源がある。この光源は四方八方に光を放つ。
今、xy平面上において、
y≦x^2
x≦y^2
なる部分に、光を遮断する壁を設置した。
この空間をyz平面で切ったとき影の(光の当たらない)面積はいくらか。

新作問題
C:16≦x^2+y^2≦25を満たす領域とし、Cと合同な板を用意し、Dと書くことにする。
Dの重心をGとする。GがC内を動くように、Dを動かすときDの動く領域をEとする。
Eをx軸の周りに1回転させてできる立体の体積を求めよ。

19132人目の素数さん:2006/12/25(月) 01:08:22
e^π>21を証明しろ
20132人目の素数さん:2006/12/25(月) 01:41:43
e>2.7
π>3.1を既知とする。
e^π>2.7^3.1=21.738・・・より
成立する。 (終)

√n(nは自然数)が有理数であるための必要十分条件はnが平方数であること
を証明せよ。
21132人目の素数さん:2006/12/25(月) 01:43:05
任意の自然数a,bにたいし、
23a^4+17b^2は平方数でないことを示せ。
22132人目の素数さん:2006/12/25(月) 01:46:09
以下の条件(a)(b)(c)をすべて満たす(a,b,c)の組の個数を求めよ。

(a)a^2+b^2=c^2
(b)a,b,cは1000以下の自然数
(c)a,bは奇数
23132人目の素数さん:2006/12/25(月) 01:48:39
すべての点が格子点である正五角形は存在するか。
24132人目の素数さん:2006/12/25(月) 01:58:25
>>16
Σ[1,∞](1/2)^k*2^k = ∞  期待値は∞
25132人目の素数さん:2006/12/25(月) 02:48:23
解いてみたけど
1.D*****2.C****3.C*****4.D****5.D♯6.D♯7.B**8.C******
9.D***10.C**11.D**12.D#13.D#14.C***15.D*****新作C***
くらいかと

26すべてを超えしもの:2006/12/25(月) 02:55:09
>>20
地道に場合わけすれば容易に証明可能。
>>21
あるわけがない。無限工加法でもやれば。(終)
>>22
0
>>23
ない
27132人目の素数さん:2006/12/25(月) 02:58:33
8と13は使えそう。問題文長いのは読むのマンドイ。
28132人目の素数さん:2006/12/25(月) 03:01:54
>>15
4. ベクトルOAをa,ベクトルOBをb,ベクトルOCをc,ベクトルODをdとおく。このとき
AD^2+BD^2+CD^2=3|d|^2−2d・(a+b+c)+|a|^2+|b|^2+|c|^2
=3|d-(a+b+c)/3|^2+|a|^2+|b|^2+|c|^2−|a+b+c|^2/3
=3|d-(a+b+c)/3|^2+{|a−b|^2+|b−c|^2+|c−a|^2}/3
となるから、d=(a+b+c)/3のときにAD^2+BD^2+CD^2は最小値{|a−b|^2+|b−c|^2+|c−a|^2}/3
を取る。よって特に、A(1,0,0)B(1,0,1)C(0,0,1)とするとき、Dの座標は(1/3,1/3,1/3)となる。
29132人目の素数さん:2006/12/25(月) 03:38:32
13
2^4+3^2=5^2、…氏ね
>>25
問題間違ってるのによく解けたね
30132人目の素数さん:2006/12/25(月) 05:19:45
4がDって時点でたかが知れてる

コテハンの作問者カモン
31KingOfUniverse ◆667la1PjK2 :2006/12/25(月) 07:53:24
talk:>>8 私に何か用か?
32体積馬鹿一代:2006/12/25(月) 15:42:16
ネタ切れの予感が

【xyz空間内の原点O、点Pは次を満たす。

 中心が点P、半径1/OP^2の球は、xy平面とz軸の両方に共有点を持つ。

 このとき点P全体がなす立体の体積を求めよ。】
33すべてを超えしもの:2006/12/25(月) 16:52:54
2.素数pについて次のことが成り立つか。成り立つなら証明し、成り立たないなら
反例をひとつ挙げよ。
(1)pが素数ならばp^2+1も素数とは限らない。
(2)pが素数でないならばp^2+1は素数かもしれない。
(3)pが素数でないとすればp^2-1は素数にならないかもしれない。
(4)pとp^3+2p^2+3p+1が共に素数になることはない。

(1)p=3ならp^2+1=10よりtrue
(2)pは素数という仮定があるからtrue
(3)pは素数という仮定があるからtrue
(4)p=2なら23になるのでfalse
34すべてを超えしもの:2006/12/25(月) 16:53:49
5.不等式
(a^2+b^2+c^2)(a^4+b^4+c^4)≧a^6+b^6+c^6
を常に成り立たせるような(a,b,c)の条件のうち、最も広いものを答えよ。

a,b,cは0以上の数
35すべてを超えしもの:2006/12/25(月) 16:55:36
6.α、β、γ、δを4次方程式f(x)=0の解とする。またα、β、γ、δはすべて素数である。
f(x)=ax^4+bx^3+cx^2+dx+eとする。
(1)a+b+c+d+eの最小値はいくらか。
(2)abcdeの最小値はいくらか。
(3)a^5+b^5+c^5+d^5+e^5とα^4+β^4+γ^4+δ^4の大小を比較せよ。

(1)解2,3,5,7のとき-A(x-2)(x-3)(x-5)(x-7)=0
Aを大きくすればいくらでも大きくなるので最小値はない。
(2)(1)と同様
(3)ケースbyケース場合わけはできない。
36すべてを超えしもの:2006/12/25(月) 16:56:57
7 やるべき

9a,b,cは次の3つの条件を満たしている。
条件1 a^2+b^2+c^2=3
条件2 2a+b+4c=4
条件3 (a+b)(b+c)(c+a)=1/64
このとき、次の問いに答えよ。
(1)a,b,cがすべて虚数で条件1〜条件3を満足するものが存在することを示せ。
(2)aのみが虚数で、b,cが実数であるような(a,b,c)で条件1〜条件3を満足する
ものは存在しないことを示せ。

(1)ない。
(2)条件2を満たさないから。
37すべてを超えしもの:2006/12/25(月) 16:57:57
10.f(x)=x^3-3xとする。
f(f(f(x)))をf(f(x))で割った余りをg(x)、
f(f(x))をf(x)で割った余りをh(x)、
g(h(x))をh(g(x))で割った余りをi(x)とする。
このとき、
{i(x)}^2+{f(x)}^2={g(x)}^2+{f(x)}^2
は常には成り立たないことを示せ。

x=1を代入すれば成り立たないことがわかる。


11.11.f(x)、g(x)、h(x)は3次関数で
f(1)=g(2)=h(3)=1
f(2)=g(3)=h(1)=2
f(3)=g(1)=h(2)=3
が成り立っている。このとき、f(x)+g(x)+h(x)は最大値も最小値も持たないことを
示せ。

前すれ参照
38すべてを超えしもの:2006/12/25(月) 16:59:54
12.次の問いに答えよ。
(1)y=1/xとy=1/x^2(x>0)とで囲まれた部分の面積を求めよ。
(2)y=1/x^2とy=1/x^3(x>0)とで囲まれた部分の面積を求めよ。
(3)y=1/x^nとy=1/x^[n+1](x>0)とで囲まれた部分の面積を求めよ(nは自然数)。
(4)AV=SEXで、A=1/a V=1/v、S=1/s E=1/e X=1/xのときavをs e xで表せ。
(5)以上によりeの値を求めよ。
※(4)のみ解決。

(1)(2)(3)∞
(5)e=lim[n→∞](a+1/n)^n
 eの値は∫[0,1]e^xdx+1
39すべてを超えしもの:2006/12/25(月) 17:01:03
14.
n>m

15.
光は回折するから0
40すべてを超えしもの:2006/12/25(月) 17:01:39
まともな問題を作れるやつはいないのか。
41132人目の素数さん:2006/12/25(月) 20:11:05
君が作問者になればよい
42132人目の素数さん:2006/12/25(月) 21:19:14
ここまで愚問作れるのもある意味才能だよな。
43132人目の素数さん:2006/12/25(月) 21:39:27
場違いなのはわかってますが、もう、ここにくるしかありませんでした。
どなたか教えていただけませんでしょうか?
V=π*h^2(3r-h)/3
での、hを求めるh=の式を出したいのですが全然分かりません。
お願いします。
44132人目の素数さん:2006/12/25(月) 21:52:22
>>43
向こうでも答えたけど、こっちでも答えておこう。
中学レベルの問題ではない。

問題文を確認しろ。それで合ってるのなら、カルダノの解法でぐぐれ
45132人目の素数さん:2006/12/25(月) 21:57:52
時空は曲がっているから、時間がたてば全部の面が照らされる
46体積馬鹿一代:2006/12/25(月) 22:20:19
>>18

15 ttp://up.spawn.jp/file/up61164.jpg
   考え中。。。

16 400π^3/3

>>32 解かれる前から愚問扱いされちゃ困る
47体積馬鹿一代:2006/12/25(月) 22:22:25
16は 400π/3だった スマソ
48132人目の素数さん:2006/12/25(月) 23:01:31
>>43よ、前のスレで俺の君に対する指摘は見てくれたかね。
一応俺なりの解を考えてみたよ。

V=π*h^2*(3r-h)/3と書き直せる。
V = (3πrh^2-πh^3) / 3
3V = 3πrh^2-πh^3
πh^3 - 3πrh^2 + 3V = 0となるのでこれをhについての3次方程式と
みなすことができる。しかし、中学校の問題でこんな問題を出すとは
考えられない。難しいのではなくて、問題としての意義がないから。
だからたぶん>>43は問題を間違って全部書かなかったのか、問題が
間違ってるのかのどちらかだろう。
49132人目の素数さん:2006/12/26(火) 00:08:39
>>18
15
点A(3,5,7)とする。
yz平面上のP(0,a,b)について、線分PAに遮蔽物がある条件を求める。
遮蔽物はxy平面上にあるからb<0で、線分PAとxy平面の交点Qは
Q(-3b/(7-b),(7a-5b)/(7-b),0)
よって遮蔽物がある条件は
b<0,√(-3b(7-b)) + 5b≦7a≦b(4b-35)/(7-b)
2曲線が交わるのは(a,b)=(-1,-7/2),(0,0)のとき
b(4b-35)/(7-b) - √(-3b(7-b)) - 5b を-7/2から0までbで積分して7で割ればよい
計算して
(7/4)(9+log(3/2)+(√3)log(2+√3)) ≒ 20.45
計算自信なす(´・ω・`)
50132人目の素数さん:2006/12/26(火) 00:14:30
>>15
ところで4はa,b,cがすべてxz平面上にあるけど
B(0,1,0)の間違いだろ?
51132人目の素数さん:2006/12/26(火) 01:24:09
x<y<zを満たす実数x,y,zについて次の問いに答えよ。
(1)1/x+1/y+1/z=1を満たすものは無限に存在することを証明せよ。
(2)1/x^2+1/y^2+1/z^2=1を満たすものは無限に存在することを証明せよ。
(3)1/x^n+1/y^n+1/z^n=1を満たすものは無限に存在することを証明せよ。

52132人目の素数さん:2006/12/26(火) 01:29:07
↑17番

18番
方程式f(x)=0はn次方程式(nは自然数とする)であることがわかっている。
(1)この方程式は、高々n個しか解を持たないことを示せ。
(2)この方程式の解のうち虚数の数が奇数ということはありうるか。
(3)kを0以上の実数とし、g(x)をn個の単項式a(1)x^(b(1))〜a(n)x^(b(n))の和からなる
方程式とする。b(1)〜b(n)はすべて実数であり、max{b(1),b(2)、・・・b(n)}=kであるという。
このとき、g(x)=0は高々[k]個しか解を持たないことを示せ。ただし[x]はxを超えない最大の整数
とする。
53132人目の素数さん:2006/12/26(火) 01:31:07
酷い問題だな
54132人目の素数さん:2006/12/26(火) 01:32:18
19番

平面上にn個の点A(1)〜A(n)がある。ただし、これらの点は互いに異なる点であり、
どの3点も同一直線上にない。任意のkとiについて、線分A(k)A(i)でこの平面を切断
したとき、この平面は最大でいくつの部分に分かれるか。


55132人目の素数さん:2006/12/26(火) 01:35:55
20
1辺の長さがそれぞれ1,2,3である直方体Sを考える。
(1)Sの内部または周に2点A,Bをとる。線分ABの最大値はいくらか。
(2)Sの内部または周に4点A,B,C,Dをとる。AB+BC+CD+DAの最大値を求めよ。
56132人目の素数さん:2006/12/26(火) 01:37:34
21番
(1)a^2+b^2-2abを因数分解せよ。
(2)a^3+b^3+c^3-3abcを因数分解せよ。
(4)a^4+b^4+c^4+d^4-4abcdを因数分解せよ。
57132人目の素数さん:2006/12/26(火) 01:39:54
21訂正(4)→(3)
 すまない
22番

定数a,bに対し、(y-b)^2=(x-a)^2を満たす曲線をC(a,b)とする。
(1)C(1,1)を図示せよ。
(2)C(a,b)とC(c,d)が無数に交点を持つためのa,b,c,dの必要十分条件を求めよ。
58132人目の素数さん:2006/12/26(火) 01:41:02
23番(京大向け)
自然数n,rについて(n≧r)、
(n!)C(r!)とnCrの大小を比較せよ。
59132人目の素数さん:2006/12/26(火) 01:44:10
24番
f(x)=∫[0,x]xsin(t^2)dt、g(x)=∫[0,x]x(sint)^2dtとする。
(1)f'(x)、f''(x)、g'(x)、g''(x)をそれぞれ求めよ。
(2)0≦x≦aを満たすすべてのxに対して、f(x)≦g(x)が成立するようなaの最大値を求めよ。
60132人目の素数さん:2006/12/26(火) 01:46:04
24訂正
f(x)=∫[0,x]xsin(t^2)dt、g(x)=∫[0,x]x(sint)^2dtとする。

f(x)=∫[0,x]tsin(t^2)dt、g(x)=∫[0,x]t(sint)^2dtとする。

ごめん。
61132人目の素数さん:2006/12/26(火) 01:50:23
25番
(1)a,bは実数である。xの方程式ax=bを解け。
(2)a,bは虚数である。xの方程式ax=bを解け。
(3)a,b,cは実数である。xの方程式ax^2+bx+c=0を解け。
(4)a,b,cは複素数である。xの方程式ax^2+bx+c=0を解け。
62132人目の素数さん:2006/12/26(火) 01:51:13
25番問題文追加

「ただし、適宜文字を置いて説明してもよい。」
をつけたしてください
63132人目の素数さん:2006/12/26(火) 02:13:23
>>52
単項式だからb(i),kは非負整数。
64132人目の素数さん:2006/12/26(火) 02:28:27
17:明らか。非常にクダラナイ。消えろクズ。

18:クダラナイ。しかも問題が間違っている。消えろクズ。
(1)因数定理から明らか。
(2)f(x)=x−iとすれば、f(x)=0の虚数解は奇数個。
(3)>g(x)をn個の単項式a(1)x^(b(1))〜a(n)x^(b(n))の和からなる方程式とする。
この文章は間違い。「g(x)=0」という式に対しては”方程式”という用語が使えるが、
「g(x)」そのものに対しては”方程式”という用語は使えない。あと、「単項式」という
用語はxの指数が非負整数のときしか使わない。例えば、x^2は単項式だがx^(1/2)は単項式
ではない。さらに、g(x)=x^(√2)−1とすれば、方程式g(x)=0の解(複素数解)は無限個ある
から、解が高々[k]個なんて大間違い。

19:ここは「★東大入試作問者になったつもりのスレ★ 第八問 」です。消えろクズ。

20:非常にクダラナイ。消えろクズ。
(1)直方体の対角線の長さが最大。つまり√14が最大。
(2)AB+BC+CD+DA≦4√14 等号はA=C=P,B=D=Qのとき。ただし、
直方体の対角線を1つ選び、その両端点をP,Qとした。
65132人目の素数さん:2006/12/26(火) 02:51:07
>>62
>>29>>50についてレスよろ
あと解答書いてるの合ってるかよろ
66132人目の素数さん:2006/12/26(火) 04:01:50
21から25は解けないのかw
67132人目の素数さん:2006/12/26(火) 07:07:01
a^2+b^2-2ab=(a+b)^2.
a^3+b^3+c^3-3abc=(a+b+c)^3.
a^4+b^4+c^4+d^4-4abcd=(a+b+c+d)^4.
68132人目の素数さん:2006/12/26(火) 11:41:40
既出問題や入試問題をそのまま貼ったり、
理解できていない(解けていない)のに貼ったり、
分からない問題を質問代わりに貼ったり、
なにかとクズの多いスレですね。

(゚Д゚)≡゚д゚)、カァー ペッ!!
69すべてを超えしもの:2006/12/26(火) 14:23:31
>>64
18の問題の書き方の悪さには同感だが、
17や18(1)や20はどう見ても明らかな問題をどのように説明するかを要求している
のではないだろうかと思う。よってその解答はお世辞にもいいとはいえない。



>>67
右辺を展開してみよ
70132人目の素数さん:2006/12/26(火) 17:18:37
26番
s(n)=1^n+2^n+・・・100^nとする。
(1)s(2)を7で割った余りを求めよ。
(2)あなたの好きなnを1つ決めて(ただしnは自然数)、s(n)を10で割った余りを
2倍した数を答えよ。なお、その答をこの問題におけるあなたの得点とする。
71132人目の素数さん:2006/12/26(火) 17:23:23
27番
3円C1、C2、C3は中心がそれぞれ(a,0)、(0,b)、(a,b)で半径がそれぞれr1、r2、r3(r1>0、r2>0、
r3>0)である。r1+r2+r3=1を満たしている。
(1)うまくr1、r2、r3を定めるとC1、C2、C3のどの2つも共有点を持つような(a,b)の範囲をA
とする。Aの面積を求めよ。
(2)どのようにr1、r2、r3を定めてもC1、C2、C3のどの2つも共有点を持つような(a,b)の範囲
をBとする。Bの面積を求めよ。
72132人目の素数さん:2006/12/26(火) 17:25:16
28番京大向け
sin1°は0.01より大きいことを示せ。
73体積馬鹿一代:2006/12/26(火) 17:40:04
制限時間:今年中 Aは改題。


 頂点:(0,0,2),底面:x^2+y^2≦4/3,z=0の円錐と、円柱y^2+z^2=1の側面との交線をCとする。

 @ 円柱の側面のうちCによって囲まれている曲面Sの面積を求めよ。

 A 円錐の側面と曲面Sとで囲まれた立体Vの体積を求めよ。

 B 円錐の側面のうちCによって囲まれている曲面Tの面積を求めよ。


どうせまたスルーされるんだろうな……
74132人目の素数さん:2006/12/26(火) 17:56:28
糞問解答者にもスルーされる問題
75体積馬鹿一代:2006/12/26(火) 18:45:00
>>74
能書きはいらない

まずは空間ベクトルと円錐曲面の連立方程式を使って交線Cを割り出す
76体積馬鹿一代:2006/12/26(火) 18:53:13
……なんかもういいや
77132人目の素数さん:2006/12/26(火) 20:06:58
29番
2次の正方行列A,B,Xには次のような関係式がある。
AB=BX
BA=AX
BX=XA
このような、A,B,Xを3つ求めよ。
78132人目の素数さん:2006/12/26(火) 20:11:06
29追加
ただしどれも零行列、単位行列の実数倍ではない。

30番
l(n):y=(tannθ)xとする(nは自然数)。
(1)l(m)とl(m+1)(mは自然数)のなす角αを求めよ。ただしαは直角または鋭角である。
(2)l(a)とl(b)のなす角をβ、l(c)とl(d)のなす角をγ(a,b,c,dはそれぞれ異なる自然数)
とするとき、常にβ≠γであることを示せ。
79132人目の素数さん:2006/12/26(火) 20:14:57
31番
楕円C:x^2/a^2+y^2/b^2=1、D:x^2/b^2+y^2/a^2=1(a>0、b>0)は4つの交点を持ち、それらの交点を
P,Q,R,Sとする。
(1)4点P,Q,R,Sを通る円が存在することを証明し、その円の中心の座標、半径を必要ならばa,bを用いて表せ。
(2)あなたの好きなようにP,Q,R,Sを定めて、1/PQ+1/RSの値を必要ならばa,bを用いて表せ。
80132人目の素数さん:2006/12/26(火) 20:35:05
>>65
>>29
すいませんでした。出題ミスでした。もうしわけない。
この問題は無視してください。以後気をつけます。
>>50
いいえ、それで合っていますよ

解答とは>>64のことですか?
17 ×
18(1)×説明がない。証明問題でこれはひどい。(2)○(4点)(3)◎(10点)(出題側にミスがあったから)
19 ×
20 答のみで説明が全くない。どうしてそれが最大なのかという説明
がない。(1)×(2)×

君の得点は14点/80点
81132人目の素数さん:2006/12/26(火) 20:41:57
>>33->>39
2 ◎(20点)
5 ×説明欠ける
6 (1)○(4点)(2)(3)××答になっていない
7 ×答のみ
9 (1)×答のみ(2)○(7点)
10△説明不足、具体的な説明を(3点)
11×答になっていない
12(1)(2)(3)△△△説明不足(3点)(5)×書き換えただけ
14×答のみ
15×題意誤解

君の得点は37点/200点
82132人目の素数さん:2006/12/26(火) 20:56:00
指摘するのも馬鹿らしくなってきた…
>>76
東大生の方ですか?
83132人目の素数さん:2006/12/26(火) 21:15:27
いい加減、体積の問題は専用のスレ作ってそこに掲示してほしいもんだ。
84132人目の素数さん:2006/12/26(火) 21:58:00
だが体積以外のほとんどどれも東大らしくない問題ばかりな件
85132人目の素数さん:2006/12/26(火) 22:12:12
>>84がいいこと言った!
体積ヲタクは別スレ立てて隔離しろ!
86132人目の素数さん:2006/12/26(火) 23:38:49
東大対策にはどんな問題集がいいですか。
理系東大数学があんまり解けません
87132人目の素数さん:2006/12/26(火) 23:44:49
スレ違い
88132人目の素数さん:2006/12/26(火) 23:50:59
>>86
馬鹿か、スレ違いな質問をする時点で、猿未満の知能しかないことに気づけ!
さっさと練炭買いに行け!
89132人目の素数さん:2006/12/27(水) 00:03:16
B**さいころをn個同時に投げるとき,出た目の数の最大公約数が1になる確率を求めよ
C***半径1,高さ1の円錐を底面に垂直で中心から1/2離れた平面で切断する.断面積を求めよ
90132人目の素数さん:2006/12/27(水) 00:08:16
>>89
最初のはA***くらい?
二つめのは、何の中心かわかんない。
91132人目の素数さん:2006/12/27(水) 00:10:44
正弦定理を証明せよ
92132人目の素数さん:2006/12/27(水) 00:11:11
>>89
100年ROMってから出題をお願いします
93132人目の素数さん:2006/12/27(水) 00:12:58
>>91
おまいさんは、さっさとペプ…(ry
94132人目の素数さん:2006/12/27(水) 00:30:17
>90Aでいいかもね,円の中心
>86新数演,過去問を記述で約20年分解いた
東大模試ではSS70いった
95132人目の素数さん:2006/12/27(水) 00:47:35
ペプ…シ?
96132人目の素数さん:2006/12/27(水) 00:49:47
ペプ・・・チド結合
97132人目の素数さん:2006/12/27(水) 00:54:32
>>94
何の中心かわかったけど、出題意図がさっぱりわかんない。
直円錐限定なら減点して遊ぶ、ということ?
98132人目の素数さん:2006/12/27(水) 01:02:32
【ペプシ工場】
 いつのころからか、数学板では考える力の弱い、或いは考えることを放棄した馬鹿の就職先として使われるようになる。
(用例) ペプシキャップを詰めていろ!など
99132人目の素数さん:2006/12/27(水) 01:06:36
意図は式で表すことと(x^2+y^2=(1-z)^2、0≦z≦1をy=1/2の平面で切断)積分出来るか
100132人目の素数さん:2006/12/27(水) 01:10:08
直円錐限定かよ!
101132人目の素数さん:2006/12/27(水) 01:13:27
x^2+y^2=z^2、0≦z≦1 ならさらに簡単に。
102132人目の素数さん :2006/12/27(水) 01:24:20
e~π-1>20を示す
∫[x=0,π] (e^x)dx=e~π-1
∫[x=0,π] (e^x)dx>∫[x=0,3] (e^x)dx+0.14*e~3
=1.14*e~3-1
>21.49
>20
よってe~π-1>20よりe~π>21
103132人目の素数さん :2006/12/27(水) 01:28:10
不完全な状態で書き込んでしまった
今更ながら>>19への答え
>>20の2.7^3.1は試験会場で起算するのは無理だろう

e^π-1>20を示す
∫[x=0,π] (e^x)dx=e^π-1
∫[x=0,π] (e^x)dx>∫[x=0,3] (e^x)dx+0.14*e^3
=1.14*e^3-1
>21.49
>20
よってe~π-1>20よりe~π>21
104132人目の素数さん:2006/12/27(水) 01:29:02
>>102
π=0.3の怨念Part2としては良い問題だね。
言われりゃサルでも分かるけど、試験じゃS級の難問だと思う。
105132人目の素数さん:2006/12/27(水) 01:33:45
あ、、、
π=3.の間違いだ
106132人目の素数さん:2006/12/27(水) 02:28:27
東大のパクリ
107132人目の素数さん:2006/12/27(水) 03:37:44
>>94
4,5,6は1通り、3は2^n-1通り、2は3^n-2通り
∴1-(1/2)^n-(1/3)^n
切断面は0≦z≦1-√(x^2+1/4)=f(x)
S=∫[-√3/2,√3/2]f(x)dx=√3-1/2∫[0,π/3](1/cos^3θ)dθ
108132人目の素数さん:2006/12/27(水) 04:01:02
e~π≒e~3×e~0.14 e~x>1+x+x~2 /2 +x~3 /6
109132人目の素数さん:2006/12/27(水) 07:01:34
>>19
 π - 4.5*log(2) > 3.1415 - 4.5*0.6932 > 0.022 ,
 e^π > 2^4.5 = 16√2 > 16*1.4 = 22.4

>>70

[26](1)
 m^n + (m+1)^n + …… + (m+6)^n ≡ (-3)^n + (-2)^n + (-1)^n + 0^n + 1^n + 2^n + 3^n (mod 7)
 nが2 or 奇数のとき ≡0 (mod 7),
 ∴ (与式) ≡ 1^2 + 2^2 =5 (mod 7).

>>72

[28]
 f(x) = sin(x) - (3/π)x とおく。
 f(0) = f(π/6) = 0,
 f "(x) = -sin(x) <0 (0<x<π/3)
∴ f(x) >0
 sin(1゚) = sin(π/180) > 1/60.
110132人目の素数さん:2006/12/27(水) 07:04:23
双子の素数の小さいほうの逆数の和は双子の素数が無限にあっても
発散しないこと とか?
111132人目の素数さん:2006/12/27(水) 09:34:11
e^πの問題は99年に東大で出てるけど…
112すべてを超えしもの:2006/12/27(水) 15:49:54
番号付けで糞問連発しているものよ、お前の問題がどれだけ幼稚か私が教えてやる。
22
(y-b)^2=(x-a)^2⇔y-b=±(x-a)より2直線。
(1)y=±(x-1)+1=x,-x+2を図示。
(2)2直線が無数に交点を持つから一致する。
傾きが常に1か-1だからa=c、b=dが条件。

24
(2)a=0


25
適宜文字を置いてよいので、答の集合をAとおく。
(1)〜(4)x=A


29
A   B   X
(0,0) (0,0) (0,0)
(0,1) (0,1) (0,1)

30
(1)y=(tan(n+1)θ)x、y=(tannθ)xのなす角よりθ。
(2)n=1、2、3のときβ=少しは考えろ




113132人目の素数さん:2006/12/27(水) 16:00:25
a,b,cを正の実数として、abc=1を満たすとき
(a^2+b^2)c/(a^3+b^3) +(b^2+c^2)a/(b^3+c^3)+(c^2+a^2)b/(c^3+a^3)
の取り得る値の範囲を求めよ
114132人目の素数さん:2006/12/27(水) 16:40:55
>>113
ニヤニヤ…
115132人目の素数さん:2006/12/27(水) 16:57:35
>>114
何だよ?
116132人目の素数さん:2006/12/27(水) 17:02:38
32番
(1)a,bの連立方程式
{a^2+a+1=b-1
{b^2+b+1=a-1
を解け。
(2)x,yの連立方程式
{y=x^3-1
{x=y^3-1
を解け。
117132人目の素数さん:2006/12/27(水) 17:06:19
33番
正方形ABCDは1辺の長さが1である。線分AB、BC、CD、DA上にP,Q,R,Sをとり、
AP=x、BQ=y、CR=z、DS=wとおく。
(1)四角形PQRSが正方形のとき、x,y,z,wに成り立つ関係式を求めよ。
(2)四角形PQRSが長方形のとき、x,y,z,wに成り立つ関係式を求めよ。
118132人目の素数さん:2006/12/27(水) 17:10:38
34番
∫[0,1]e^(-x^2)dx>0.6を示せ。

119132人目の素数さん:2006/12/27(水) 17:18:28
35番
ある町には、4つの八百屋A,B,C,Dがあり、品揃えは次のようである。但し種類の同じものは
すべて同じものとする。
A キャベツ4、レタス2、トマト3、
B キャベツ3、レタス3、トマト3
C キャベツ2、レタス4、トマト4
D キャベツ3、レタス2、トマト2
5人の客P,Q,R,S,Tがこの店を自由に行き来して買い物をする。
買い方は何通りあるか。買い方は全くの自由である。またここで買い方
とは、店に行く順序も考慮するものとする。
120132人目の素数さん:2006/12/27(水) 18:08:02
※愚問を解くと相手に調子づかせます
121132人目の素数さん:2006/12/27(水) 19:58:57
>>116
ちゃんと解けることを確認したか?
122132人目の素数さん:2006/12/27(水) 21:18:39
彼は天才良問作成者なんだから解いたに決まってんじゃん
123132人目の素数さん:2006/12/27(水) 22:40:52
>73x=tで切断?
124132人目の素数さん:2006/12/27(水) 23:54:03
ダメだ体積馬鹿の問題とは相性が悪いorz
125132人目の素数さん:2006/12/28(木) 14:29:19
36番
(1)f(n)=12345678910111213・・・・n(nは自然数)は,自然数を小さい順に並べていってできる数
である.n≧2のとき、f(n)の中に平方数は存在しないことを証明せよ。
(2)n≧2において、f(n)は立方数となることがあるか。
126132人目の素数さん:2006/12/28(木) 14:33:27
37番
(1)
n
婆(k+1)を計算せよ。
k=1

(2)
n
婆(k+1)(k+2)を計算せよ。
k=1

(3)
n
婆^4を求めよ。
k=1
127132人目の素数さん:2006/12/28(木) 14:35:47
38番
楕円x^2/2+y^2=1を原点を中心に45°回転してできる曲線をCとする。
Cをx軸の周りに1回転してできる立体の体積を求めよ。
128132人目の素数さん:2006/12/28(木) 14:43:33
39番
(1)次の等式を示せ。
 log(a)b=log(c)b/log(c)a
また、このときa,b,cに成り立つ条件をすべて答えよ。
(2)a,bに、(1)の条件が成立しているとき、a^log(a)bの値を求めよ。
129132人目の素数さん:2006/12/28(木) 14:46:12
40番
a,b,cは正の実数で、a+b+c=1が成り立っているとき、
ab(a+b)+bc(b+c)+ca(c+a)の最大値を求めよ。
130132人目の素数さん:2006/12/28(木) 14:50:46
41番
∫[0, π]sinxf(t)dt+sinx+cosx=f(x)を満たすような関数f(x)を求めよ。
131132人目の素数さん:2006/12/28(木) 15:10:34
1レスにつき4〜5問くらい いっぺんに書いてくれ。1レスに1問とかウザイ。
132132人目の素数さん:2006/12/28(木) 15:46:34
逆に俺は一問一レスでないと読む気がせん。
133132人目の素数さん:2006/12/28(木) 17:14:37
ついに、クズの質問スレになったな
>>126
134132人目の素数さん:2006/12/28(木) 18:28:50
逆に糞問君に出題したらどうかな?
135132人目の素数さん:2006/12/28(木) 19:55:06
ab(a+b+c-c)+bc(b+c+a-a)+ca(c+a+b-b)
136132人目の素数さん:2006/12/28(木) 20:01:41
abc(a+b+c-c)/c+abc(b+c+a-a)/a+cab(c+a+b-b)/b
abc(1/c+1/b+1/a-3)
137132人目の素数さん:2006/12/28(木) 20:12:46
あれ?>>121はスルーっすかw?
138132人目の素数さん:2006/12/28(木) 20:22:33
>>137
解けないと思うなら証明してください
そうすれば正解です
139132人目の素数さん:2006/12/28(木) 20:47:49
↑x^3-x-1=0を解いて(高校レベルの道具で
140132人目の素数さん:2006/12/28(木) 21:11:54
カルダノやればいいじゃん
高校範囲で簡単に導けるだろ
141132人目の素数さん:2006/12/28(木) 21:40:16
>118
[34]
 exp(-x^2) > 1-x^2,
 (左辺) > ∫[0,1] (1-x^2)dx = [ x -(1/3)x^3 ](x=0,1) = 2/3.
 ( 0.746824… ぐらい)

>>126
[37]
(1) k(k+1) = {k(k+1)(k+2) - (k-1)k(k+1)}/3 より n(n+1)(n+2)/3.
(2) k(k+1)(k+2) = {k(k+1)(k+2)(k+3) - (k-1)k(k+1)(k+2)}/4 より n(n+1)(n+2)(n+3)/4.
(3) (1/30)n(n+1)(6n^3 +9n^2 +n-1).


>>128
[39]
(1) 左辺に log_a(x) = ln(x) / ln(a) を代入する。a>0,a≠1, b>0, c>0, c≠1.


>>130
[41]
∫[0,π] f(t)dt =C とおくと、f(x) = (C+1)sin(x) +cos(x).
∫f(t)dt = -(C+1)cos(t) +sin(t) より 2(C+1)=C, C=-2.
 f(x) = -sin(x) + cos(x).
142132人目の素数さん:2006/12/28(木) 23:19:03
30番θ=0で矛盾だけど、成り立つことを示せってどういうこと?

それと君この問題解いてみてくれ、入試問題としては簡単な方だから
xy平面においてx≦y≦x^2-100xの領域に存在する格子点の個数は何個か?
143132人目の素数さん:2006/12/28(木) 23:26:45
はいはい
144132人目の素数さん:2006/12/29(金) 00:27:38
G=(ab+bc+ca)-3abc-r(a+b+c-1)
Ga=b+c-3bc-r=0
ab+bc+ca=2/3-r
ab+ac-3abc-ar=0
2(2/3-r)-9abc-r=0
abc=4/27-r/3,3abc=4/9-r
K=2/3-r-4/9+r=2/9
145132人目の素数さん:2006/12/29(金) 00:30:36
42番
媒介変数tを用いて
x={t^2+t}/{t^2+t+1}、y={t^3+t^2}/{^3+t^2+1}
と表される曲線をCとする。tは実数全体を動くものとする。
(1)Cの概形を書け。
(2)変曲点の座標を求めよ。
(3)C、x軸、x=1で囲まれる部分の面積と1/2との大小を調べよ。
>>142
x^2-100x=xを解くとx=0,101
0≦x≦101で考えればよい。f(x)=x^2-100x
x=a→f(a)というふうに書くことにする。
x=1→-99、x=2→-196、x=3→-291、x=4→-384、x=5→-475、x=6→-564
x=7→-651、x=8→-736、x=9→-819、x=10→-900、x=11→-979、x=12→-1056
x=13→-1131、x=14→-1204、x=15→-1275、x=16→-1344、x=17→-1411
x=18→-1476、x=19→-1539、x=20→-1600、x=21-1659、x=22→-1716
x=23→-1771、x=24→-1824、x=25→-1875、x=26→-1924、x=27→-1971
x=28→-2016、x=29→-2059、x=30→-2100、x=31→-2139、x=32→-2176
x=33→-2211、x=34→-2244、x=35→-2275、x=36→-2304、x=37→-2331
x=38→-2356、x=39→-2379、x=40→-2400、x=41→-2419、x=42→-2436
x=43→-2451、x=44→-2464、x=45→-2475、x=46→-2484、x=47→-2491
x=48→-2496、x=49→-2499、x=50→-2500、
よって、格子点の数は、
2×(99+196+291+384+475+564+651+736+819+900+979+1056+1131
+1204+1275+1344+1411+1476+1539+1600+1659+1716+1771+1824+2875+1924
+1971+2016+2059+2100+2139+2176+2211+2244+2275+2304+2331+2356+2379
+2400+2419+2436+2451+2464+2475+2484+2491+2496+2499)+2500+100+(1+2
+3+4+5+6+7+8+9+10+11+12+13+14+15+16+17+18+19+20+21+22+23+24+25+26
+27+28+29+30+31+32+33+34+35+36+37+38+39+40+41+42+43+44+45+46+47+48
+49+50+51+52+53+54+55+56+57+58+59+60+61+62+63+64+65+66+67+68+69+70
+71+72+73+74+75+76+77+78+79+80+81+82+83+84+85+86+87+88+89+90+91+92
93+94+95+96+97+98+99+100)+1=171801個
146132人目の素数さん:2006/12/29(金) 00:36:31
>>145
それだとx^2-100x≦y≦x
x≦y≦x^2-100xだと無限個
147132人目の素数さん:2006/12/29(金) 00:36:56
すげぇ・・・
148132人目の素数さん:2006/12/29(金) 00:37:54
>>146
 なぜ無限と言い切れる?
149132人目の素数さん:2006/12/29(金) 00:48:00
>>140
高校の範囲じゃ無理。
150132人目の素数さん:2006/12/29(金) 00:49:09
x≦y≦x^2-100x
x^2-99x
151132人目の素数さん:2006/12/29(金) 00:49:49
>>148
n<0について点(n,n)は条件を満たす
152132人目の素数さん:2006/12/29(金) 00:52:20
半径rの円の中の整数座標の数をrであらわせとか?
153132人目の素数さん:2006/12/29(金) 00:57:21
>>149
お前が無理なだけ
154132人目の素数さん:2006/12/29(金) 01:01:02
>xy平面においてx≦y≦x^2-100xの領域に存在する格子点の個数は何個か?
x≦y≦x^2-100xだから、特にx≦x^2-100xが成り立つ。これを満たすxの範囲は
x≦0またはx≧101となり、求める格子点は無限個。
155132人目の素数さん:2006/12/29(金) 01:02:15
>>149
カルダノの公式自体は高校範囲外だけど導出は高校範囲で容易
156132人目の素数さん:2006/12/29(金) 01:04:03
カルダノを使うような悪問なんて東大は出さないよ。
157132人目の素数さん:2006/12/29(金) 01:04:30
そんなのどこも出すわけない
158132人目の素数さん:2006/12/29(金) 01:07:00
>>153
>>155
無理じゃないというならカルダノの公式を使う入試問題があるの?
159132人目の素数さん:2006/12/29(金) 01:08:32
>>158
アホですか?
出題の有無と公式の導出に何の関係がある
160132人目の素数さん:2006/12/29(金) 01:09:53
>>159
無理だからだされない
161132人目の素数さん:2006/12/29(金) 01:12:26
>>160
それはギャグで言ってるんだよね?
ちょっとびっくりしたよw
162132人目の素数さん:2006/12/29(金) 01:14:37
>>145
無限個が正解
俺が順番ミスっただけだけど
試験でもいちいち調べるんすかw?
163132人目の素数さん:2006/12/29(金) 01:15:00
>>159
>>161
ない?
164132人目の素数さん:2006/12/29(金) 01:15:51
>>162
お前は遊ばれてることに気付いたほうがいい
165132人目の素数さん:2006/12/29(金) 01:17:17
>>163
お前はアホなだから不精するな
意味不明だぞ
166132人目の素数さん:2006/12/29(金) 01:23:00
171802.
167132人目の素数さん:2006/12/29(金) 01:24:45
いい感じに糞スレになってきたな
糞出題者のせいでアホしか残ってないしな
168132人目の素数さん:2006/12/29(金) 01:31:09
誤魔化して終わりか
169132人目の素数さん:2006/12/29(金) 01:32:25
自己紹介イラネ
170132人目の素数さん:2006/12/29(金) 01:34:20
アホが恥を晒しただけだろ
誤魔化せてもいない
171132人目の素数さん:2006/12/29(金) 01:37:39
カルダノの公式を高校の範囲で証明しなさい
172132人目の素数さん:2006/12/29(金) 01:59:09
>>113
くだらねぇ問題はここへ書け ver.3.14(47桁略)0582
http://science4.2ch.net/test/read.cgi/math/1163520000/977
分からない問題はここに書いてね269
http://science4.2ch.net/test/read.cgi/math/1167229683/103
173132人目の素数さん:2006/12/29(金) 06:59:26
直接じゃないがカルダノ使う問題あったよ

わかりやすくPowerPointにした
http://p.pita.st/?fmps3oij
2時間掛かったぜ…
174132人目の素数さん:2006/12/29(金) 09:10:34
たかがこんなものに2時間もかけるなんて究極の素人だな。
175132人目の素数さん:2006/12/29(金) 14:42:32
>>129

40番
a,b,c≧0, a+b+c = s とする。
 (左辺) = s(ab+bc+ca) -3abc = (1/4)(s^3 -F_1 -3abc) ≦ (1/4)s^3.
ここに F_1 = a(a-b)(a-c) + b(b-a)(b-c) + c(c-a)(c-b) ≧ 0. (←Schur不等式) >>38 >>399-401
等号は (a,b,c) = (s/2,s/2,0) のとき。

http://science4.2ch.net/test/read.cgi/math/1105911616/896-897
不等式スレ2
176175:2006/12/29(金) 15:23:02
>175 のアンカーは間違いですた。取消しまつ。
177132人目の素数さん:2006/12/29(金) 15:25:11
43番
曲面:x^2+y^2-z^2=1と立方体|x|≦1,|y|≦1,|z|≦1とで囲まれた立体の体積を求めよ
178132人目の素数さん:2006/12/29(金) 17:02:12
>>177
2^3=8
179132人目の素数さん:2006/12/29(金) 17:22:59
x^3-x-1=0
(x^2-1)=1/x
(x+1)(x-1)=1/x
x=z
zz-1=z^-1
r^3e^i2t-1=e^-it
r^3=(1+e^-it)/e^2it
180132人目の素数さん:2006/12/29(金) 17:28:11
zz-1=z^-1
r^3e^i2t-r=e^-it
r^3=(r^2+1)^.5
arctan(rsint/(r+rcost)=2t
181132人目の素数さん:2006/12/29(金) 18:02:37
懲りずにまた来たよwwリアル工房か?ww
182132人目の素数さん:2006/12/29(金) 18:50:52
>>181
お前の脳内面白いな
183132人目の素数さん:2006/12/29(金) 19:05:14
このスレ別所でwatchされて,住民馬鹿にされてるぞ
184132人目の素数さん:2006/12/29(金) 19:26:37
Let D be a symmetric connexion on a manifold M. Prove the Bianchi identity: Ri;jhk|p+Ri;jkp|h+Ri;jph|k=0
185132人目の素数さん:2006/12/29(金) 19:28:20
connection が connexion になってるよこの本、bishop
186132人目の素数さん:2006/12/29(金) 19:29:10
糞問ばかり出してる奴は馬鹿にされても仕様がない
187132人目の素数さん:2006/12/29(金) 19:35:16
フランス語か。カルタンめ
188132人目の素数さん:2006/12/29(金) 19:41:40
高校生も終わりならリーマン幾何ぐらい冬休み中に終わらせておけよ。
189132人目の素数さん:2006/12/29(金) 20:28:16
45 中学問題?
190132人目の素数さん:2006/12/29(金) 20:51:06
44番
[x]はxをこえない最大の整数を表す。
f(x)=cosx-cos[x]とおく。
(1)-5≦x≦5の範囲で、y=f(x)の概形を描け。
(2)f(x)のとりうる値の範囲を答えよ。
>>162
2×(99+196+291+384+475+564+651+736+819+900+979+1056+1131
+1204+1275+1344+1411+1476+1539+1600+1659+1716+1771+1824+2875+1924
+1971+2016+2059+2100+2139+2176+2211+2244+2275+2304+2331+2356+2379
+2400+2419+2436+2451+2464+2475+2484+2491+2496+2499)+2500+101+(1+2
+3+4+5+6+7+8+9+10+11+12+13+14+15+16+17+18+19+20+21+22+23+24+25+26
+27+28+29+30+31+32+33+34+35+36+37+38+39+40+41+42+43+44+45+46+47+48
+49+50+51+52+53+54+55+56+57+58+59+60+61+62+63+64+65+66+67+68+69+70
+71+72+73+74+75+76+77+78+79+80+81+82+83+84+85+86+87+88+89+90+91+92
93+94+95+96+97+98+99+100)+1=171802個
の間違い。
そろばんやってたから計算はかなり得意。これも1分かからなかった。
あと速記も。
打つのは苦労した
試験でもやると思うお
191132人目の素数さん:2006/12/29(金) 20:53:16
45番
3つの整数a,b,cはa^2+b^2+c^2=10000を満たしている。
(1)(a,b,c)は有限個であることを示せ。
(2)a,b,cのうち少なくとも1つは4の倍数であることを示せ。
(3)これを満たす(a,b,c)は何個あるか。ただし、(1,2,3)と(2,1,3)のように順番が違うものも
異なるものとして数えよ。
192132人目の素数さん:2006/12/29(金) 20:54:43
46番(45が物足りない人向け)
3つの整数a,b,cはa^b+b^c+c^a=10000を満たしている。
(1)(a,b,c)は有限個であることを示せ。
(2)これを満たす(a,b,c)は何個か。ただし、(1,2,3)と(2,1,3)のように順番が違うものも
異なるものとして数えよ。
193132人目の素数さん:2006/12/29(金) 20:57:40
47番
台形ABCDはAD//BCで、AB=CD=1、AD=a、BC=bであるという。
(1)3≦a≦5のとき、bのとりうる値の範囲を求めよ。
(2)台形ABCDは円に内接することを証明せよ。
(3)台形ABCDの外接円の半径を求めよ。
194132人目の素数さん:2006/12/29(金) 21:01:52
48番(47が物足りない人向け)
1辺の長さがa+b、a+c、a+d、b+c、b+d、c+d、である六角形ABCDEFがある。
ただし、a,b,c,dはすべて実数である。
(1)a,b,c,dが満たすべき条件を答えよ。
(2)六角形ABCDEFがある円に内接するためのa,b,c,dの満たすべき
条件を答えよ。また、そのときの外接円の半径をa,b,c,dで表せ。
195132人目の素数さん:2006/12/29(金) 21:03:46
49番

     ∞
無限級数蚤^(2k+1)/(2k+1)をaを用いて簡単にせよ。ただし、-1<a<1である。 
     k=0
196132人目の素数さん:2006/12/29(金) 21:10:49
50番
f(x)=x^3-x-1=0とおく。
(1)f(x)=0の実数解は1つであることを示せ。
(2)f(x)=0の実数解をαとする。αはe/2より小さいことを示せ。
(3)∫[0,β]f(x)dx=0を満たす実数β(>0)を求めよ。
(4)(3)より、-∫[0,α]f(x)dx=∫[α,β]f(x)dxが成り立つことがわかる。
これにより、αを決定せよ。
197132人目の素数さん:2006/12/29(金) 21:16:56
51番
a,bを異なる正の実数とする。
ある有限の点集合を、どの二点間の距離もaまたはbのどちらかになる集合とする。
この時、点集合として考えられるものを全て求めよ。
198132人目の素数さん:2006/12/29(金) 21:23:03
52番(51番が物足りない人向け)
a,b,cを異なる正の実数である。
ある有限の点集合を、どの2点間もaまたはbまたはcのどちらかになる集合とする。
このとき、点集合として考えられるものをすべて求めよ。
199132人目の素数さん:2006/12/29(金) 21:25:27
52訂正
冒頭「a,b,cを異なる正の実数とする。 」にしてください
53番
3つの実数a,b,cはa^6+b^6+c^6=1を満たしている。
このとき、F=a^4bc+b^4ca+c^4ab-a^3b^2c-b^3c^2a-c^3a^2bの最大・最小を求めよ。
200132人目の素数さん:2006/12/29(金) 21:27:30
54番
任意の実数xに対して、cos(sinx)>sin(cosx)が成り立つことを示せ。
201132人目の素数さん:2006/12/29(金) 22:16:33
>191
45番
 絶対値は (48,60,64) (36,48,80) (0,60,80) (0,28,96)

>195
49番
 (1/2)log{(1+a)/(1-a)} = arctanh(a) のマクローリン展開
 ∵ log(1-x) = -納n=1,∞) (x^n) /n.

>200
54番
 |sin(x)| ≦1, |cos(x)| ≦1,
 |sin(x)| + |cos(x)| = √{1+sin(2x)} ≦ √2 < π/2.
 cos(sin(x)) = cos(|sin(x)|) ≧ cos(π/2 -|cos(x)|) = sin(|cos(x)|) ≧ sin(cos(x)).
202132人目の素数さん:2006/12/29(金) 23:24:07
>184
共変微分は一般に可換でない。
すなわち 任意のベクトル Va↑ に対して ∇m∇n Va↑ -∇n∇m Va↑ は 0↑とはならない。
これを計算すると
 [∇m,∇n] Va↑ ≡ ∇m∇n Va↑ -∇n∇m Va↑= Ra;bmn Vb↑.     (25.1)
となる。ここで
 Ra;bmn ≡ ∂mΓa;nb - ∂nΓa;mb + Γa;mt・Γt;nb - Γa;nt・Γt;mb  (25.2).
は Riemann-Christoffel の曲率テンソル(curvature tensor)と呼ばれる。
まず (25.1) から
 ∇L[∇m,∇n] Va↑ = Ra;bmn|L Vb + Ra;bmn Vb|L.
また少し面倒な計算をすると
 [∇m,∇n]∇L Va↑ = Ra;bmn Vb|L + Rt;Lmn Va|t.
となる。そこで両者の差を求めると
 [∇L[∇m,∇n]] Va↑ = Ra;bmn|L Vb + Rt;Lmn Va|t.         (25.6)
L,m,nを順々におきかえれば
 [∇m[∇n,∇L]] Va↑ = Ra;bnL|m Vb + Rt;mnL Va|t,
 [∇n[∇L,∇m]] Va↑ = Ra;bLm|n Vb + Rt;nLm Va|t.
これら3個の式を加えると 左辺はJacobiの恒等式のために0となる。したがって
 0 ≡ (Ra;bmn|L + Ra;bnL|m + Ra;bLm|n)Vb + (Rt;Lmn + Rt;mnL +Rt;nLm) Va|t.
時空内の任意の点において、Vb および Va|t の成分の値は全く任意に、また互いに独立に選びうる。
したがって 上の恒等式は
 Ra;bmn|L + Ra;bnL|m + Ra;bLm|n = 0,               (25.7)
 Rt;Lmn + Rt;mnL + Rt;nLm = 0.                  (25.8)
という2組の恒等式に分かれる。
(25.7)はBianchiの恒等式と呼ばれ、これは非常に重要な恒等式である。

(参考書)
 内山龍雄: 「相対性理論」 岩波全書292, p.147-151 (1977) 第Y章 25 曲率
 山内恭彦・内山龍雄・中野薫夫: 「一般相対性および重力の理論」 裳華房 物理学選書10, p44-47 (1967)
     第U章 §13 Riemann-Christoffel テンソル
203132人目の素数さん:2006/12/29(金) 23:46:14
204132人目の素数さん:2006/12/30(土) 00:21:00
http://jbbs.livedoor.jp/bbs/read.cgi/study/4125/1160364871/
>>200ここの住人はここよりはるかにレベル高いからここに書き込んでみては?
205132人目の素数さん:2006/12/30(土) 00:21:12
>177

43番
xy平面で切れば 半径√(1+z^2) の円と正方形の共通部分だから
S(z) = 4|z| + (1+z^2){π-4arctan(|z|)},
∫[0,z] S(z')dz' = (8/3)z^2 + {z+(1/3)z^3}{π-4arctan(z)} + (4/3)log(1+z^2).
V = 2∫[0,1] S(z')dz' = (8/3){2+log(2)} = 7.181725815…


>193

47番
(1) a-2 < b < a+2.
(2) a=b, AB//CD すなわち平行4辺形になる場合は除くんだろうな…
(3) √{(1+ab)/(4-(a-b)^2)}.
206132人目の素数様:2006/12/30(土) 00:33:53
>>205
すまない。47番でa≠bがぬけていました。
55番(54番が物足りない人向け)
(1)y=cosxは周期関数であることを示し、基本周期は2πであることを証明せよ。
(2)y=sin(cosx)+cos(sinx)は周期関数であることを示せ。ただし、基本周期は求めなくてよい。


207132人目の素数様:2006/12/30(土) 00:38:33
表現が悪かったです。書き直します。
47番 改
台形ABCDはAD//BCで、AB=CD=1、AD=a、BC=bであるという。また、台形ABCDは
平行四辺形ではないという。
(1)3≦a≦5のとき、bのとりうる値の範囲を求めよ。
(2)台形ABCDは円に内接することを証明せよ。
(3)台形ABCDの外接円の半径を求めよ。

56番
(1)e^π >π^eを証明せよ。ただし、e=2.7・・・・、π=3.1・・・・である。
(2)a=e^π、b=π^eとおく。a^bとb^aの大小を比較せよ。
208132人目の素数さん:2006/12/30(土) 00:51:58
>>207

56番は
f(x) = log(x)/x が e<x で単調減少であることを示せばお終い、だよね。
209132人目の素数さん:2006/12/30(土) 00:55:16
>e^π >π^e

これ、散々既出なんだけど……
210132人目の素数さん:2006/12/30(土) 01:27:50
上で話題が出てたけど、3次方程式はCardanoの公式知らなくても
3倍角の公式知ってりゃ解ける場合も多いですよ。

答えが実解だという前提は必要だけど。
211132人目の素数さん:2006/12/30(土) 01:39:44
>196

50番
α = {(1-b)/2}^(1/3) + {(1+b)/2}^(1/3) ≒ 0.337726750973 + 0.986991206271 = 1.324717957245
b = √(1 - 4/27) ≒ 0.9229582069909
212132人目の素数さん:2006/12/30(土) 06:41:58
>>207
a,b,cがこの順に等差数列をなしsina,sinb,sincがこの順に等比数列をなすときは
どのようなときか
213132人目の素数さん:2006/12/30(土) 09:43:17
【50周年】月刊大学への数学【栄光の歴史】 [大学受験]
2007年大河ドラマ「風林火山」 Part18 [大河ドラマ]
┠AA厨┨布袋寅泰 98beats┠ファッキン┨ [邦楽男性ソロ]
2ch版Wikipediaを作ろう [資料室]
アク禁になった人の為にレス代行するスレその51@規制議論 [2ch規制議論]
214132人目の素数さん:2006/12/30(土) 15:41:38
>207-208

56番
 e^y > 1 + y,  (y≠0)
 e^{(x/e) -1} = x/e,  (x≠e)
 e^(x/e) > x,
 e^x > x^e.
 e^(1/e) > x^(1/x).
 f(e) = 1/e > log(x)/x = f(x).
215132人目の素数さん:2006/12/30(土) 16:18:20
>19

 20+π > e^π > 23

は無理かな?
216132人目の素数様:2006/12/30(土) 16:45:50
>>212

(sinb)^2=sina sinc
2b=a+cを解いて
nを整数として、
b=a+nπ
c=a+2nπ


a,b,cがこの順に等比数列をなしsina,sinb,sincがこの順に等差数列をなすときは
どのようなときか。

217132人目の素数さん:2006/12/31(日) 01:59:05
>216

 f(x) = sin(e^x) とおくと
 f '(x) = (e^x)cos(e^x) > 0,
 f "(x) = (e^x)cos(e^x){1-(e^x)tan(e^x)}
e^x =y とおけば
 f "(x) = y・cos(y){1-y・tan(y)}
 零点は y0 = 0.86033358901938…
 y < y0 では f "(x) > 0,
 y0 < y < π/2 では f "(x) < 0.
 a < y0 < c.

(例)
 a = 0.375285736340592…
 b = 0.750571472681184…
 c = 1.501142945362369…
は等比数列(公比2)で
sin(a) = 0.366538393973000…
sin(b) = 0.682056788895123…
 sin(a) = 0.997575183817245…
は等差数列
218132人目の素数様:2006/12/31(日) 02:09:59
57番
f(x)は3次関数である。
(1)f(f(x))は6次関数であることを証明せよ。
(2)f(f(f(x)))は9次関数であることを証明せよ。
(3)f(f(x))=a(x^6+x^5+x^4+x^3+x^2+x+1)(aは正の実数)となるf(x)は存在するか。
219132人目の素数様:2006/12/31(日) 02:13:38
58番
すべての正の実数xにたいし、x^a≦a^xを満たすようなaの値の範囲を求めよ。
220132人目の素数様:2006/12/31(日) 02:18:01
59番
数列sinx、sin2x、sin3x、・・・、sin(nx)、・・・
を考える。
この中からある3つを取り出して、適当な順に
並べると等差数列になるような3つが存在する
ようなxの条件を求めよ。
221132人目の素数様:2006/12/31(日) 02:23:01
60番
4数α、β、γ、δはすべて実数で
α+β=γ+δ
α^2+β^2=γ^2+δ^2
α^3+β^3=γ^3+δ^3
α^4+β^4=γ^4+δ^4
を満たしている。(α、β、γ、δ)をすべて求めよ。
222132人目の素数さん:2006/12/31(日) 03:03:48
B**関数f_n(x)=f_[n-1](x)+1/(x+n) f_0(x)=0とする
lim[n→∞]f_n(n)を求めよ
B**(n^2+8n+7)/(2n+1)が整数となるような整数nを全て求めよ
C***2n+1人が硬貨を1枚ずつ投げ,表裏の少ない方を勝者とする
a_n=C[2n,n]/4^nとし,このときの勝者の人数の期待値E(n)をa_nを用いて表せ
223132人目の素数さん:2006/12/31(日) 03:16:42
>>222
f_1(x)=1/(x+1)
f_2(x)=1/(x+1)+ 1/(x+2)
f_n(x)=1/(x+1) + 1/(x+2) + ・・・ + 1/(x+n)
f_n(n)=1/(n+1) + 1/(n+2) + ・・・ + 1/(n+n)
=Σ[k=1,n]{1/(n+k)}=(1/n)*Σ[k=1,n]{1/(1+k/n)}
lim[n→∞]f_n(n)=∫{0to1}(1/(x+1))dx=log2
224132人目の素数さん:2006/12/31(日) 04:10:34
>>18
問題15
壁の範囲は y≧x^2, x≧y^2 の間違いでしょ

A(3,5,7), Q(x,y,0) とし、
QA↑ 方向の単位ベクトルを u↑、
x軸、z軸方向の単位ベクトルをそれぞれ i↑、k↑とする。

点Qの周囲の微小面積 dxdy が作る影の面積 dS は
dS = (3^2/(3-x)^2)(u↑・k↑)/(u↑・i↑) dxdy
= 63/(3-x)^3 dxdy

S = ∫[0≦x≦1, x^2≦y≦√x] 63/(3-x)^3 dxdy
= 63∫[0≦x≦1] ((√x)-x^2)/(3-x)^3 dx
= (7/8)*(33 - 72*log(3/2) - (√3)*log(2+√3))
≒ 1.33478995


>>49
被積分関数間違ってる
(1/7)∫[-7/2,0] {√(-3b(7-b)) + 5b - b(4b+35)/(7-b)} db
≒ 1.33478995
225132人目の素数さん:2006/12/31(日) 13:01:40
双子素数(5・7、11・13)は無限にあることを証明せよ。
226132人目の素数さん:2006/12/31(日) 14:43:47
>689文系チックに
@‐Aよりcos(2x+ρ)=-sin(2x-θ)B
x=0を代入しても=は成り立つのでBは
cosρ=-sin(-θ)=sinθ
cos30゚=sin60゚,cos0゚=sin90゚…のように足したら90゚になるので90゚
227214:2006/12/31(日) 16:40:30
>219

58番
 x^a ≦ a^x,
 x^(1/x) ≦ a^(1/a),
 f(x) = log(x)/x ≦ log(a)/a = f(a),   >214
∴ a=e.
228132人目の素数さん:2006/12/31(日) 18:07:33
>>127

38番
 x=(X+Y)/√2, y=(Y-X)/√2 を与式に代入して
 Y(+) = {X + 2√(3-2X^2)} /3,
 Y(-) = {X - 2√(3-2X^2)} /3.
ただし |X|≦√(3/2).
 Y(-) の零点は Xo = 2/√3,
V = 2π∫[0, Xo] Y(+)^2 dX + 2π∫[Xo, √(3/2)] {Y(+)^2 - Y(-)^2}dX
  = (2π/9)∫[0,Xo] {12 -7X^2 +4X√(3-2X^2)}dX + (8π/9)∫[Xo,X1] 2X√(3-2X^2) dX
  = (2π/27)[ 36X -7X^3 -2(3-2X^2)^(3/2) ](X:0→Xo) + (2π/27)[ -4(3-2X^2)(3/2) ](X:Xo→X1)
  = (2π/9√3)(24 -56/9 -2/9 +6) + (2π/9√3)*36
  = 2(2 + 50/81)π/√3 + (8/81√3)π
  = (16/3√3)π.

http://science4.2ch.net/test/read.cgi/math/1166904000/127
東大入試作問者スレ8
229132人目の素数さん:2007/01/01(月) 00:19:21
>>14
 C****

V(0) = (4/3)π.
V(1) = V(0) + π∫[7/8,1] {(1/4)-(x-1)^2 -(1-x^2)}dx + π/12
  = V(0) + π∫[7/8,1] (2x -7/4)dx + π/12
  = V(0) + π[ x^2 -(7/4)x ](x=7/8,1) + π/12
  = V(0) + π/64 + π/12
  = V(0) + (19/(24・8))π.
V(2) = V(0) + (19/(24・8))π + (19/(24・8^2))π,
V(n) = V(0) + (19/(24・8))π + (19/(24・8^2))π + … + (19/(24・8^n))π 公比1/8の等比級数,
   = V(0) + (19/(24・7))π * {1 - (1/8)^n},
Lim[n→∞) V(n) = V(0) + (19/(24・7))π = (81/56)π.
230132人目の素数さん:2007/01/01(月) 22:38:17
n!≧n(n-1)の関係を上手く使った問題作れないかな
231132人目の素数さん:2007/01/02(火) 00:42:31
   ∧_∧  ノロウイルス撒いてやんよ
  ( ・ω・)ノ>゚+。:.゚
  C□ / ゚。:.゚.:。+゚
  /  . |
  (ノ ̄∪

   ∧_∧   彡
  ( ・ω+。:.゚ 彡
  C□゚。:.゚.:。+゚
  /  . |
  (ノ ̄∪

   <⌒/ヽ-、 ___
 /<_/____/
232132人目の素数さん:2007/01/02(火) 01:17:17
>>230
そんな甘すぎる評価式では、たとえ問題を作っても別解が出来てしまう。
233132人目の素数さん:2007/01/02(火) 15:05:39
>>199

53番
 -0.358831700744616… ≦ F ≦ 0.40354678557889…
等号成立は
左側: (a,b,c) = (-0.60180089, 0.719539815, 0.96622635)
右側: (a,b,c) = (-0.74912440, -0.55987256, 0.961974165) 
と その巡回置換 と 反転。
234132人目の素数さん:2007/01/02(火) 17:03:46
>>218
>>220
>>221
解いた上で出してるか?
235132人目の素数さん:2007/01/02(火) 18:18:19
解けないなら解けないことを証明してください。正しければ正解ですみたいなこと言ってた。
実際解いてるとは思えないが。
236132人目の素数さん:2007/01/02(火) 23:18:40
このすれってちょっと難しい問題が出たら「解いた上で出してるか?」が口癖なのなwww
237132人目の素数さん:2007/01/02(火) 23:23:27
(x^3)^3=x^9って難しいのか?
238132人目の素数さん:2007/01/02(火) 23:35:11
>>236
問題不成立の場合もあるから言ってるんだろ
239132人目の素数さん:2007/01/03(水) 00:10:41
せめて答書いてから煽れよw
240132人目の素数さん:2007/01/03(水) 00:36:27
>218

57番
 f(x) がn次の多項式 なら
 f(x) = a・x^n + b・x^(n-1) + …… + r (a≠0)
 f(f(x)) = a^(n+1)・x^(n^2) + …… + f(r), だから (n^2)次式
 f(f(f(x))) = a^((n+1)n+1)・x^(n^3) + …… + f(f(r)), だから (n^3)次式

>221

60番
 α^i + β^i = γ^i + δ^i  …… (i)
とおく。ただし i = 1,2,3,4.
 2*(2) - (1)^2 より
 (α-β)^2 = (γ-δ)^2
∴ α-β = γ-δ または α-β= -(γ-δ).
これと(1)から
 α=γ,β=δ または α=δ, β=γ.
なお、
 (3) = (1)*{3*(2) - (1)^2} /2,
 (4) = (2)^2 - {(1)^2 -(2)}^2 /2.
も成立する。
(実数に限る必要もないかと…)
241132人目の素数さん:2007/01/03(水) 08:14:56
>>238
神大入試であったな。問題不成立。
もちろん頭のいい奴はそれを証明した奴もいたわけだが。
242132人目の素数さん:2007/01/03(水) 12:58:36
>>241
問題キボンヌ
243132人目の素数さん:2007/01/03(水) 19:21:20
次の方程式で表されるxy平面上の3直線l_1, l_2, l_3を考える。
l_1 : x+y-1=0
l_2 : x-y+1=0
l_3 : x+k=0
(ただしkは0でない定数)
このとき、次の各問いに答えよ。
(1) 1次変換fによりl_1がl_2に、l_2がl_3にうつされるとき、この1次変換fを表す行列Mをkを用いて表せ。
(2) 3直線l_1, l_2, l_3でつくられる3角形の重心が、(1)の1次変換fにより原点にうつされるとき、kの値を求めよ。
(82 神戸大)
244132人目の素数さん:2007/01/03(水) 20:59:40
B**中心Oの単位円に内接する正n角形の頂点をX_1,X_2,…X_nとする
このとき↑OXの異なる2つのベクトルの内積の和を求めよ
B***x>0とし,xの小数部分が1/x等しいものを小さい順にx_1,x_2,…とし,S(n)=Σ[k=1,n]x_kとする
lim[n→∞]S(n)/n^2を求めよ
245132人目の素数様:2007/01/03(水) 22:24:42
>>240
57番は問題ミスを指摘したので正解。
60番は説明不足。部分点5点。

61番
3数a^2+2b^2、b^2+2c^2、c^2+2a^2(a,b,cは整数)について、次の問いに答えよ。
(1)「3数がすべてp(pは自然数)の倍数になる」ようなa,b,cが存在するような最大のpを求めよ。
(2)「3数がすべてq(qは自然数)の倍数になる」が常に成り立つような最大のqを求めよ。
246132人目の素数さん:2007/01/03(水) 22:30:14
>>243
ありがとう
247132人目の素数さん:2007/01/03(水) 22:31:32


248132人目の素数様:2007/01/03(水) 22:34:13
62番
f(x)=x^3-3px^2+qx+1について、y=f(x)は定点P(1,a)を通る(a>0)。Pにおけ
るy=f(x)の接線lとy=f(x)の交点をQとする。ただし、Qのy座標は負である。
PQとy=f(x)とで囲まれる面積をS1とする。x=1、x軸、PQとで囲まれる面積をS2
とする。また、接線lに平行な直線でlとは異なるy=f(x)上の接線をmとする。
mとx軸との交点をRとし、mとx軸、y=f(x)とで囲まれる面積をS3とする。
S1=S2=S3=S(定数)が成り立つとき、a,p,q、Sの値をそれぞれ求めよ。
249132人目の素数様:2007/01/03(水) 22:38:18
63番
N=100!、M=10!、L=20!とおく。
(1)pを自然数とする。N/(M^p)が整数となるようなpの最大値を求めよ。
(2)qを自然数とする。N/(L^q)が整数となるようなqの最大値を求めよ。
(3)r,sを自然数とする。N^10/(M^r L^s)が整数となるようなr,sのうち、
rsを最大にするr,sの値を求めよ。
250132人目の素数さん:2007/01/03(水) 22:45:08
素数様氏ね
251132人目の素数さん:2007/01/03(水) 22:52:01
197 名前: 132人目の素数さん [sage] 投稿日: 2006/12/29(金) 21:16:56
51番
a,bを異なる正の実数とする。
ある有限の点集合を、どの二点間の距離もaまたはbのどちらかになる集合とする。
この時、点集合として考えられるものを全て求めよ。

198 名前: 132人目の素数さん 投稿日: 2006/12/29(金) 21:23:03
52番(51番が物足りない人向け)
a,b,cを異なる正の実数である。
ある有限の点集合を、どの2点間もaまたはbまたはcのどちらかになる集合とする。
このとき、点集合として考えられるものをすべて求めよ。

---

ここら辺の流れをみて、ろくに考えずに問題出してるんだなぁって気がした。
51番は解けたんだろうか……
252132人目の素数さん:2007/01/03(水) 23:29:50
素数様もそろそろネタ切れだなww
253132人目の素数さん:2007/01/04(木) 02:50:41
素数様は東大後期系で難しすぎて誰も解けない
254132人目の素数さん:2007/01/04(木) 04:27:52
>東大後期系

むしろ30〜40年前の入試問題みたいな雰囲気だけど。
255132人目の素数様:2007/01/04(木) 14:14:06
64番
(1)1次関数y=f(x)で、f(α)=β、f(β)=αを満たす。f(x)をα、βを用いて表せ。
(2)2次関数y=g(x)で、g(α)=β、g(β)=γ、g(γ)=αを満たす。g(x)をα、β、γを用いて表せ。
(3)3次関数y=h(x)で、h(α)=β、h(β)=γ、h(γ)=δ、h(δ)=αを満たす。h(x)をα、β、γ、δを用いて表せ。
(4)n次関数y=i(x)で、異なる定数p_1、p_2、p_3、・・・・、p_nに対して、
i(p_n)=p_1
i(p_j)=p_[j+1](j=2,3,4,・・・・,n-1)
を満たす。このようなi(x)は唯1つ存在することを証明せよ。
256132人目の素数様:2007/01/04(木) 14:17:54
素数pのうち、p^p+2が素数であるとき、pを超素数ということにする。
(1)最も小さな超素数を求めよ。
(2)超素数は無限に存在することを証明せよ。ただし、素数が無限に存在することは証明なしに使ってよい。
257132人目の素数様:2007/01/04(木) 14:21:23
↑65番

66番
f(x)=x^2-2x+5、g(x)=-x^2+x/2a+a+4とする。(a≠0)
(1)任意の整数α、βに対し、f(α-β)>g(α+β)が成立するようなaの値の範囲を求めよ。
(2)任意の整数α、βに対し、f(α√2)>g(β√3)が成立するようなaの値の範囲を求めよ。
258132人目の素数様:2007/01/04(木) 14:27:23
64番
(1)〜(3)でα〜δはすべて異なる定数。


67番
P(x)をn次式の整数係数多項式とする。このとき、どのようなP(x),自然数mに対しても、P([m]√3)=√2となり得ないことを証明せよ。ただし、[m]√3=3^(1/m)である。
259132人目の素数さん:2007/01/04(木) 15:23:04
>>249
N=100!=2^97*3^48*5^24*7^16*11^9*13^7*17^5*19^5*...
M=10!=2^8*3^4*5^2*7
L=20!=2^19*3^8*5^4*7^2*11*13*17*19 だから

(1) 8p≦97, 4p≦48, 2p≦24 ,p≦16 より 求める最大値は12
(2) 19q≦97,8q≦48 4q≦24, 2q≦16, q≦9,q≦7,q≦5,q≦5より
求める最大値は5
(3) 8r+19s≦97かつ4r+8s≦48かつ2r+4s≦24かつr+2s≦16かつs≦5
のときのrsの最大値を考える。
(r,s)=(5,3)のときrs=15で最大。
260132人目の素数さん:2007/01/04(木) 15:54:19
>>255
64番(4)
定数の数が一個足りなくないですか?
(修正案)
異なる(n+1)個の定数p_1、p_2、p_3、・・・・、p_n、p_[n+1]に対して、
i(p_[n+1])=p_1
i(p_j)=p_[j+1](j=2,3,4,・・・・,n)

261132人目の素数さん:2007/01/04(木) 20:24:17
>>260
素数様が間違えるわけないだろ!!氏ね!!
262132人目の素数さん:2007/01/04(木) 20:52:43
A=B,B=CならばB=Cとなるが

俺=人間、イケメン=人間ならば俺=イケメンとならないのはなぜか
263132人目の素数さん:2007/01/04(木) 20:54:35
お前と人間はイコールでないから。
264132人目の素数さん:2007/01/04(木) 21:00:02
すげえ、正論じゃん。
265132人目の素数さん:2007/01/04(木) 22:10:17
>A=B,B=CならばB=Cとなるが
266132人目の素数さん:2007/01/04(木) 22:17:08
>>262
次の命題を考えてみろ!
「俺∈人間、イケメン∈人間 ⇒ 俺=イケメン」

それにしても、いつからココは高校生の自由研究スレになったんだ?
267132人目の素数さん:2007/01/04(木) 22:22:52
はじめからじゃね?
268132人目の素数さん:2007/01/04(木) 22:32:00
問題を作れない解けない雑魚がwww
269132人目の素数様:2007/01/05(金) 02:26:03
>>261
 ありがとう。でも>>260のほうが正しかった。深く謝罪します。


65番
4定点A,B,C,Dは同一円周上にあるという。この円をPとし、Pと同じ半径で、Pとは異なる円であり、かつA,Bを通る円をQとする。Qは唯1つしか存在しないことを証明せよ。
270132人目の素数様:2007/01/05(金) 02:31:08
66番
(1)3つの正の数a,b,cが与えられたとき、その3つの数を3辺の長さとする三角形が存在するためのa,b,cの必要十分条件を求めよ。
(2)4つの正の数a,b,c,dが与えられたとき、その3つの数を4辺の長さとする四角形が存在するためのa,b,c,dの必要十分条件を求めよ。ただし、その四角形をABCDとすると、AB=a,BC=b,CD=c,DA=dになるとする。
271132人目の素数様:2007/01/05(金) 02:37:03
66番(2)3つの数→4つの数

67番
(1)極限値lim[x→∞]sin(logx)/xを求めよ。
(2)不等式∫[0,10]sin(logx)/xは0より大きいことを示せ。ただし,e>2.718、π>3.141を使ってもよい。
(3)極限lim[x→∞]exp(sinx)sinx/x^2の収束・発散を調べよ。収束するときは極限値を求めよ。ただし、exp(x)=e^xである。
272132人目の素数さん:2007/01/05(金) 02:56:00
>>271
大学入試問題で広義積分は出ないし収束もしてない。
273132人目の素数さん:2007/01/05(金) 03:09:47
素数様は高校何年生ですか?
274132人目の素数さん:2007/01/05(金) 04:18:56
>>273
高校中年生です。
275132人目の素数さん:2007/01/05(金) 14:11:16
>>255
64(4)
i(x)がn-1次以下になる可能性が排除できません。
素数様教えてください。
276132人目の素数様:2007/01/05(金) 16:16:30
>>275
馬鹿野郎!!!!!問題文を嫁!!!n次関数と書いてあるだろうが!!お前はいつもそうだ!!お前は100点を取れる実力は持っているのにつまらないミスでそれを逃してしまう!!もっと落ち着け!!!!お前ならできるはずだ!頑張れ!!

>>272
 しまった。広義積分は範囲外だった!!収束はする。
 67番改
67番
(1)極限値lim[x→∞]sin(logx)/xを求めよ。
(2)不等式∫[1,10]sin(logx)/xは8/5より大きいことを示せ。ただし,e>2.718、π>3.141を使ってもよい。
(3)極限lim[x→∞]exp(sinx)sinx/x^2の収束・発散を調べよ。収束するときは極限値を求めよ。ただし、exp(x)=e^xである。

68番
 a,b,cを3辺とする三角形の3頂点がすべてy=x^2上にあるようにできないためのa,b,cの条件を求めよ。
277132人目の素数様:2007/01/05(金) 16:28:42
>>273
 俺は浪人生2だ。

67番改2
(1)極限値lim[x→∞]sin(logx)/xを求めよ。
(2)不等式∫[1,10]sin(logx)/xdxは8/5より大きいことを示せ。ただし,e>2.718、π>3.141を使ってもよい。
(3)極限lim[x→∞]exp(sinx)sinx/x^2の収束・発散を調べよ。収束するときは極限値を求めよ。ただし、exp(x)=e^xである。
 
69番
 5000個のおもりがあり、4999個は1000gで、1個は999gであるという。天秤を使って、この999gのおもりを見つけたい。ここで、方法はおもり
を無作為に二つとって、天秤の両側に1つずつ乗せるという方法のみとする。999gのおもりを見つけるまでに天秤を使う回数の期待値を求めよ。ただし、皿の重さは等しいものとし、天秤は正確であるとする。


278132人目の素数さん:2007/01/05(金) 16:35:00
67の(1)と(3)は瞬殺。(2)はcos(log 10)の評価が必要になるので面倒。
279132人目の素数さん:2007/01/05(金) 17:53:36
素数様はどこの大学志望ですか?数学は得意な方ですか?
280132人目の素数さん:2007/01/05(金) 23:27:06
n次関数と書いてあるからこそn次の係数が0でないことを
示さなくちゃいけないってことも分からないのか。
(成り立たないから示せるわけないけど)
281132人目の素数さん:2007/01/05(金) 23:33:32
70
pを2以上の整数とする。
αがcosαπ=1/(2p) , 0<α<1をみたすとき
αが無理数であることを示せ。
282132人目の素数さん:2007/01/06(土) 00:16:27
(RRBW)p_n (RBWW)q_n (RBBW)r_n (RRBB)s_n (RBBB)t_n (BBWW)u_n (BBBW)v_n (BBBB)w_n
p_n+1=1/4p_n q_n+1=1/4q_n r_n+1=1/2r_n+1/2p_n+1/2q_n+1…v_n+1=1/4r_n u_n+1=1/4q_n
白2の確率はΣ[k=1,n]u_k
白1の確率はΣ[k=1,n]v_k以下略
283132人目の素数さん:2007/01/06(土) 00:35:58
284132人目の素数さん:2007/01/06(土) 01:34:39
二回で終わる確率より白2が小さいはずない
285282:2007/01/06(土) 02:54:50
>284
白2(1-4(1/4)^n)/6
白1(1-6(1/2)^n+8(1/4)^n)/3 って出たか?
286132人目の素数様:2007/01/06(土) 16:02:32
71番
y=x^2+ax+bが-1<x<1、2<x<3の範囲で常に正となるような(a,b)の範囲を図示せよ。
287132人目の素数様:2007/01/06(土) 16:07:15
72番
f(x)=ax^3+bx^2+cx+d(a>0)とする。f(-1)=2,f(0)=1,f(2)=3であり、y=f(x)はx軸と3点で交わっている。
この3点をx座標の小さい順にA,B,Cとする。線分ABとy=f(x)で囲まれた面積をS1、線分BCとy=f(x)で囲まれた面積をS2とするとき、
S1+S2のとりうる値の範囲を求めよ。
288132人目の素数様:2007/01/06(土) 16:11:16
73番
2次の正方行列A,B,Cがある。これらはすべて逆行列を持つ。
「AB=BA、AC=CA、BC=CB」がすべて成り立つことがわかっている。
(1)A,B,Cに成り立つ関係式を求めよ。
(2)
(1 0)=A
(1 2)
のとき、自然数nにたいし、A^n+B^n+C^nの値を求めよ。
289132人目の素数様:2007/01/06(土) 16:13:40
行列Aは、ある自然数nに対して、A^n=Oを満たしている。
(1)A^2を求めよ。
(2)Aを求めよ。
(3)(2)で求めたAのうち、逆行列を持つものをBとする。
B+2B^2+3B^3+4B^4+・・・+100B^100を求めよ。
290132人目の素数様:2007/01/06(土) 16:18:42
↑74番
75番
(1)任意の自然数nに対して、n(n+1)(n+2)/6は自然数であることを証明せよ。
(2)任意の自然数nに対して、nとn-1は互いに素であることを証明せよ。
(3)任意の自然数nに対して、2^n+3^n+5^nを割り切る最大の自然数を求めよ。
(4)pを無理数とする。2^pを有理数にするpは存在するか。
(5)pを無理数とする。2^p+3^p+5^pを有理数にするものは存在するか。
(6)pを無理数とする。p^pを有理数にするものは存在するか。
(7)pを無理数とする。p√2+p^2√3+p^3√5を有理数にするものは存在するか。
291132人目の素数さん:2007/01/06(土) 16:33:05
>>289
つまり、Bは逆行列を持ちつつも、B^n = 0を満たす奇特な行列な訳だな?
292132人目の素数さん:2007/01/06(土) 17:42:44
素数様センター前ですよ!多浪になりますよ!
293132人目の素数さん:2007/01/06(土) 19:20:41
素数様!四角形に外接円が存在する条件教えてください!
294132人目の素数さん:2007/01/06(土) 21:50:29
各辺長が1桁の整数で、かつ一つの角が60゚であるような、正三角形でない三角形をすべて求よ。
295132人目の素数さん:2007/01/07(日) 00:54:45
B**a^b=b^c=c^aを満たす組(a,b,c)を求めよ
B**p!+q!=r!を満たす組(p,q,r)を求めよ
B**1/a+1/b=1/cを満たす素数の組(a,b,c)は存在しないことを示せ
296132人目の素数さん:2007/01/07(日) 00:56:33
>>295
a,b,c
p,q,rの条件は?
297132人目の素数さん:2007/01/07(日) 01:00:11
1番目0以上,2番目は自然数
298295:2007/01/07(日) 01:07:34
すまんどっちも自然数(^_^;)
299132人目の素数さん:2007/01/07(日) 03:36:21
>293

 外接円の中心をOとおけば
 ∠ABC + ∠ADC = (1/2)∠(O-弧ADC) + (1/2)∠(O-弧ABC) = (1/2)*360゚ = 180゚

http://mathworld.wolfram.com/PtolemyInequality.html

>294
 A=60゚, cos(A) =1/2.
 第二余弦定理より
 a^2 = b^2 +c^2 -2bc・cos(A) = b^2 +c^2 -bc.

>295
(1) a=b=c の他には…
(2) min(p,q,r)=s とし、各項をs!で割る。項の一つが1になる。
(3) 与式より a,b>c.
 両辺にabcを掛けると (a+b)c = ab.
 左辺はcの倍数だが、右辺はそうでない(矛盾)。 
300295:2007/01/07(日) 04:16:01
正解です、久々に解いてくれた人が現れた〜
(1)はa<bとして矛盾を導きます
301132人目の素数さん:2007/01/07(日) 17:00:38
>>197-199, >>251

51番
 正8面体(6) 1 : √2, (正方形も含む),
 正5角形(5) 1 : (1+√5)/2,
 2等辺3角形(3).

52番
 正20面体(12) 1 : (1+√5)/2 : √{(5+√5)/2},
 正7角形(7) 1 : 2cos(π/n) : 4cos(π/n)^2 -1,
 正6角形(6) 1 : √3 : 2,
 立方体(6) 1 : √2 : √3,
 菱形(4),
 不等辺3角形(3).
302132人目の素数さん:2007/01/07(日) 18:59:22
>271, >277-278

67番
(1) |sin(log(x))| ≦1, | 与式 | ≦ 1/x → 0.
(2) x = exp(t) とおくと、
 I = ∫[0,log(10)] sin(t)dt = [ cos(t) ] = 1 - cos(log(10))
  = 1 - cos(3π/4 - 0.0536…) > 1 - cos(2π/3 -1/18) = 1 - cos(2π/3 -a)
  = 1 - cos(3π/4)cos(a) - sin(3π/4)sin(a) = 1 - {-cos(a)+sin(a)}/√2
  > 1 - {-1 +(1/2)a^2 +a}/√2 = 1 + 611/(81*8*√2) > 1 + 432/(81*8) = 5/3.
(3) -1 ≦ sin(x) ≦ 1, 1/e ≦ exp(sin(x)) ≦ e, | 与式 | ≦ e/x^2 → 0.
303301:2007/01/08(月) 18:42:46
>>197-199, >>251 (追加)

51番
 2つの正4面体の底面を貼り合せた物(5) 1 : √(8/3),
 正3角錐(4)
 正3角形 + 中心(4) 1 : √3

52番
 立方体(8) 1 : √2 : √3,
 正8面体 + 中心(7) 1 : √2 : 2,
 正5角錐(6)  1 : (1+√5)/2 : ?,
 正5角形 + 中心(6) 1 : √{(5-√5)/2} : √{(5+√5)/2},
 正4角錐(5) 1 : √2 : ?,
 正方形 + 中心(5) 1 : √2 : 2
 3方両錐(5) (2つの合同な正3角錐の底面を貼り合せたもの)
304132人目の素数さん:2007/01/08(月) 20:45:22
>255, 258, 260
64番
(1) f(x) = {f(α)(x-β) - f(β)(x-α)}/(α-β) = α+β-x,
(2) g(x) = g(α)(x-β)(x-γ)/{(α-β)(α-γ)} + g(β)(x-γ)(x-α)/{(β-γ)(β-α)} + g(γ)(x-α)(x-β)/{(γ-α)(γ-β)},
(3) h(x) = h(α)(x-β)(x-γ)(x-δ)/{(α-β)(α-γ)(α-δ)} + h(β)(x-γ)(x-δ)(x-α)/{(β-γ)(β-δ)(β-α)}
     + h(γ)(x-δ)(x-α)(x-β)/{(γ-δ)(γ-α)(γ-β)} + h(δ)(x-α)(x-β)(x-γ)/{(δ-α)(δ-β)(δ-γ)},
(4) i(x) = Σ_(j=1〜n+1) i(p_j)・Π_(k≠j) {(x-p_k)/(p_j-p_k)},   但し p_[n+2]=p_1.

http://mathworld.wolfram.com/LagrangeInterpolatingPolynomial.html


>269
65番
 Pの中心を O(P), Qの中心をO(Q) とすると、3辺相等により △ABO(P) ≡ △ABO(Q).
 O(Q)≠O(P) だから O(Q)は 直線ABに関して O(P)と対称の位置にある。


>270
66番
(1) a+b+c > 2max(a,b,c).
(2) a+b+c+d > 2max(a,b,c,d).


>294
 a =7, b=3, c=8, cos(B)=13/14, cos(C)=-1/7.
 a =7, b=5, c=8, cos(B)=11/14, cos(C)=1/7.
と その置換え。
305301:2007/01/09(火) 23:55:46
>>197-199, >>251 (追加)

51番
 @側面が正方形の正3角柱(6) 1 : √2,
 Aアンチプリスム, h=a√(2/3),(6) 1 : √2,

52番
 @ + 中心(7) 1 : 2√(3/7) : 2√(6/7),
 A + 中心(7) 1 : √2 : 2,
 正3角柱(6)
 アンチプリズム(正3角柱の上底or下底だけを60゚回したもの)(6)
306301:2007/01/10(水) 00:48:50
>>197-199, >>251 (追加)

52番
 5方両錐(2つの合同な正5角錐の底面を貼り合わせたもの)で、
 (i) 10面の正3角形からなるもの。 1: √{2(1-√(1/5))} : (1+√5)/2.
 (ii) 2h=a (hは高さ、aは1辺の長さ)。
 (iii) 2h=a√{2(1-√(1/5))} のもの。
307301:2007/01/10(水) 01:09:04
>306 の訂正

 (iii) 2h=a(1+√5)/2 のもの。 1 : √{(7+9/√5)/8} : (1+√5)/2,
308132人目の素数さん:2007/01/10(水) 01:30:53
東大系…微積分重視、計算量多い
京大系…文章も解答量もコンパクト、最近易化
東工大系…問題数少、微積分重視
阪大系…?
309132人目の素数さん:2007/01/10(水) 12:10:56
>197
点集合は面上にあるのか、3次元空間なのかで、答も難易度も変わるが。
平面上の点なら「初等的に解いた高等数学の問題」(東京図書、絶版)に解答が載ってる。つか、過去スレと同じ問題だ。
310132人目の素数さん:2007/01/10(水) 14:22:12
1辺が1の正20面体の対角線の長さを求めよ
311おいら:2007/01/10(水) 22:39:22
宣伝に来ました。

数学の難しい問題・放置された問題・マルチ問題・口の利き方が悪くて撥ねられた問題募集してます。
解きたいんです、説きたいんです!!!
丁寧でかつ谷川九段もびっくりな光速の回答を用意してます!!!
ちなみに回答者もどうぞ。

●難問題・無回答問題・放置問題を質問するスレ●
http://science5.2ch.net/test/read.cgi/math/1168282029/l50
312132人目の素数さん:2007/01/10(水) 23:36:05
>>290
75
(1)どれかひとつ以上2の倍数と3の倍数が因数に含まれる
(2)1より大きい共通因数が存在すると仮定するとその数で割った2整数の差が1より小さくなる
(3)n=1のとき2+3+5=10=2*5 , n=2のとき4+9+25=38=2*19
2^nは偶数で3^n,5^nはだから2^n+3^n+5^nは偶数
よって2^n+3^n+5^nを割り切る最大の自然数は2
(4)log_2(3)=m/lと仮定すると、3^l=2^mとなるのでlog_2(3)は無理数
よってp=log_2(3)とすればよい
(5)存在するだろうけど・・・
(6)pはp^p=2をみたすとする。中間値よりpは存在し、pが自然数でないのは明らか、p>1
p=m/l(m,lは互いに素,m>l>1)と仮定すると
m^m=2^l*l^mよってmとlは1より大きい公約数lをもつので矛盾
よってp^p=2をみたすpは無理数
(7)pはp√2+p^2√3+p^3√5=1をみたすとする。pの実数解は少なくともひとつ存在し、
p=m/lと仮定すると
ml^2 √2+lm^2 √3+m^3 √5=l^3
何乗かしたり整理したりして√2=(係数が有理数の有理式)の形になるので矛盾
よってp√2+p^2√3+p^3√5=1をみたすpは無理数
313132人目の素数さん:2007/01/11(木) 15:40:50
C**sin10゚は有理数か

B***さいころをn個(n≧2)同時に投げるとき,どの二つのさいころをとっても出た目の数の差が2以下になる確率を求めよ
314132人目の素数さん:2007/01/11(木) 22:11:58
素数様の問題をみんなが解かないから、難問題スレに貼りまくってるじゃないか。(´・ω・`)
315132人目の素数さん:2007/01/11(木) 22:44:46
>>281

70番
 背理法による。 α = m/n (0<m<n 自然数) と仮定する。
 cos(nθ) = T_n(cosθ) = (1/2){(2cosθ)^n - n(2cosθ)^(n-2) + …… },  
 cos(n(απ)) = T_n(cos(απ)) = T_n(1/2p) = = (1/2){(1/p)^n - n(1/p)^(n-2) + …… } = {1/(2p^(n-2))}{(1/p^2) + (整数) } ≠ (整数).
これは cos(mπ) = (-1)^m = 整数, に矛盾。

http://mathworld.wolfram.com/ChebyshevPolynomialoftheFirstKind.html

>>197-199, >>251 (追加)
51番
 正4面体 + 中心(5) 1 : √(8/3),
316132人目の素数さん:2007/01/12(金) 01:30:36
>>309
 51番 ここら辺にも解凍↓

「数学の問題」第(3)集, 数セミ増刊, 日本評論社 (1988) 第78問

>>305
 Aは8面の正3角形より成るから、正8面体だお。
317132人目の素数様:2007/01/12(金) 23:35:05
76番
f(x)=log(x)log(1/x)とおく。ただし、x>0とする。
(1)y=f(x)のグラフの概形を図示せよ。
(2)limf(x)/x^pが0でない極限値に収束するとき、pの値とその極限値を求めよ。
   x→∞
318132人目の素数様:2007/01/12(金) 23:38:34

77番

3辺が1、√3、2である三角形の3角は30度、60度、90度であることを証明せよ。
319132人目の素数様:2007/01/12(金) 23:41:11
78番
(a,b)を次の規則1か規則2に従って、順番に変化させていくものとする。
規則1 (a,b)を(a^2+b^2、a^3+3b^3)にする
規則2 (a,b)を(a^2-b^2、a^3-2b^3)にする
(1)最初(1,0)から始めるとき、規則1か規則2を何回か好きなように使用して、
(1000,1001)を作れるか。
(2)最初(p,q)から始めるとき、(r,s)が作れるための4つの実数p,q,r,sの必要十分条件を求めよ。

320132人目の素数様:2007/01/12(金) 23:45:41
79番
(1)f(x)=ax^2+bx+c(a≠0)とする。y=f(x)の頂点が(p,q)でf(a)=b、f(b)=a、f(c)=c
が成り立つという。a,b,c,p,qに成り立つ関係式を求めよ。
(2)f(x)=ax^2+bx+c、g(x)=px^2+qx+r(ap≠0)とする。y=f(x)、y=g(x)が異なる2点のみで交わり、
その2点をP、Qとするとき、PQの長さをa,b,c,p,q,rのうち必要なものを用いて表せ。
(3)(2)でa,b,c,p,q,rをすべて1以上2以下の範囲で動かすとき、PQの長さの最大値を求めよ。
321132人目の素数様:2007/01/12(金) 23:46:40
80番
自然数xに対し、
f(x)=sin1°+sin2°+sin3°+・・・+sinx°
と定義する。y=f(x)に格子点は存在するか。
322132人目の素数様:2007/01/12(金) 23:48:09
81番
2次行列A、Bは
A^2B+B^2A=E
A^2+B^2=B^3
A^5+B^5=A^4B
を満たしている。A,Bとして考えられるものをすべて求めよ。
323132人目の素数さん:2007/01/12(金) 23:55:03
条件が足らない問題はどうすればいいんだよ。
324132人目の素数さん:2007/01/13(土) 03:36:38
77ゴミ問,中学に戻って三平方の定理でも勉強してろカス

80カス問,x=360のとき明らか。だから浪人なんだよカス
325132人目の素数さん:2007/01/13(土) 05:15:23
>>324
釣れますか?
326132人目の素数さん:2007/01/13(土) 05:30:30
いや324の解答は正しいけど。
327132人目の素数さん:2007/01/13(土) 05:54:15
素数様の問題、解く気全くしないんだがお前ら解いてる?
328132人目の素数さん:2007/01/13(土) 06:20:25
俺も解く気しない。
ありがちな問題だし、面白みがない。
329132人目の素数さん:2007/01/13(土) 09:30:37
NG登録してるから見えない
330132人目の素数さん:2007/01/13(土) 10:32:59
>>329
お前は俺か!
331132人目の素数さん:2007/01/13(土) 10:42:51
>>330
いや。俺は俺であって、お前ではないはずだ。
332132人目の素数さん:2007/01/13(土) 10:57:19
>>331
お前は俺で俺はお前だろ?
333132人目の素数さん:2007/01/13(土) 11:55:27
それだったらいる意味なくね?
334132人目の素数さん:2007/01/13(土) 12:16:00



     \    |       ,,...-‐‐‐--、,      l    /
     |、  /  |   ,.べ;;;;;::、--- 、:::;;`ヽ、   ''‐-‐'゙/
   、,_,.! ゙'-'゙(.    //::/´       ``ヾ、      l 
    )    (.  /:,`!ヾ、.      ,      ゙>-    ヽ、_,,
   '゙"`ヽ, /``/::;:゙;゙::!:.:| ``'''‐--‐''゙   '-‐'゙ ゙、       /
     ヽ ヾ /:;'/:.:!:::l  、,r''"゙`'ヾ    ,:',-‐-、,'l      /
   -=,'゙   ./:::/:.:.|::::l  /  (・),.    ヾ,_(・) ,'゙l      (
     `ヽ ,゙:::,'::::;':!::::l   `"´ ''"´     | ̄__,,l,,...,,_   \   ひぃっ! 
     -='゙ l::::l::::;':. l:::::l               ` ゙、.,,_,,.``ヽ、
     __) l!゙,l:::;'-、 ';::::!          ,.-‐‐:、    l `ヽ、  \   つまり329は、いらない子。
      ヽ. !:l:::l/-ヽ.゙;::',    U    /:::::::::::r=‐'''"´`''‐,.-゙'‐-、ヽ.
       ) ゙!::l l"''、l ヾ゙:、   u     l:::::::::::::::`'''‐‐''''‐i'゙  ,,..、  `\
         l:::l,ヽ、_,  ヾ;\       ゙、.,,_ノ/ ̄ ̄ノ /ヽ. `   \     ____
     /    !::l:゙ヽ、. ,、 ヾ、;、         ゙'‐''"7'゙/ /゙ヽ       \ _,./´-、ヽ
   / /  l::l``;::,`´:ヽ、  `ヾ:;、.,__   ::::   / `'''゙  \   ゙、   ヾ;‐、''-、゙;
    /  / l:l ";'::;'::';';! `''‐ 、.,_` ̄      / r'゙´ ̄ ̄``'''‐、 ゙、    ' l  l:::!
.       /  l:!  !:;'::!::::;!     ``'''‐‐---┬'゙   `'''''''''‐-、      ゙、   ./  l:::l
335132人目の素数さん:2007/01/13(土) 13:11:58
>313

C***
 sin(10゚) が有理数でないことを 背理法で示す。
 sin(10゚) = m/n (0<m<n, 自然数) と仮定する。n≧2.
 m/n は既約であるとしてよい。(可約のときは約分すればよい.)
 さて、sin(3θ) = 3sinθ -4(sinθ)^3 より
 1/2 = 3(m/n) - 4(m/n)^3,
 8m^3 = (6m-n)n^2,
 nは偶数となるから、n=2n' とおく。
 m^3 = (3m-n')n'^2,
・n'=1 のとき
 n=2, m=1 しかないが、上式に代入しても不成立。
・n' ≧2 のとき
 n'の素因数をpとすれば p|n', p|m より, m/n' が既約であることと矛盾する。


>318
 http://science5.2ch.net/test/read.cgi/math/1168096620/
算数チャチャチャ
336132人目の素数さん:2007/01/13(土) 14:10:33
>>313

B***
 A: どの目も {1,2,3} に属する。
B: どの目も {2,3,4} に属する。
C: どの目も {3,4,5} に属する。
D: どの目も {4,5,6} に属する。

P = P(A∪B∪C∪D)
 = P(A) + P(B) + P(C) + P(D) - P(A∩B) -P(A∩C) - P(B∩C) - P(B∩D) - P(C∩D) + P(A∩B∩C) + P(B∩C∩D)
 = 4*(3/6)^n - 3*(2/6)^n - 2*(1/6)^n + 2*(1/6)^n
= 4*(1/2)^n - 3*(1/3)^n.


>>318

77番
 a=1, b=√3, c=2 について a^2 + b^2 = c^2,
 3平方の定理の逆から、長さcの辺に対する角Cは直角。
 これは辺が a:b = 1:√3 の長方形ACBXの2辺と対角線である。
 2本の対角線はそれらの中点Mで交わる。
 ∴ BM = CM = AB/2 = c/2,
 題意より、BC = a = c/2,
 ∴ BC = CM = BM,
 ∴ △BCM は正3角形、3つの内角は等しい。
 一方、△の内角の和は180゚ だから 各々 60゚
337315:2007/01/13(土) 17:20:25
>>281

70番 (補足)
 cos(nθ) = (1/2){(2cosθ)^n - n(2cosθ)^(n-2) + (整数)(2cosθ)^(n-4)},
は和積公式
 cos((n+1)θ) = 2cosθcos(nθ) - cos((n-1)θ),
を使って nについての帰納法で出せる。(n>1)
338132人目の素数さん:2007/01/13(土) 18:59:20
個人的に上の人に問題出してほしい。
339132人目の素数さん:2007/01/14(日) 18:14:32
B***nは自然数で,グラフy=0 (0≦x<n),x=n {k-1<y<k (k=1,2…n)}をy軸の周りに回転させた容器がある
容器を水で満たした瞬間,水面の高さをy=hとして,y=k(k=0,1…n-1)の側面から単位時間あたりh-k(h≧k)の水が漏れ出す
水を満たしてからy=n-1の側面から水が出なくなるまでの時間をT(n)とするとき,lim[n→∞]T(n)を求めよ
340132人目の素数さん:2007/01/14(日) 18:17:14
B***って元ネタ大数か何かな
341336:2007/01/14(日) 20:03:47
>>313

B***
 X = max{出た目の数の差} とおくと Xの分布は
 P(X=k) = (M-k){((k+1)/M)^n - 2(k/M)^n + ((k-1)/M)^n},
 P(0≦X≦k) = (M-k)((k+1)/M)^n - (M-1-k)(k/M)^n,
になるらしい。ただし, 6面なので M=6.
342132人目の素数さん:2007/01/14(日) 22:31:22
>340難易度表示は大数表示で書いた
>341さいころ→1からNまでのカードは考えてたけど,max-min≦k(問いはk=2)までは考えてなかったお

私の過去の問題で解答の指針書いて欲しいのある?
343132人目の素数さん:2007/01/14(日) 22:39:45
四次元をtと考えると、t<Cの場合はどうなるか。考えよ。
344132人目の素数さん:2007/01/14(日) 22:42:35
考えますた
345132人目の素数さん :2007/01/14(日) 23:33:22
2点(0,0,1)、(2,2,5)を直径の両端とする球面をS、
2点(-1,0,3)、(3,4,1)を直径の両端とする球面をPとする。
SとPの交わりの円Cの中心と面積を求めよ。
346132人目の素数様:2007/01/15(月) 15:21:44
私の過去の問題で解答の指針書いて欲しいのある?
 
80番をくず問題といったものへ
82番
(1)自然数xにたいし、
f(x)=納k=1,x]k^4 sink°と定義する。y=f(x)上に格子点は存在するか。
(2)自然数xにたいし、
g(x)=納k=1,x]k(sink°)^kと定義する。y=g(x)上に格子点は存在するか。
347132人目の素数様:2007/01/15(月) 15:26:03
>>327>>328
愚痴を言っていいのはちゃんと解けた人だけだと思います

83番
x,は0<x<1、0<y<1、0<x+y<1をすべて満たす。
(1)このときlogx×logyのとりうる値の範囲を求めよ。
(2)x^y+y^xのとりうる値の範囲を求めよ。
(3)x^y×y^xのとりうる値の範囲を求めよ。
348132人目の素数さん:2007/01/15(月) 18:11:50
ありがちか?俺は素数様のような問題一度も見たことないぞ
349132人目の素数さん:2007/01/15(月) 18:49:36
不成立だったり解けないものはくず問題
350132人目の素数さん:2007/01/15(月) 19:04:49
大学入試っぽい問題出すなら高校生が習うはずの範囲を出せよ。
わけわからない記号とか、用語を使うな。

f(t)=t^n (n=1.2.3.…)のラプラス変換を求めよ。
351132人目の素数さん:2007/01/15(月) 19:07:08
じゃあ82番の解答の指針をおながいします
352132人目の素数さん:2007/01/15(月) 19:07:40
353132人目の素数さん:2007/01/15(月) 19:31:11
>>350
>ラプラス変換
習わねーよ
354132人目の素数さん :2007/01/15(月) 19:51:25
>>350  矛盾しとるよ君
355132人目の素数さん:2007/01/15(月) 20:26:43
わかってるよ!
「格子点」なんて習わないだろ、と思って検索かけたら中学校受験でも出てくるのね・・・
356132人目の素数さん:2007/01/15(月) 21:59:40
素数様、早く解答の指針書いてこんなやつらギャフンと言わしてくださいよwww
357132人目の素数さん:2007/01/15(月) 23:17:53
何を知りたいん?
んで素数様ってだれだよ。
解ける問題であればおいらに解けない問題は、まーないね。
358132人目の素数さん:2007/01/15(月) 23:19:39
素数様を知らないとは、新参者か
359132人目の素数さん:2007/01/15(月) 23:26:51
あー  〜番って貼り付けてくる奴か。
それにしてもわかりにくいハンドル。
360素数様:2007/01/16(火) 01:28:04
84番
(1)y=logxとy=e^(-x)の交点をPとする。Pのx座標をαとするとき、αと1.3との大小を比較せよ。
(2)y=e^(x)+aとy=e^(-x^2)の交点の個数を求めよ。
(3)極限値lim[x→∞]x^(x)e^(-x^2)を求めよ。
361素数様:2007/01/16(火) 01:32:23
85番
(1)任意の実数a,b,cに対して、a^3+b^3+c^3≦k(a^4+b^4+c^4)が成り立つようなkの最小値を求めよ。
(2)ある自然数nが与えられている。任意の実数a,b,cに対して、a^n+b^n+c^n≦l(a^(n+1)+b^(n+1)+c^(n+1))が成り立つようなlの最小値を求めよ。
必要ならば、nを用いてもよい。
362132人目の素数さん:2007/01/16(火) 01:36:45
素数様って浪人生なんでしょ?
馬鹿にするわけじゃないけど、自分の勉強がんばったほうがいいよ。
363素数様:2007/01/16(火) 01:40:11
いや、もう決まったからいい。
364132人目の素数さん:2007/01/16(火) 01:41:01
そりゃ、おめでとうございます。
んでどこの大学?
365素数様:2007/01/16(火) 01:50:46
琉球大学
366132人目の素数さん:2007/01/16(火) 01:52:07
国立大学ですか。
おめでとうございます。

あと、問題出すなら a や k といった定数?などの定義をしっかりと書いてください。
367132人目の素数さん:2007/01/16(火) 01:53:49
早く解答の指針な
あと推薦て現役オンリーじゃなかったけ?
368素数様 ◆.Cbdncl/Jc :2007/01/16(火) 01:56:15
>>365は偽者。
369132人目の素数さん:2007/01/16(火) 01:59:54
かんころりん
370132人目の素数さん:2007/01/16(火) 03:35:06
なあ、これ自分で作った問題じゃないよな?
自分で解けない問題まで出題してるだろ?
371132人目の素数さん:2007/01/16(火) 21:34:40
85超糞問a→+0、b=c=0を考えれば存在しないのは明らか、うんこ。

早く指針書けよ。
372132人目の素数さん:2007/01/16(火) 21:41:37
>>371
ワロタ
確かにwww
373132人目の素数さん:2007/01/16(火) 22:55:39
今頃んなこと言っちゃって。
んなの比喩的に表現すれば2秒でわかるじゃん。
現実的に挑戦しようと思って20秒で3回問題文読めばわかるじゃん。

あえて遠まわしに 〜様批判してるのに。

東大って言ったって様々なんだな、蹴ってよかったよ。

……
374素数様:2007/01/16(火) 23:20:19
>>371
正解 よくできましたね

86番

87^92の下4桁を求めよ。また一番上の位を求めよ。ただし、logに関する不等式はどれも知らないものとする。

375132人目の素数さん:2007/01/17(水) 00:01:40
昨日こんにゃくオナヌーしたんだが、今日の夕食のおでんに
縦に穴が開いたコンニャクでてきたぞ・・・・
台所に捨てたはずんなんだが・・・
OrZ
376132人目の素数さん:2007/01/17(水) 00:03:39
はいはい。
377132人目の素数さん:2007/01/17(水) 00:12:26
378132人目の素数さん:2007/01/17(水) 00:21:31
>>375
きもちよかった?
379132人目の素数さん:2007/01/17(水) 00:24:38
オナホはシリコンと称していても実際には塩ビ製がほとんど。
あのブヨブヨは可塑剤を大量に添加して実現してるので、
時間と共に可塑剤が抜ける事で縮んでしまうわけ。
新品のオナホってなんか油みたいなのが染み出してくるだろ?あれが可塑剤。

ちなみに可塑剤は数種類あるが、環境ホルモンである事が非常に多いw
380素数様 ◆.Cbdncl/Jc :2007/01/17(水) 00:27:04
素数様(゚听)イラネ
381132人目の素数さん:2007/01/17(水) 01:06:35
海外の入試問題やFランの入試問題や過去に実際に出された愚問ありますか
382132人目の素数さん:2007/01/17(水) 09:30:53
>>381
> Fラン

何ですか、それは?
383132人目の素数さん:2007/01/17(水) 11:46:19
>>382
Fast Runner
384132人目の素数さん:2007/01/17(水) 13:14:15
>>255 >>304
この答えってn次式なの?
(n次の係数は絶対0にならない?)
385132人目の素数さん:2007/01/18(木) 01:14:57
>>346

82番
(1) とりあえず計算した。 a= exp(iπ/180) とおくと、
 f(n) = Im{ Σ[k=1,n] (k^4)・a^k }
   = Im{ a[ (1+a)(1 +10a +a^2) - (n+1)^4・a^n + {3n(n+1)^3 -6(n+1)^2 -4(n+1) -1}a^(n+1) - {6(n^2)(n+1)^2 -12n(n+1) +11}a^(n+2) + {4(n+1)n^3 -6n^2 +4n -1}a^(n+3) - n^4・a^(n+4) ] / (1-a)^5 },

ハァハァ

Σ[k=1,n] a^k = a(1 - a^n) / (1-a),
Σ[k=1,n] k・a^k = a[1 -(n+1)a^n + n・a^(n+1) ] / (1-a)^2,
Σ[k=1,n] (k^2)・a^k = a[ 1+a - (n+1)^2・a^n + {2n(n+1) -1}a^(n+1) - n^2・a^(n+2) ] / (1-a)^3,
Σ[k=1,n] (k^3)・a^k = a[ 1+4a+a^2 - (n+1)^3・a^n + {3n(n+1)^2 -3(n+1) -1}a^(n+1) - {3(n^2)(n+1) -3n +1}a^(n+2) + n^3・a^(n+3) ] / (1-a)^4,
Σ[k=1,n] (k^4)・a^k = a[ (1+a)(1+10a+a^2) - (n+1)^4・a^n + {3n(n+1)^3 -6(n+1)^2 -4(n+1) -1}a^(n+1) - {6(n^2)(n+1)^2 -12n(n+1) +11}a^(n+2) + {4(n+1)n^3 -6n^2 +4n -1}a^(n+3) - n^4・a^(n+4) ] / (1-a)^5,
386385:2007/01/18(木) 01:19:15
>>346

蛇足だが、 |a| < 1 のとき
Σ[k=1,∞) a^k = a / (1-a),
Σ[k=1,∞) k・a^k = a / (1-a)^2,
Σ[k=1,∞) (k^2)・a^k = a(1+a) / (1-a)^3,
Σ[k=1,∞) (k^3)・a^k = a(1+4a+a^2) / (1-a)^4,
Σ[k=1,∞) (k^4)・a^k = a(1+a)(1+10a+a^2) / (1-a)^5,
387132人目の素数さん:2007/01/18(木) 04:34:00
>>347
83番

(1) logx×logy > log(1/2)×log(1/2) =(log2)^2
かしら…?
388132人目の素数さん:2007/01/18(木) 04:57:13
今、複素数 z = r(cosθ+i_sinθ) に対して z^n = r^n * {cos(nθ)+i_sin(nθ)}が成立することを
(@) n = 0.1.2… の時
(A) n = -1.-2.… の時
それぞれ証明しろ。
389132人目の素数さん:2007/01/18(木) 05:39:28
半径 r の円に内接する正n角形(n=3.4…)を考える。

2*3.14*r を Lo 、正n角形のn辺の和を Ln とした時 Lo < Ln となる最小の n を求めよ。
390132人目の素数さん:2007/01/18(木) 11:07:06
アルキメデスもびっくり。
391理一:2007/01/18(木) 11:10:55
このスレの問題ザーっと見たが、適当に作った問題ばっかとしか思えない。
〜の値を求めよ、〜の大小関係を求めよ、〜は存在するかとか作問者が問題解けてなくても出せるパターンばっか。
392132人目の素数さん:2007/01/18(木) 11:36:40
>>391
何をいまさら。
あなたがもっと東大入試っぽい問題出してよ。
393132人目の素数さん:2007/01/18(木) 12:16:24
g(n,x)=納k=0,n] an*x^k
g'(n,x)=g(n-1,x),g(n,0)=1
f(n,x)=e^x -g(n,x) ただしe=lim[h→0] (1+1/h)^h

g(n,x)=0はnが偶数のとき解を持たず、奇数のときただ一つの解を持つことを示せ。
lim[n→∞] f(n,x)=0を示せ。
394132人目の素数さん:2007/01/18(木) 12:38:40
前問のnは自然数です。書き忘れすみません。

nを自然数とする。
f(n,x)=x^nの係数が1のxのn次式
M(n)= -1≦x≦1での|f(x)|の最大値
M(n+1)=M(n)/2を示せ。(n=1,2,…)
395132人目の素数さん:2007/01/18(木) 13:07:35
四面体OABCがあり、その重心を通る平面が辺OA,OB,OCと交わっている。
その交点をそれぞれP,Q,R、四面体OABC、OPQRの体積をV,vとするとき、
その二つの体積比v/Vのとりうる値の範囲を求めよ。
396132人目の素数さん:2007/01/18(木) 13:13:01
>>393>>394は京大みたいな問題なので却下。ゴミ箱行き。
397132人目の素数さん:2007/01/18(木) 13:20:51
>>393->>395
素数様の問題に比べたらくそだな。もう来るな。
398132人目の素数さん:2007/01/18(木) 13:23:30
そうですか。とっさに思いついた問題ではやはりだめでしたか。
失礼しました。
399132人目の素数さん:2007/01/18(木) 13:28:14
何この流れ
400132人目の素数さん:2007/01/18(木) 14:55:13
俺は彼の問題見たときピンとキタねw彼は○数様だってwww
401132人目の素数さん:2007/01/18(木) 16:05:21
おい、みんな。素数様にお願いするんだ
彼(もしかして彼女?)に良心があるなら、本当のことを教えてくれるかもしれない。
素数様、本当のことを教えてください。
あなたがここに書いている問題は本当に解ける問題なのですか。
それとも>>391が言ってるみたいに適当に作って放置しているだけですか。
お願いです。教えてください。


402132人目の素数さん:2007/01/18(木) 16:41:36
実は、適当に書いているだけです。
解けるかどうかは分かりません。
403132人目の素数さん:2007/01/18(木) 21:34:13
>347

83番
(1) 0 < log(x)・log(y) ≦ ∞, 
 (x,1-x) → (0,1), (1,0) のとき →0.
 (x,x) → (0,0) のとき → ∞.
(2) 1 < x^y + y^x < 2,
 (x,1-x) → (0,1), (1,0) のとき →1, (x,x)→(0,0) のとき →2.
(3) 0 < (x^y)(y^x) < 1,
 (x,1-x) → (0,1), (1,0) のとき →0, (x,x)→(0,0) のとき →1.

>374

86番
 Ver. 27275407070315(150桁略)540741863533281

 ……って、「くだらんスレ」ぢゃなかった......ort.
404132人目の素数さん:2007/01/18(木) 23:19:23

ざわざわ…
405387:2007/01/19(金) 14:11:14
>>403 なんで?
406132人目の素数さん:2007/01/19(金) 17:18:57
平面上に、長さLの線分ABがある。線分上(端を含む)から点P,Q,Rを
それぞれでたらめに選ぶ。

このときAP+AQ+AR≦2L
となる確率を求めなさい。
407132人目の素数さん:2007/01/19(金) 17:20:52
すいません、質問なんですが数学の^ってどういう意味ですか?
408132人目の素数さん:2007/01/19(金) 17:23:02
(^o^)<ばーか
409132人目の素数さん:2007/01/19(金) 17:24:31
笑った顔の半分だよ 2つ書いたら笑顔みたいになるでしょ
^^
ほらね
410132人目の素数さん:2007/01/19(金) 17:25:13
^^ワラタオ
411132人目の素数さん:2007/01/19(金) 18:05:24
168 名前:名無しさん@4周年[] 投稿日:04/01/17 10:43 ID:X7vkUwRR
いよいよ明日がセンター試験本番ですよ!
むっちゃドキドキしてきた…。
受験生の皆さん、今日くらいは勉強は休んで明日に備えますよね?


169 名前:名無しさん@4周年 投稿日:04/01/17 10:57 ID:zUQVw2G7

>>168

  . .... ..: : :: :: ::: :::::: :::::::::::: : :::::::::::::::::::::::::::::::::::::::::::::
       Λ_Λ . . . .: : : ::: : :: ::::::::: :::::::::::::::::::::::::::::
      /:彡ミ゛ヽ;)ー、 . . .: : : :::::: :::::::::::::::::::::::::::::::::
     / :::/:: ヽ、ヽ、 ::i . .:: :.: ::: . :::::::::::::::::::::::::::::::::::::::
     / :::/;;:   ヽ ヽ ::l . :. :. .:: : :: :: :::::::: : ::::::::::::::::::
 ̄ ̄(_,ノ  ̄ ̄ ̄ヽ、_ノ ̄
今日と明日だよ
来年こそはがんばってよ
シーズン開幕からこんなことになるなんて


173 名前:168 投稿日:04/01/17 11:10 ID:X7vkUwRR
受験要綱を見た。
どうやら今日と明日、両方とも試験があるらしい…。
親に話したら泣かれた。怒られた。殴られた。
学校の先生に電話したら怒鳴られた。今すぐに学校に来いって言われた。
今から学校に行ってきます……もうだめぽですか
412132人目の素数さん:2007/01/19(金) 18:42:18
殺人事件を笑って報道するアナってどう思う?
フジ新人女子アナがやらかしたらしいけど
謝罪も無く、お咎めも無しらしい。

詳しくは
http://ex13.2ch.net/test/read.cgi/tv/1131977130/l50
413403:2007/01/20(土) 00:43:02
>347,>405

83番
(1) は略
(2)-右 と (3) は 0<x^y<1, 0<y^x<1 から。

問題は (2)-左 だが、
 (1-x)(1-y)>0 のとき x^y + y^x > 1+xy.
(略証) b>0 としておく。
 f(x) = (1/b)^x は下に凸だから、(1,1/b) - (x,(1/b)^x) の平均変化率は単調増加。
 {(1/b)-(1/b)^x}/(1-x) > (1/b)-1.
 x<1 のとき、 b^x > b/{1-(1-x)(1-b)} = b/(x+b-xb),  (ベルヌーイの不等式)
 ∴ y^x > y/(x+y-xy),
 ∴ x^y + y^x > (x+y)/(x+y-xy) = 1 + xy/(x+y-xy) > 1+xy. (終)
414132人目の素数さん:2007/01/20(土) 05:45:38
誰か150分の六問構成の試験作ってくれないか(出来れば完全に東大型か京大型のどちらか
415132人目の素数さん:2007/01/20(土) 06:11:22
>>414
ここに近いものはあるが・・・。
http://homepage2.nifty.com/enjoying_math/index.htm
416132人目の素数さん:2007/01/20(土) 10:17:48
灯台は蹴落とす試験だから、クイズタイムショックかパネルクイズ25
形式でネットで試験すればいじゃないか。

ち、ち、ち、ち、7から数えて25番目の素数は・・・・
417132人目の素数さん:2007/01/20(土) 10:19:07
ち、ち、ち、ち、いま何問目?
418132人目の素数さん:2007/01/20(土) 10:54:18
灯台より設備のよさやノーベル賞級の教授が多い欧米の大学のほうが格段にいい。
419132人目の素数さん:2007/01/20(土) 11:00:05
青木さやかも絶賛!!アンダーグラウンド
ttp://jbbs.livedoor.jp/bbs/i.cgi/news/2092/
420132人目の素数さん:2007/01/20(土) 15:04:19
>416
 π(7) = π(10) = 4,
 π(100) = 25.
 ∴ 100より大きい (4-1)人目の素数さん >>107
421132人目の素数さん:2007/01/20(土) 15:31:40
>406
 AP /L =x, AQ /L =y, AR /L =z とおき、3次元空間で考える。
点(x,y,z) は一辺1の立方体 {(x,y,z)| 0≦x,y,z≦1} に含まれる。
そのうち x+y+z ≦ 2 の領域が占める割合は 1 -(1/6) = 5/6.
422132人目の素数さん:2007/01/20(土) 22:52:38
>>421
正解です。
423132人目の素数さん:2007/01/21(日) 00:29:42
>>290

75番 (1)
 n(n+1)(n+2)/6 = 納0≦i,j,k, i+j+k<n] 1,
n(n+1)…(n+m-1)/m! = 納0≦x_1,…,x_m, x_1+x_2+…+x_m<n] 1.
424132人目の素数さん:2007/01/21(日) 00:42:18
97 :132人目の素数さん :2007/01/17(水) 05:38:55
>>33-47
変な問題が多いな。
2. (1)p=3で成立 (2)(3)p=6で成立 (4)p=2が反例
4. 重心をとるだけ。(2/3, 0, 0)
5. とりあえず任意の実数a,b,cで成立するけど、複素数まで考えろと?
6. α=β=γ=δ=2の場合だけ考えれば終了。(1)(2)非有界 (3)不定
9. (2)条件2より自明
11. f(x)+g(x)+h(x)=6 (定数関数) となるf,g,hが存在するから問題が間違い
12. ネタ。答えが無い
14. n>m
19. 線分で平面を切断するって?
20. (1)√14 (2)C=A,D=Bとして4√14
23. (n!)C(r!)≧n!≧nCr
24. (2)原点のまわりでf(x)≧g(x)なのでa=0
26. (2)任意のnに対し0点
29. A=B=Xなら何でも
39. logの定義
53. F=a^4bc+b^4ca+c^4ab-a^3b^2c-b^3c^2a-c^3a^2b-a^3bc^2-b^3ca^2-c^3ab^2じゃなく?
55. (2)明らかに周期2πが存在
60. 3次、4次の条件は冗長。(γ,δ)=(α,β),(β,α)
61. (1)任意のpが条件を満たす (2)q=1
73. (2)決まらない。例えばB=O, C=kAのときA^n+B^n+C^n=(1+k^n)A^n
74. 行列の次数は?
425132人目の素数さん:2007/01/21(日) 04:01:02
素数様は、因数分解が得意な中学生に例えるなら、「そんなもんPCで計算させろよw」と突っ込みを
入れたくなるような、滅茶苦茶な因数分解をさせる問題を出す中学生である。「どうだ難しいだろう」
「これが解けない奴は低脳w」という、間違った難易度感・間違った学力感を持った中学生である。
426132人目の素数さん:2007/01/21(日) 04:11:31
お願いだから正しい難易度感・正しい学力感を提示してくれ。
427132人目の素数さん:2007/01/21(日) 04:28:04
素数様 玉、国、本、風 力が同じなのは?という無茶ななぞなぞをふっかける
428132人目の素数さん:2007/01/21(日) 14:14:04
>399

流 政之(ながれ まさゆき)氏は、世界的に活躍する彫刻家。
 "Samurai Artist"の異名をもつ。
 1923(大正12)年長崎県に生まれ。
 1942年立命館大学法文学部へ進学、その後中退。
 海軍予備学生出身の零戦搭乗員として終戦を迎える。
429132人目の素数さん:2007/01/21(日) 14:34:17
>>428
そんなことより、日本アカデミー賞の最優秀賞受賞者に送られるブロンズ像のデザイン者であり、
さらに、「立命館大学」を命名した中川小十郎を父に持つ。
あたりを押さえておいた方が雑学としてはおいしいぞ。
430132人目の素数さん:2007/01/21(日) 16:01:08
>>426
体で感じろ。
431132人目の素数さん:2007/01/22(月) 06:30:05
妹に「牛タンってウマイよな」って言ったら、
「うん、人間の舌も牛タンだったらいいのにね」って言われた。

「そしたら何も食べてなくても、常に牛タンの味がしておいしいのに」だって。
たしかに、人間の舌っていつも口の中にあるのに味がしないなー。

妹と話し合った結果、それはもしかすると
ずっと同じ味の舌が口の中に入ってるから味覚が麻痺してるんじゃないか?
ということになって、お互いの舌を舐め合って確かめてみることにした。

そしたらすごい!妹の舌おいしい!!まろやか!
お互いに相手の舌を舐めながら「おいしいよー」「おいしいねー」
「デリシャスだよー」「デリシャスだねー」ってやってたら、
だんだんお互いの口の中に唾が溜まってきて
自分の唾はおいしくないけど相手の唾はコクがあっておいしいことに気付いたんだ。

それからは二人とも夢中で唾液を交換してぐちゅぐちゅ混ぜ合わせて共有して
お互いの味をしっかり確かめて味わって、最後に溜まった唾を仲良く半分こして飲んじゃった。
「お兄ちゃんの舌と唾、すっごくおいしくてびっくりしちゃった。」て興奮気味の妹。
二人ともこの味をすっかり気に入っちゃってそれから毎日ぺろぺろちゅぱちゅぱしてます。
432132人目の素数さん:2007/01/22(月) 20:32:52
(1)無限に広い碁盤がある。4目並べでは先手必勝であることを証明せよ。
(2)碁盤の広さが5X5マス(石は交点に打つとする)ではどうか。
433132人目の素数さん:2007/01/22(月) 20:36:30
>>352
一週間
434132人目の素数さん:2007/01/22(月) 20:46:21
1目=先手必勝
n目=先手必勝ー>n+1目は先手必勝
435132人目の素数さん:2007/01/22(月) 20:54:45
>>434
理由の厳密な証明をお願いします。あと(2)の答えもお願いします。
436132人目の素数さん:2007/01/22(月) 21:25:16
リナックスのおまけのlispでヒューレリステイックで調べるといい。
この手の問題はたいていそれで片が付く。
437132人目の素数さん:2007/01/23(火) 00:46:16
3つの連続したピタゴラス数が無限に存在する事を示せ。
438132人目の素数さん:2007/01/23(火) 00:51:20
5×5マスで交点に打つってことは線は6本ってこと?
439132人目の素数さん:2007/01/23(火) 01:57:22
>>437
n^2+(n+1)^2=(n+2)^2
2n^2+2n+1=n^2+4n+4
n2^-2n=3=0
n=3,-1のみ
氏ね
440132人目の素数さん:2007/01/23(火) 02:13:24
>>437
バカですか?
441132人目の素数さん:2007/01/23(火) 02:36:12
素数様よりひどいな…

そんな問題を嬉々悠々解いてるのもまぁ、アレだが
442132人目の素数さん:2007/01/23(火) 04:51:00
>>438 そのとうり。
4X4マスの場合も考えると面白いが、簡単なので5X5にした。
厳密な証明は結構やっかい。
443132人目の素数さん:2007/01/23(火) 05:49:33
>>442
> そのとうり。

余計なお世話だろうが、「そのとおり」 だ!
日本語がなっちゃいねぇ!
444132人目の素数さん:2007/01/23(火) 05:54:39
ぇ なんて使う奴に言われたくないって108歳のばあちゃんが言ってた。
445132人目の素数さん:2007/01/23(火) 06:07:56
>>442は児玉清さんじゃない?
446132人目の素数さん:2007/01/23(火) 10:33:05
そのとうり
447132人目の素数さん:2007/01/23(火) 11:33:40
園と瓜
448132人目の素数さん:2007/01/23(火) 11:56:36
児玉清の物真似の上手い芸能人がいたよな。
449437:2007/01/23(火) 20:24:20
>>439
ごめん。シンプルにしようとする余り問題文がイミフになってた。
これなら大丈夫だろう。

n,n+1,n+2がそれぞれピタゴラス数になるようなnが無限に存在する事を示せ。
450132人目の素数さん:2007/01/23(火) 21:04:38
(3,4,5) (-1,0,1)以外に一例挙げて
451437:2007/01/23(火) 21:33:04
>>450
3、-1はピタゴラス数ではないだろ。

80=8^2+4^2
81=9^2+0^2
82=9^2+1^2
ってな感じの数が無限にある事を証明して欲しいんだ。81=9^2+0^2みたいな書き方はアリとする。
多分「ピタゴラス数」って書き方がまずかったか・・・。
452132人目の素数さん:2007/01/23(火) 21:34:07
>>451
ピタゴラス数の定義ヨロ
453132人目の素数さん:2007/01/23(火) 22:28:26
>>451
はやく定義書けよw

つうか、このスレ、素数様以外は、カスしかいねーのな。
454132人目の素数さん:2007/01/23(火) 22:32:19
素数様でない⇒カス
素数様はカス
よってみんなカス
455132人目の素数さん:2007/01/23(火) 23:02:41
>>454
こちらにおわそう方をどなたと心得る、 畏れ多くも、糞問作りの神様、素数様にあらせられるぞ!!

456132人目の素数さん:2007/01/23(火) 23:10:08
センターの結果に悩んでいてもしょうがない。
はやく出てきてくれ素数様。
457437:2007/01/23(火) 23:38:07
a^2+b^2=c^2を満たす数をピタゴラス数というんだろ。皮肉のつもりか?
もう最後。

n,n+1,n+2それぞれ2つの平方数の和として書き表せるような自然数nが無限に存在する事を示せ。
458132人目の素数さん:2007/01/23(火) 23:42:27
>>457
3^2+4^2=5^2はピタゴラス数だろ。皮肉のつもりか?
459132人目の素数さん:2007/01/23(火) 23:47:28
>>457
カスはすっこんでろw
460132人目の素数さん:2007/01/23(火) 23:48:55

α+β+γ=πを満たす点(sinα,sinβ,sinγ)全体で囲まれる立体の体積を求めよ
461訂正:2007/01/23(火) 23:50:03

α+β+γ=πを満たす点(sinα,sinβ,sinγ)全体がなす曲面で囲まれる立体の体積を求めよ
462132人目の素数さん:2007/01/23(火) 23:52:33
無理。囲まれないから。
463132人目の素数さん:2007/01/24(水) 00:02:52
明らかに有界だがほんとに囲まれないか?
形が想像できないが
464132人目の素数さん:2007/01/24(水) 00:35:24
>>457
a^2+b^2=c^2を満たす 3 つ の 自 然 数 の 組 (a,b,c) を
ピタゴラス数と呼ぶ。c単体でピタゴラス数と呼ぶことは無い。クズは消えろ。
465132人目の素数さん:2007/01/24(水) 00:37:52
遠回しに馬鹿をからかうのが面白いのに。すぐムキになる人って嫌だね。
466KingOfUniverse ◆667la1PjK2 :2007/01/24(水) 10:55:37
要するに、人の脳を読む能力を悪用する奴を潰せばいいのか?
467132人目の素数さん:2007/01/24(水) 11:17:08
遠回しに馬鹿をからかうのを面白く感じるような根暗って嫌だね。
468132人目の素数さん:2007/01/24(水) 13:05:03
ピ タ ゴ ラ ス 数
469132人目の素数さん:2007/01/24(水) 13:11:06
d(m^2−n^2)^2+d(2mn)^2=d(m^2+n^2)^2    d、m,nは自然数

470437:2007/01/24(水) 15:15:16
ちょw 書き直したのに未だにピタゴラス数〜〜とか言ってるのかよ。
限りなく粘着だな。まぁ俺もだが。
ピタゴラス数〜〜は忘れて、これを解いてみてくれ。(コピペ)

n,n+1,n+2それぞれ2つの平方数の和として書き表せるような自然数nが無限に存在する事を示せ。
471132人目の素数さん:2007/01/24(水) 15:24:03
今年の東大入試にここの問題が!
472132人目の素数さん:2007/01/24(水) 16:24:41
>>470
非負整数nに対して、17^(2^n)−1,17^(2^n),17^(2^n)+1という連続する3つの
整数は、それぞれ2つの平方数の和で表せる。これを数学的帰納法で示す。
(n=0のとき)3つの整数は16,17,18である。
16=4^2+0^2
17=1^2+4^2
18=3^2+3^2
であるから、確かに成り立つ。
(n=k≧0のとき成り立つとすると、n=k+1のとき)
3つの整数は17^{2^(k+1)}−1,17^{2^(k+1)},17^{2^(k+1)}+1である。
17^{2^(k+1)}=0^2+{17^(2^k)}^2
17^{2^(k+1)}+1={17^(2^k)}^2+1^2
であるから、あとは17^{2^(k+1)}-1が2つの平方数の和になることが示せればよい。
仮定から、ある非負整数a,b,x,yが存在して
17^(2^k)−1=x^2+y^2
17^(2^k)+1=a^2+b^2
と表せるので、
17^{2^(k+1)}−1={17^(2^k)+1}{17^(2^k)−1}=(a^2+b^2)(x^2+y^2)
=(ax+by)^2+(ay−bx)^2
となり、確かに成り立つ。
473472:2007/01/24(水) 16:38:11
結論が抜けていたな。

(>>472の続き)
従って、n,n+1,n+2それぞれ2つの平方数の和として書き表せるような自然数nは確かに
無限に存在し、そのような自然数nは具体的にはn=17^(2^m)−1 (m=0,1,2,…)である。
474472:2007/01/24(水) 17:07:35
>>470
これを解いてみてくれ。

kは自然数とし、A={kn+1|nは非負整数}∪{0}とおく。さらに、
自然数mに対してA[m]={x1+x2+…+xm|x1,x2,…,xm∈A}とおく。
(1)A[2]に含まれない自然数が無限に存在することを示せ。
(2)「どんな自然数もA[m]に含まれる」が成り立つ自然数mのうち、最小のものを求めよ。

次に、B={(2^a)(3^b)(5^c)|a,b,cは非負整数}∪{0}とおく。さらに、
自然数mに対してB[m]={x1+x2+…+xm|x1,x2,…,xm∈B}とおく。
(3)B[2]に含まれない自然数が無限に存在することを示せ。
(4)「どんな自然数もB[m]に含まれる」が成り立つ自然数mは存在しないことを示せ。
475472:2007/01/24(水) 17:14:46
あかん。またミスってもうた。連投スマソ。

誤:kは自然数とし、A={kn+1|nは非負整数}∪{0}とおく。
正:kは3以上の自然数とし、A={kn+1|nは非負整数}∪{0}とおく。
476132人目の素数さん:2007/01/24(水) 19:58:57
>470
釣れたよーって言って溺れてるAAあったよね
477132人目の素数さん:2007/01/24(水) 23:15:08
半径1の円に内接する正五角形のある1つの頂点から他の4つの頂点までの距離の積を求めよ。
478132人目の素数さん:2007/01/24(水) 23:22:50
ちょ、え、中学レベル。
479132人目の素数さん:2007/01/24(水) 23:52:14
>477
 半径1の円に内接する正n角形の辺・対角線の長さは 2sin(kπ/n),  k=1,2,…,n-1
 (辺: k=1, 対角線: k=2,…,n-1)

Π[k=1,n-1] {2sin(x + kπ/n)} = sin(nx)/sin(x) = U_n(cos(x)),   (x≠mπ)
より x→0 として、
 Π[k=1,n-1] {2sin(kπ/n)} = U_n(1) = n.
ここに U_n(x) は第2種チェビシェフ多項式。
480素数様:2007/01/25(木) 01:33:41
最近書き込めなくてすまん。
実は事故にあって腕を骨折した。
そのせいでセンターは受けられなかった。
今年の浪人も決定した。
だからもう俺大学に行かないことにした。
大学が俺を拒んでるとしか思えない。
ここで就職探しながらずっと問題を提供して君達の力になろうと思う。

87番
数列{a_n}は等差数列は、初項a、公差dの等差数列で、S_n=a_1+a_2+・・・・+a_nとすると、任意の
自然数m,nに対してS_(m+n)=s_(m)+S_(n)が成り立つという。このような数列{a_n}をすべて求めよ。
481素数様:2007/01/25(木) 01:36:36
87番訂正
s_(m)になってるけどS_m_の間違い

88番
任意の自然数nに対して、
 an^2+(2a+3)n+4
―――――――――
a^2n^3-(a+2)n^2+2n+3
が整数とならないような整数aの値を決定せよ。
482素数様:2007/01/25(木) 01:40:32
89番

自然数aに対して、T(a)で各桁の自然数の和を表す。たとえばT(243)=2+4+3=9である。
(1)T(x)=10となる自然数xのうち、0が2個以上連続して並ばないものは何通りあるか。
(2)T(y)=nとなる自然数yのうち、0が2個以上連続して並ばないものは何通りあるか。
(3)T(x)+T(y)=10となる自然数x,yのうち、ともに0が2個以上連続して並ばないものは何通りあるか。
483132人目の素数さん:2007/01/25(木) 03:23:12
>ここで就職探しながらずっと問題を提供して君達の力になろうと思う

おまえもうここにこなくていいよ。
マジいらない。
484132人目の素数さん:2007/01/25(木) 03:44:17
ニートは帰れよ
数学科出てからまた来い
485132人目の素数さん:2007/01/25(木) 08:53:14
また素数様叩きですね。ホント、いい加減にしてください><
486132人目の素数さん:2007/01/25(木) 14:14:31
以下の条件(1),(2)をともに満たすような自然数の組(x,y)は存在するか。
存在するならば例を挙げ、存在しないならばその事を証明せよ。
 (1)x,y≧2
 (2)x^yの下4桁が2007
487132人目の素数さん:2007/01/25(木) 16:50:54
>>486
存在する。x^y≡2007 (mod 10^4)を満たすx,y≧2が存在することを示す。gcd(2007,10^4)=1だから、
オイラーの定理より2007^φ(10^4)≡1 (mod 10^4)となる。φ(10^4)=4000なので、2007^4000≡1 (mod 10^4)
となる。そこで、x=2007,y=4001とおけば、x^y≡(2007^4000)*(2007^1)≡2007 (mod 10^4)となるので、
これが求めるx,yの一例である。
488132人目の素数さん:2007/01/26(金) 15:19:15
3つの連続したピタゴラス数が無限に存在する事を示せ。
489132人目の素数さん:2007/01/26(金) 15:34:03
3,4,5
490132人目の素数さん:2007/01/26(金) 18:17:24
>>488
意味不明
491437 人目の素数さん:2007/01/26(金) 20:07:48
>>472
乙。
問題だが、x1,x2・・・はAから任意に選んだのか?

>>488
wwwwww
492132人目の素数さん:2007/01/26(金) 21:00:54
>問題だが、x1,x2・・・はAから任意に選んだのか?
>問題だが、x1,x2・・・はAから任意に選んだのか?
>問題だが、x1,x2・・・はAから任意に選んだのか?
>問題だが、x1,x2・・・はAから任意に選んだのか?

集合の表記もロクに知らないときたかwwwwwwwwwwwwwwww
493132人目の素数さん:2007/01/26(金) 21:03:51
>486 例
 (x,y) = (7,61), (23,35).

(略証)
7^2 = 50-1,
7^4 = (50-1)^2 = 2400+1,
7^20 = (2400+1)^5 = 79792266297612001 ≡ 12001 (mod 10^5),
7^60 ≡ (12000+1)^3 ≡ 36001 (mod 10^5),
7^61 ≡ 52007 (mod 10^5).
494132人目の素数さん:2007/01/26(金) 21:22:48
>480

87番
題意より a_k = a + d(k-1),
 S_n = Σ[k=1,n] a_k = Σ[k=1,n] {a+(k-1)d} = (a - d/2)n + (d/2)n^2,
題意より 
 0 = S_(m+n) - S_m - S_n = (d/2)(m+n)^2 - (d/2)m^2 - (d/2)n^2 = dmn,
これが任意の自然数m,nに対して成り立つから、d=0, a_k=a.


〔問題486’〕
 半径1の円に内接する正n角形のある1つの頂点から他の(n-1)個の頂点までの距離の和を求めよ。
495486:2007/01/26(金) 23:22:58
>>487>>493
正解!オイラー使っていいのか知らないけどw
俺は>>493同様に7^61の下4桁が2007である事を示しました
496132人目の素数さん:2007/02/03(土) 12:49:48
定期試験入試で過疎
497132人目の素数さん:2007/02/03(土) 13:23:09
正方形の頂点を反時計回りにABCDとおく。この頂点上を動く点X,Yがあり、コインを投げて表が出ればXを、裏が出ればYを反時計回りに隣の頂点に移動させる。
X,Yは最初Aにあるとする。n回目の動作の後に同じ頂点にある確率をp(n)とおく。
(1)p(2)を求めよ
(2)p(4)を求めよ
(3)p(n)を求めよ
498132人目の素数さん:2007/02/03(土) 13:57:29
>>497は模試の問題。
答えるべからず。
499132人目の素数さん:2007/02/03(土) 16:25:04
てか,その程度の問題ぐらい自力で解け
500132人目の素数さん:2007/02/03(土) 18:10:36
一辺が1の正方形の内部または周上に存在しうる正五角形の面積の最大値を求めよ。
501132人目の素数さん:2007/02/03(土) 22:48:18
>494

 2sin(x) = {1/sin(π/2n)} {cos(x - π/2n) - cos(x + π/2n)}    (← 積和公式)
 L = Σ[k=1,n-1] 2sin(kπ/n)
  = {1/sin(π/2n)} Σ[k=1,n-1] {cos[(k-1/2)π/n] - cos[(k+1/2)π/n]} 
  = {1/sin(π/2n)} {cos(π/2n) - cos((n-1/2)π/n)}
  = 2/tan(π/2n).

>500

最大の正5角形の外接円の半径Rは
 R = 1/{cos(3π/20) + cos(π/20)} = 1/{2cos(π/10)cos(π/20)} = 0.5322844…
 (正5角形の1頂点から対辺への垂線が正方形の対角線上に来るように置く)
 sin(2π/5) = √{(5+√5)/2} = 0.9510565… より
 S = (5/2)sin(2π/5)・R^2 = 2.377641… R^2 = 0.67364926…
502501:2007/02/03(土) 22:55:03
>501 の訂正
 sin(2π/5) = cos(π/10) = √{(5+√5)/8} = 0.9510565…
503132人目の素数さん:2007/02/04(日) 19:50:33
>>501
「正5角形の1頂点から対辺への垂線が正方形の対角線上に来るように置」
けば良い理由が核だと思うが。
504132人目の素数さん:2007/02/04(日) 20:20:17

Problem 256.
 sin(1゚)sin(2゚)……sin(89゚) = (3/2^89)√10.
を示せ。

http://www.math.ust.hk/excalibur/v11_n4.pdf

hint >>479, n=180.
505132人目の素数さん:2007/02/04(日) 23:29:31
>>500
5α/4(α+1) (ただしα=cos18°)

発想を転換し、五角形を単位円に内接するものと固定し、
これを回転させるときx座標のとりうる値の幅とy軸のとりうる値の幅のうち大きな方をminimizeすれば良い。
すると、結局、0≦θ≦π/5での
min{cosθ,sin(θ+(2π/5))}
の最小点を調べれば良い。
これはすぐにθ=π/20だと分かる。
これは、>>503のことの証明になっている。
506132人目の素数さん:2007/02/04(日) 23:31:35
間違えた

すると、結局、0≦θ≦π/5での
max{cosθ,sin(θ+(2π/5))}
の最小点を調べれば良い。
507132人目の素数さん:2007/02/04(日) 23:56:02
3つの連続したピタゴラス数が無限に存在する事を示せ。
508132人目の素数さん:2007/02/07(水) 00:31:13
入試も近いので本番形式の問題でも。
1. xy平面上の曲線y=x^2上に点P,Qが、Pのx座標がの値とQのx座標の値の差が4であるように動く。
このとき、線分PQが通過する範囲を図示せよ。
2. α,β,γはどれも実数ではない複素数の定数である。このとき
α+β+γ、α^2+β^2+γ^2、α^3+β^3+γ^3の少なくとも一つは実数でないことを示せ。
3. xyz空間で原点を中心とする半径1の球の内部で
|x^2+2y^2-1|≦(1-x^2)cos(πx/2)を満たす部分の体積を求めよ。
509132人目の素数さん:2007/02/07(水) 00:35:47
4. 一辺の長さが1の正三角形ABCの辺AB上に点Pを、辺BC上に点Qを、辺CA上に点Rと点Sを、四角形PQRSが長方形となるようにとる。
このとき直線ABを軸として長方形PQRSを回転してできる立体の体積の最大値を求めよ。
5. aを正の実数の定数とし、f(x)=x^3-axとする。xy平面上でy=f(x)で表される曲線をCとする。
原点を中心とし曲線Cと接するような円が二つ存在し、その二つの円の面積の差が4πであるようなaの値を求めよ。
6. 数列{a[n]}はa[1]=ー1、a[n+1]=√(2+a[n]) で定まる。
(1)lim[n→∞] a[n]を求めよ。
(2)(1)で求めた値をαとしkを正の定数とする。このとき lim[n→∞] (αーa[n])*(k^n) が0以外の値に収束するkの値とその時の収束値を求めよ。
510132人目の素数さん:2007/02/07(水) 00:41:43
素数様来てくれ!!
511132人目の素数さん:2007/02/07(水) 01:27:59
>>509
30分で1,2,5解けた
4と6(1)は半分くらい出来た感じ
あとはわからん
512132人目の素数さん:2007/02/07(水) 02:36:51
>508
1.
 P(a-2,(a-2)^2), Q(a+2,(a+2)^2) とおく。
 PQの中点は (a, a^2 +4), PQの傾きは 2a.
 PQ: y= 2a(x-a) + a^2 +4 ≧ -(a+2-x)(x-a+2) + 2a(x-a) + a^2 +4 = x^2 (∵ 下に凸)
 PQ: y= 2a(x-a) + a^2 +4 ≦ (x-a)^2 +2a(x-a) +a^2 +4 = x^2 +4,
∴ 求める領域は x^2 ≦ y ≦ x^2 +4. (2つの放物線の間)

2. 背理法による。
S1 = α+β+γ、S2 = α^2+β^2+γ^2、S3 = α^3+β^3+γ^3 がすべて実数だったと仮定する。
然らば、基本対称式
 α+β+γ= S1, αβ+βγ+γα = (S1^2 -S2)/2, αβγ = (S1^3 -3・S1・S2 + 2・S3)/6,
も実数となる。
 f(x) = (x-α)(x-β)(x-γ) = x^3 - S1・x^2 + {(S1^2 -S2)/2}x - {(S1^3 -3・S1・S2 +2・S3)/6}
は実3次函数だから f(-∞) <0, f(∞) >0.
中間値の定理により、少なくとも一つ実根をもつ。


>509
6.
 a[n] = 2cos(θ[n]), 0≦θ≦π とおくと、題意より
 θ[n+1] = θ[n] /2 = …… = θ[1]*(1/2)^n, θ[1] = 2π/3.
 (1) Lim[n→∞) θ[n] = 0, Lim[n→∞) a[n] =2 =α.
 (2) α- a[n] = 2(1-cosθ[n]) = {2(1-cosθ[n])/θ[n]^2}・θ[n]^2
     = {2(1-cosθ[n])/θ[n]^2}・(θ[1]^2)*(1/4)^(n-1),
    ∴ k=4,  Lim[n→∞) (α- a[n])*(4^n) = 4θ[1]^2.
513132人目の素数さん:2007/02/07(水) 04:02:01
>>509
計算ミスがあるかもしれない。
6.(1)
n≧2のときa[n]=2cos{x/2^(n-2)} (x=π/3)となる…*ことを数学的帰納法で示す。
n=2のとき…明らかに成り立つ。
n=k≧2のとき成り立つとすると、n=k+1のときa[n]=a[k+1]=√(2+a[k])
=√(2+2cos{x/2^(k-2)})=2√{(1+cos{x/2^(k-2)})/2}=2√{(cos{x/2^(k-1)})^2}
=2|cos{x/2^(k-1)}|=2cos{x/2^(k-1)} となり、成り立つ。
以上より、確かに*は成り立つ。よって、lim[n→∞]a[n]=2となる。

6.(2)β=lim[n→∞](k^n)*(α−a[n])とおく。α=2であるから、n≧2のときα−a[n]
=2−2cos{x/2^(n-2)}=4{(1−cos{x/2^(n-2)})/2}=4(sin{x/2^(n-1)})^2となる。
よって、(k^n)*(α−a[n])=4(k^n)(sin{x/2^(n-1)})^2となる。簡単のため
c[n]=x/2^(n-1)とおくと、4(k^n)(sin{x/2^(n-1)})^2=4(k^n)(sinc[n])^2
=4{(k^n)/c[n]^2}{(sinc[n])/c[n]}^2 となる。c[n]→0であるから、
{(sinc[n])/c[n]}^2→1となる。これと4(k^n)/c[n]^2=4(k^n){4^(n−1)}/x^2
=(4k)^n/x^2であることより、β=lim[n→∞]{(4k)^n/x^2}{(sinc[n])/c[n]}^2となり、よって
|4k|<1のとき…β=0
|4k|>1のとき…β=∞あるいは不定
4k=−1のとき…β=不定
4k=1のとき…β=1/x^2=9/π^2
となる。以上より、k=1/4,lim[n→∞] (αーa[n])*(k^n)=9/π^2となる。
514513:2007/02/07(水) 04:08:11
さっそく間違えたお( ^ω^)

6.(2)(途中から)
簡単のためc[n]=x/2^(n-1)とおくと、4(k^n)(sin{x/2^(n-1)})^2=4(k^n)(sinc[n])^2
=4{(k^n)c[n]^2}{(sinc[n])/c[n]}^2 となる。c[n]→0であるから、
{(sinc[n])/c[n]}^2→1となる。これと4(k^n)c[n]^2=4(k^n){x^2/4^(n−1)}
=(16x^2)(k/4)^nであることより、
β=lim[n→∞]{(16x^2)(k/4)^n}{(sinc[n])/c[n]}^2となり、よって
|k/4|<1のとき…β=0
|k/4|>1のとき…β=∞あるいは不定
k/4=−1のとき…β=不定
k/4=1のとき…β=16x^2=(16π^2)/9
となる。以上より、k=4,lim[n→∞] (αーa[n])*(k^n)=(16π^2)/9となる。
515132人目の素数さん:2007/02/08(木) 11:31:32
6.(1)は
Aをある有限値として
lim[n→∞]a[n]=Aとおくと
lim[n→∞]a[n+1]=Aであるから
与式より A=√(2+A)>0
両辺二乗 A^2=2+A
∴A=2 (A=-1<0より不適)
∴lim[n→∞]a[n]=2

では駄目?
516132人目の素数さん:2007/02/08(木) 16:51:44
>>515
物理や工学の人はよく使ってるね
数学的にはダメ
収束することの証明が必要
517132人目の素数さん:2007/02/08(木) 17:08:16
>>515 値の目星を付けるのにその方法は使えるけど、厳密性にかける。2に収束しそうだなと予想したら
2-a[n+1]=2-√(2+a[n])
=(2-a[n])/{2+√(2+a[n])}
<(2-a[n])/2
とか大雑把に大小関係を評価して2-a[n]が0に収束することを示せばOKかと。
518132人目の素数さん:2007/02/08(木) 18:50:07
>>516-517
どうもです
519132人目の素数さん:2007/02/12(月) 15:06:57
〔問.479〕
 x,y,z は自然数で、 1/x + 1/y = 1/z, xとzは互いに素とする。
 このとき x+y, x-z, y-z が平方数であることを示せ。

http://www.cms.math.ca/Competitions/MOCP/2007/prob_jan.pdf
例 (x,y,z) = (x, x(x-1), x-1)
520132人目の素数さん:2007/02/12(月) 16:22:17
>>519
例の形以外ないじゃん。
521132人目の素数さん:2007/02/12(月) 17:59:58
>>519
なんか、簡単な問題だな……
明らかに、x,y>zが成立する。与式の両辺にxyzをかけて整理すると、条件式は
(x-z)(y-z) = z^2
と変形する事が出来る。
g=gcd(x-z, y-z)とおき、 x-z = a'g、y-z = b'gと置くと、a',b'は互いに素になり
a'b'g^2 = z^2より、a',b'は共に平方数になる。従って、
x-z = a^2g
y-z = b^2g
z = abg
と表せ、
x = a^2g + abg
y = b^2g + abg
z = abg
となる。gcd(x,z)=1より、g=1が成立し
x = a^2 + ab
y = b^2 + ab
z = ab
となり、x+y=(a+b)^2、x-z=a^2、y-z=b^2となって、
多分平方数に近い値をとっている。
522132人目の素数さん:2007/02/13(火) 04:19:03
>>520-521
 どうもでつ
523132人目の素数さん:2007/02/13(火) 05:09:58
〔問.476〕
p >0, |x_0| ≦ 2p とし、
 x_n = 3x_(n-1) -(1/p^2){x_(n-1)}^3   (n≧1)
と定義する。
 x_n を n と x_0 の函数として表わせ。


〔問.478〕
  √{2+√[2+√(2+x)]} + (√3)√{2-√[2+√(2+x)]} = 2x
  x ≧ 0.
を解け。

http://www.cms.math.ca/Competitions/MOCP/2006/prob_dec.pdf
524132人目の素数さん:2007/02/15(木) 15:44:07
x_0=2p*sinθ_0とかx=2cosθっておいて3倍角、半角、合成するだけ。
東大の問題っぽくない。
525132人目の素数さん:2007/02/18(日) 16:36:53
>>507
無視されてるね
円の方程式になるからってやつ?
526132人目の素数さん:2007/02/18(日) 16:47:28
>>524
東大でも、そうやって置くと瞬殺できる問題が出たことあるよ。
527132人目の素数さん:2007/02/18(日) 19:10:20
三つの連続したピタゴラス数の意味が分からないからだと思うけど……
528132人目の素数さん:2007/02/18(日) 20:55:42
( ^ω^)
529132人目の素数さん:2007/02/18(日) 22:29:19
>>525
>>437以降のレスを参照
530132人目の素数さん:2007/02/18(日) 23:31:57
>457,>470
 b = 2a^2 +1 として
 n  = b^2 -1,
 n+1 = b^2,
 n+2 = b^2 +1.
531132人目の素数さん:2007/02/19(月) 00:15:22
>>457,>>470
d = 2c(c+1) として
n = d^2,
n+1 = d^2 + 1^2,
n+2 = (d-1)^2 + (2c+1)^2.


>530
n = (b-1)^2 + (2a)^2,
532132人目の素数さん:2007/02/19(月) 21:48:06
1999年がsin,cosの定義
2003年がπの評価
2007年は…?
今年もこういう系出るかもしれんな。
533132人目の素数さん:2007/02/19(月) 22:34:35
2^√2はどのような実数として定義されるか、とか?
534132人目の素数さん:2007/02/27(火) 04:05:58
今年のは特に面白いのなかったな
535132人目の素数さん:2007/03/04(日) 18:04:10
一応log2の評価だがこの程度はどこの大学でもあるだろうな
536132人目の素数さん:2007/03/11(日) 18:12:46
age
537132人目の素数さん:2007/03/31(土) 23:23:43
@√3-√2
Alog2
Blog(√3-√2)
について、それぞれの値を有理数で評価する場合、その評価の方法について論ぜよ。

5/√10=√10/2を証明せよ。

同数の白玉、黒玉があり、これを円の形に並べる。
(1)8個ずつのとき、並べ方は何通りあるか。
(2)n個ずつのとき、並べ方は何通りあるか。
538132人目の素数さん:2007/04/02(月) 22:47:06
さいころの問題阪大に出てたな
539132人目の素数さん:2007/04/03(火) 06:51:53
1組の対面が白、他の4面が「田」の形に仕切られた立方体があり、
「田」の部分に対して赤を8箇所、青を8箇所塗る。
このとき塗り方は何通りあるか。

1時間じゃ解けんな。
540132人目の素数さん:2007/04/03(火) 07:22:12
>>537
> 5/√10=√10/2を証明せよ。

両辺を平方して等しいことを確認してから両辺共に正であるから等しい,でいいんじゃないの?
中学生レベルだと思うが。
541132人目の素数さん:2007/04/03(火) 09:29:02
>>540
一応俺が言いたかったのはその前の評価の問題の続きみたいな感じで、
5/√10と√10/2がm1<5/√10<m2、n1<√10/2<n2のように
有理数で評価された時に、どのように厳しく評価しようとも
必ずm1<m2<n1<n2またはn1<n2<m1<m2とはならない
ということの証明だったんだけどね。
542132人目の素数さん:2007/04/03(火) 09:30:25
その証明にルートの法則はなんでもかんでも使って良いの?
それじゃ循環論法だよね。

>>537は意味不明だよ。
543132人目の素数さん:2007/04/04(水) 22:49:17
無理もない
なぜなら
537は
素数様だからだ!
544132人目の素数さん:2007/04/04(水) 23:07:04
だれそれ?
545132人目の素数さん:2007/04/05(木) 07:32:42
89 :132人目の素数さん :2006/12/27(水) 00:03:16
B**さいころをn個同時に投げるとき,出た目の数の最大公約数が1になる確率を求めよ
C***半径1,高さ1の円錐を底面に垂直で中心から1/2離れた平面で切断する.断面積を求めよ
http://www.mainichi-msn.co.jp/shakai/edu/jyuken/daigakubetsu07/graph/osaka/sugakua/2.html
546132人目の素数さん:2007/04/05(木) 07:35:38
>>543

537=3*179

素数じゃねーよ!
547132人目の素数さん:2007/04/05(木) 11:07:39
このスレや過去ログを「素数様」で検索してみれば良いと思うよ。
548132人目の素数さん:2007/04/06(金) 22:22:48
>>537だけど白玉、黒玉の問題でn個→2^n個に訂正
まあできる奴は少ないと思うがこれだと綺麗に一般化できる。
549132人目の素数さん:2007/04/06(金) 22:53:26
>545
B**
 最大公約数をGとする。
G=6 : (1/6)^n
G=5 : (1/6)^n
G=4 : (1/6)^n
G=3 : (1/3)^n -(1/6)^n
G=2 : (1/2)^n -(1/6)^n -(1/6)^n
G=1 : 1 - (1/2)^n -(1/3)^n

C***
円錐: z = 1 - √(x^2 + y^2),
 y = 1/2 とおくと
双曲線: z = 1 - √(x^2 + 1/4),
 ∫[-(√3)/2, (√3)/2] z dx
  = [ x -(x/2)√(x^2 + 1/4) -(1/8)ln(2x+√(4x^2 +1)) ](x=-(√3)/2, (√3)/2)
  = (√3)/2 -(1/4)ln(2+√3)
= 0.536785929553234…
550132人目の素数さん:2007/04/07(土) 20:33:31
問題↓
http://www-2ch.net:8080/up/download/1175945435856561.AriV6b
難しいと思うけど・・・。
551132人目の素数さん:2007/04/07(土) 22:14:16
dream come true
552549:2007/04/08(日) 02:38:04
>545

C*** (549の補足)
x = (1/2)sinhθ とおくと dx = (1/2)coshθ・dθ
z = 1 - (1/2)coshθ
∫z dx = ∫{(1/2)-(1/4)coshθ}coshθ・dθ
  = ∫{ (1/2)coshθ -(1/8)cosh(2θ) -(1/8) } dθ
  = (1/2)sinhθ -(1/16)sinh(2θ) -(1/8)θ +c
  = (1/2)sinhθ -(1/8)sinhθcoshθ -(1/8)θ +c
  = x -(x/2)√(x^2 + 1/4) -(1/8)ln(2x+√(4x^2 +1)) +c.
553132人目の素数さん:2007/04/08(日) 03:09:55
>550
nを1つ固定し、部分和を
 Σ[k=1,n] a(k) = A(n),
 Σ[k=1,n] b(k) = B(n),
 Σ[i=1,n] c(i) = C(n),
 Σ[i=1,n] d(i) = D(n),
とおく。
与式の右辺の2つの行列式は、2行目以外は一致する。
そこで、行列式が2行目について線形であることを用いて、2行目どうしを加えると
 | a(k), b(k) |
 | C(n), D(n) |
となり、1行目以外は定数になる。
そこで、行列式が1行目について線形であることを用いて、1行目の Σ[k=1,n] をとると
 | A(n), B(n) |
 | C(n), D(n) |
となる。
最後に n→∞ の極限をとる。
554553:2007/04/08(日) 03:24:09
>550 (続き)

n→∞ の極限は、題意により
| 1, 2 |
| 3, 4 |
に収束する。
555550:2007/04/08(日) 16:24:54
二次の正方行列A1,A2,A3が
|A1+A2|=α,|A2+A3|=β,|A3+A1|=γ
を満たすとする。
このとき、
|A1|+|A2|+|A3|+|A1+A2+A3|
の値を求めよ。

550をちょっと変えて(1)に、これを(2)にするといいかも。
556132人目の素数さん:2007/04/09(月) 14:13:16
a[0]=0,a[n]=sqrt((1+a[n-1])/2)で数列{a[n]}を定義するとき、
lim[n→∞](a[1]a[2]…a[n])を求めよ。
557132人目の素数さん:2007/04/09(月) 17:47:08
どっちもわかんね
558132人目の素数さん:2007/04/09(月) 18:10:17
なんかすごいできる人がいるお
【頂上】ちんちんvsいうおいじょ【決戦】
http://school7.2ch.net/test/read.cgi/jsaloon/1175709295/
559132人目の素数さん:2007/04/09(月) 22:53:40
>556

|a[0]| ≦1 のとき
 a[k] = cos(θ/(2^k)) = sin(θ/(2^(k-1)))/2sin(θ/(2^k)), 
 θ = ±arccos(a[0]),
 a[1]a[2]…a[n] = sinθ/{(2^n)sin(θ/(2^n))} → (sinθ)/θ.

|a[0]|≧1 のとき
 a[k] = cosh(t/(2^k)) = sinh(t/(2^(k-1)))/2sinh(t/(2^k)),
 t = ±log(a[0]+√{a[0]^2 -1}).
 a[1]a[2]…a[n] = sinh(t)/{(2^n)sinh(t/(2^n))} → sinh(t)/t.
560132人目の素数さん:2007/04/10(火) 20:32:50
だれか>>559の解説よろ
561559:2007/04/10(火) 20:50:45
>556

a[0] を[-1,∞) の範囲で変えてみますた。

どの場合も単調に1に近づきますた。

下の方は a[0] ≧1 のとき、ぢゃないとやばいっすね。
562132人目の素数さん:2007/04/10(火) 21:43:34
俺も>>559の言ってることがわからんww
適当にやったら2/piになった
563132人目の素数さん:2007/04/10(火) 22:33:27
>>562
正解
564132人目の素数さん:2007/04/10(火) 22:42:26
つまり、>>559 は毒電波ということだ!
565132人目の素数さん:2007/04/10(火) 22:51:19
>>562
解答plz
566132人目の素数さん:2007/04/10(火) 23:00:01
>>565
ここのサイトの下のほうに「いうお問全集」ってのがある。それの4問目らしい。
ttp://homepage2.nifty.com/enjoying_math/index.htm
567132人目の素数さん:2007/04/10(火) 23:00:30
√3+√2>πを証明せよ。電卓の使用は認めない。
568132人目の素数さん:2007/04/10(火) 23:10:01
>>566
トンクス

これ作った人すげえな……
569132人目の素数さん:2007/04/11(水) 13:47:00
555の解答はまだー?
570132人目の素数さん:2007/04/11(水) 14:56:47
>555,569
 (x1+x2)*(y1+y2) + (x2+x3)*(y2+y3) + (x3+x1)*(y3+y1) = x1*y1 + x2*y2 + x3*y3 + (x1+x2+x3)*(y1+y2+y3),
∴ (与式) = α + β + γ.
571132人目の素数さん:2007/04/11(水) 15:33:03
>567

 1.414^2 = 1.4^2 * 1.01^2 = 1.96 * 1.0201 = 1.999396 < 2,
 1.732^2 = (4*0.433)^2 = 16 * 0.433^2 = 2.999824 < 3,

∴ √3 + √2 > 1.732 + 1.414 = 3.146

 となるから、 3.146 > π を示すことに帰着する。
572132人目の素数さん:2007/04/11(水) 16:05:47
こういう評価の問題って、
1-1/3+1/5-+.........
の最初の10^10項を計算するとこうなるので、
とかさらっと書いたらやっぱ誤答扱いになるんだろうか
573132人目の素数さん:2007/04/11(水) 16:25:38
10^10項で十分であることの証明がなきゃ、もちろんバツだろうな。

数値評価は、いくらでも難しい問題がお手軽に作れる上に、
採点者は大変だし、解答する方もあまり面白くない。
よほど練られた特殊な場合以外、試験問題としては不適切だろう。
574132人目の素数さん:2007/04/11(水) 17:43:40
関数f(x)は次の条件を満たすとする。
(1)f(x)は微分可能であり、f '(x)は連続関数
(2)|x|≦1のとき|f '(x)|<1
(3)f(0)=0
このとき、次のように定義される数列{an}についてlim[n→∞]anを求めよ。

|a0|<1 , a[n+1]=f(an) (n=1,2,…)
575132人目の素数さん:2007/04/11(水) 19:01:58
α∈[1,∞]∨[-∞,-1]∨{0} に収束するように f をつくれる。
      ↑a_n→∞,-∞の意味で閉区間にした。
576132人目の素数さん:2007/04/11(水) 19:49:07
ん?lim[n→∞]an=0になるはずなのだが。
577575:2007/04/11(水) 19:51:57
うん、ごめん、勘違いしてた。
578132人目の素数さん:2007/04/12(木) 20:51:16
空間上に有限個の点がある。
ここで任意の2点を結んだ直線上に別の点があるとすれば
すべての点は同一直線上にあることを示せ。

雑誌に載ってたから有名問題かも。
579132人目の素数さん:2007/04/15(日) 21:05:11
加法定理、π>3.05の証明ときて次はなにを証明させるだろう?
lim[x→0](sinx)/x=1とか?
580132人目の素数さん:2007/04/15(日) 21:09:14
>>579
それ、入試頻出だから。
581132人目の素数さん:2007/04/15(日) 22:24:18
素数が無限に存在することの証明
582132人目の素数さん:2007/04/15(日) 22:44:39
>556
つ[参考文献]

1. 「数学の問題 = 第B集」 数セミ増刊, 日本評論社 (1988/09/20) の No.16 (解説 牛島照夫氏)
2. 高木: 「解析概論」 改訂第3版, 岩波書店 (1961/05/27) の練習問題(1)-(2), p.33
583132人目の素数さん:2007/04/16(月) 00:27:30
>578

背理法による: 「n個の点すべてが同一直線上にあるのではない」と仮定する。
Lはn点のうち少なくとも2点をとおる直線とする。
L上にない点PからLまでの距離を d(P,L) で表わすとする。
仮定より、L上にない点Pが存在する。
集合 S = { d(P,L) } は空ではなく、かつ有限である。
それゆえ、Sは 極小元 d(P0,M) をもつ。
題意より、M は与えられたn点のうちの3点以上、たとえば P1,P2,P3 を通る。
点P0 から M に下ろした垂線の足を Q とする。
3点 P1,P2,P3 のうちの少なくとも2点、たとえば P2, P3 は Q に関して同じ側にある。(ひとつは Qに一致してもよい).
 いま, P2 は P3 より P0 に近い点であると仮定してよい。
2点 P0 と P3 をとおる直線をNと表わすとき、
 d(P2,N) < d(P0,M)
となるが、これは点P0とMの選び方に矛盾する。
したがって、n個の点はすべて同一直線上にある。(終) 

つ(参考書)
秋山 仁 + ピーター・フランクル: [完全攻略] 数学オリンピック, 第1版, 日本評論社, p.91-92 (1991/11/20)
 平面幾何[10]
584132人目の素数さん:2007/04/16(月) 00:38:20
点の個数に関する帰納法で一瞬だと思うんだけど
585132人目の素数さん:2007/04/16(月) 00:41:25
>>584
一瞬じゃねーし。低脳は死ね。
586132人目の素数さん:2007/04/16(月) 01:15:06
空間に距離が入るとか勝手に仮定して良いの?
587132人目の素数さん:2007/04/16(月) 01:23:14
詳しくないんだがこういう問題ではどこまで仮定されてるの?
588132人目の素数さん:2007/04/16(月) 01:26:02
>>583
 わかるな、こういう感覚。サービス精神旺盛なのよね?
 ちょっと無理しちゃっても、いいのよいいのよ〜 ノリでいいのよ♪ 
>>586
 深く考えないでおきましょう(超・楽天家)
589132人目の素数さん:2007/04/16(月) 13:00:00
(Z/3Z)^2。
590132人目の素数さん:2007/04/16(月) 17:08:04
>>586
有限体上の空間だと、問題の反例が作れるので、
代数的な議論だけでは証明できないような気がする。
591132人目の素数さん:2007/04/17(火) 00:12:52
エルデシュだか誰だかが距離を使わずに証明してたよ。そのかわり、順序構造みたいな
ものを仮定して、それを使ってた気がする。
592132人目の素数さん:2007/04/17(火) 02:05:17
>581
 つ〔参考書〕

M.アイグナー(著), G.M.ツィーグラー(著), 蟹江幸博(翻訳)「天書の証明」 シュプリンガー・フェアラーク東京,
第1章 「素数は無限: 6つの証明」

価格: \3675
単行本: 313p.
出版年月: 2002/12/
ISBN-10: 443170986X
ISBN-13: 978-4431709862
寸法: 21.2 x 18.6 x 2.8 cm
593132人目の素数さん:2007/04/17(火) 02:36:07
そうやって参考書を出されてもな
そんなもの見に行く暇なんかないわけで

>>581
素数が有限個と仮定、そのうちの最大の素数をpとしてq=p!+1を考えると
qをp以下のどのような素数で割っても1余るからqは素数であり、明らかにp<qだから矛盾

こんな感じでいいかい?
594132人目の素数さん:2007/04/17(火) 06:20:23
>>593
> そうやって参考書を出されてもな
> そんなもの見に行く暇なんかないわけで

> そうやって参考書を出されてもな
> そんなもの見に行く暇なんかないわけで

> そうやって参考書を出されてもな
> そんなもの見に行く暇なんかないわけで

> そうやって参考書を出されてもな
> そんなもの見に行く暇なんかないわけで

> そうやって参考書を出されてもな
> そんなもの見に行く暇なんかないわけで

> そうやって参考書を出されてもな
> そんなもの見に行く暇なんかないわけで

6つの証明をさっさと書けや、ゴルァ!
595132人目の素数さん:2007/04/17(火) 06:30:53
>>594
スレ違い
596132人目の素数さん:2007/04/18(水) 01:42:57
>>581
 自然数nは、素数べきの積に分解できる:
 n = Π[p:素数] p^(e_p), e_pは負でない整数。
よって
 Σ[n:自然数] 1/n = Π[p:素数] 1/{1-(1/p)}  …… (オイラー積表示)
アルキメデスの原理を使えば、左辺が発散することを示せる。
素数が有限個と仮定すると、右辺は有限項の積だから有限となり、明らかに矛盾。

こんな感じでいいかい?
597132人目の素数さん:2007/04/18(水) 02:02:54
>>596
単なる知識問題だな。
私大文系の社会と変わらん。

加法定理の証明も円周率の近似も事実上知識問題だが。

俺はもっと作為的なパズル問題が好きなんだが。
99年2番とか98年4番とか97年2番とか91年5番とか90年6番とか
93年文系4番とか90年文系2番とか。
598132人目の素数さん:2007/04/18(水) 02:11:36
ゼータ関数も知らないとは
599132人目の素数さん:2007/04/18(水) 02:34:06
>>597
A={(2^a)(3^b)(5^c)|a,b,cは非負整数}とおき、A[m]={x1+x2+…xm|x1,x2,…,xm∈A∪{0}}と
おく。任意の自然数mに対して、A[m]に含まれない自然数が無限に存在することを示せ。
600132人目の素数さん:2007/04/18(水) 02:39:33
読み間違える可能性があるので ちょっと訂正。

任意の自然数mに対して、A[m]に含まれない自然数が無限に存在することを示せ。

P(m):A[m]に含まれない自然数が無限に存在する
とおくとき、任意の自然数mに対して、P(m)は真であることを示せ。
601132人目の素数さん:2007/04/18(水) 08:15:29
>>597
加法定理の証明は教科書に載ってるよ。
602132人目の素数さん:2007/04/18(水) 09:11:11
>>601
だからなんだ。
603132人目の素数さん:2007/04/18(水) 09:18:02
>>597
受験数学厨必死だな
604132人目の素数さん:2007/04/18(水) 09:21:22
どの辺が必死に見えた?
受験数学は好きだけど。
605132人目の素数さん:2007/04/18(水) 13:38:23
606132人目の素数さん:2007/04/18(水) 18:00:00
log(x)=o(x^a)(0<a)。
607132人目の素数さん:2007/04/18(水) 22:55:09
>>594

第1の証明(ユークリッド):
 >>593
 http://mathworld.wolfram.com/EuclidsTheorems.html

第2の証明:
 F_n = 2^(2^n) +1 とおく(フェルマー数)。F_n は奇数である。
 nについての帰納法により, F_0*F_1*…*F_(n-1) = F_n - 2.
 ∴ m≠nならば LCD(F_m, F_n) は1か2だが、奇数だから1,
 ∴ F_0, F_2, …, F_n はすべて互いに素
 http://mathworld.wolfram.com/FermatNumber.html

第3の証明:
 pは素数とし、(2^p)-1 の素因数の1つをqとする。
  2^p ≡1 (mod q)
 剰余類体F_q の乗法群は位数q-1だから p|(q-1)
 ∴ pより大きい素数qがある。 
http://mathworld.wolfram.com/GroupOrder.html

第4の証明:
 >>596

第5の証明(H.ヒュルステンベルク):
 トポロジーを使うらしい。

第6の証明(P.エルデシュ):
 オイラーによる式
  Σ[p:素数] 1/p = ∞ 
 を使うらしい。
608132人目の素数さん:2007/04/18(水) 23:56:19
第5の証明:
a>0,b∈Zに対してV(a,b)={an+b|n∈Z}とおく。B={V(a,b)|a>0,b∈Z}とおくとき、
Bはある位相の開基となるための条件を満たす。すなわち、
[1]∪B=Z
[2]B1,B2∈B,x∈B1∩B2ならばx∈B3⊂B1∩B2を満たすB3∈Bが存在する
が成り立つ。そこで、Bを開基とする位相をθとすれば、θは次の性質を満たす。
(1)φ≠O∈θは無限集合である。
(2)V(a,b)∈Bは開集合であるが、同時に閉集合でもある。
さて、素数が有限個しかないとすると、それをp1,p2,…,pnとすれば、Z−{−1,1}=∪[i=1〜n]V(pi,0)
となる。右辺のV(pi,0)は閉集合であるから、その有限個の和集合である∪[i=1〜n]V(pi,0)もまた閉集合
となる。よってZ−{−1,1}は閉集合であり、{−1,1}は開集合となる。ところが、(1)より、{−1,1}は
無限集合でなければならず、矛盾。

[1]…U(1,0)=Zなので明らか。
[2]…Bi=V(ai,bi)とするとき、d=gcd(a1,a2)としてc=a1a2/d とおけばB3=V(c,x)が
求める集合である。x∈B3は明らか。簡単な計算によりcm+x≡bi (mod ai)となることが
分かるので、B3⊂B1∩B2も成り立っている。

(1)…O∈θ,O≠φとすると、B1⊂Oを満たすB1∈Bが取れるので、B1=V(a,b)とすると、
V(a,b)は明らかに無限集合であり、よってOも無限集合となる。
(2)…Z−V(a,b)=∪[0≦r<a,r−bはaで割り切れない]V(a,r)∈θ であるから、V(a,b)は閉集合となる。
609132人目の素数さん:2007/04/19(木) 00:18:03
すげえ…こんな証明初めてみた…
U(1,0)?
610608:2007/04/19(木) 00:21:18
V(1,0)の間違いです(> <)
611132人目の素数さん:2007/04/19(木) 00:22:26
素数の積+1の素因数を考えるのを遠回りにやってるだけ
612132人目の素数さん:2007/04/19(木) 07:46:37
>>611

こういう奴は何も分かっていない。遠回りであれ何であれ、
「素数の積+1の素因数を考える」という代数的な操作を、位相空間の
議論に置き換えたり、解析的な議論に置き換えることが出来るところに
価値がある。世の中が>>611みたいな奴ばかりだったら、整数論は中学生
レベルの初等的なショボイ整数論から発達していない。
613132人目の素数さん:2007/04/19(木) 08:16:13
>>602
教科書に載ってるのは知識問題とは言わない。
全員必修なんだから。
例えば微分を知らないと解けない問題で、「俺は微分なんて知らない、こんなの知識問題だ。」とは言わないだろ。
614132人目の素数さん:2007/04/19(木) 09:20:16
>>613
それは違うんじゃないか?
「微分を知らないと解けない問題」と「微分を知ってるかどうかだけで決まる問題」との差があるじゃんか。

まあどうでもいい議論だとは思うけど。
615132人目の素数さん:2007/04/19(木) 13:38:00
>>537
問3
(1)810通り
(2)Σ[k=1,n+1]
{C[2^k,2^(k-1)]-C[2^(k-1),2^(k-2)]}/2^k 通り
616132人目の素数さん:2007/04/19(木) 13:38:59
(2)は2^nのときについて
617132人目の素数さん:2007/04/19(木) 16:57:14
>>608-609
トポロジーと素数に関係を見出すなんて洒落た証明だなwww
618132人目の素数さん:2007/04/19(木) 18:51:35
>>612
遠回りに同意?
619132人目の素数さん:2007/04/19(木) 22:39:58
620132人目の素数さん:2007/04/20(金) 03:53:03
新しい証明を思いついた
621132人目の素数さん:2007/04/20(金) 11:57:11
だがそれを書くにはスペースが少なすぎる。
622132人目の素数さん:2007/04/20(金) 16:42:29
フェルマー乙
623132人目の素数さん:2007/04/20(金) 23:48:52
>>607-608
単なる言い換えと言われればそれまでだけど、マジで感心した。
大学院入試なら出ても全然おかしくないレベルだし、用語を控えたら
かなりの部分は大学入試並。
624132人目の素数さん:2007/04/22(日) 03:53:03
第7の証明:
a>0,b∈Zに対してV(a,b)={an+b|n∈Z}とおく。B={V(a,b)|a>0,b∈Z}とおくとき、
θ={∪[i=1〜n]Bi|Bi∈B}とすれば、
[1]Z∈θ
[2]B1,B2∈B,B1∩B2≠OならばB1∩B2∈Bである
[3]T1∈θならばT1^c∈θである
[4]T1,T2∈θならばT1∪T2∈θである
が成り立つ。よって、θは有限加法族であり、次の性質を満たす。
(1)φ≠T1∈θは無限集合である。
さて、素数が有限個しかないとすると、それをp1,p2,…,pnとすれば、Z−{−1,1}=∪[i=1〜n]V(pi,0)
となる。右辺の∪[i=1〜n]V(pi,0)はθの元なのでZ−{−1,1}はθの元であり、{−1,1}はθの元となる。
ところが、(1)より、{−1,1}は無限集合でなければならず、矛盾。

[1]…V(1,0)=Zなので明らか。
[2]…Bi=V(ai,bi)とするとき、x∈B1∩B2,d=gcd(a1,a2)としてc=a1a2/d とおけばB1∩B2=V(c,x)
である。y∈B1∩B2⇔ai|(y-x)⇔c|(y-x)⇔y∈V(c,x)。
[3]…T1=∪[i=1〜n]Bi,Bi=V(ai,bi)とするとき、
T1^c=∩[i=1〜n]∪[0≦ri<ai,ri−biはaiで割り切れない]V(ai,ri)
=∪[1≦i≦nの任意のiについて0≦ri<ai,ri−biはaiで割り切れない]∩[i=1〜n]V(ai,ri)∈θ。
[4]…明らか。

(1)…T1∈θ,T1≠φとすると、B1⊂T1を満たすB1∈Bが取れるので、B1=V(a,b)とすると、
V(a,b)は明らかに無限集合であり、よってT1も無限集合となる。
625132人目の素数さん:2007/04/22(日) 05:31:48
>[3]…T1=∪[i=1〜n]Bi,Bi=V(ai,bi)とするとき、
Bは可算集合だから、θの元の一般形は∪[i=1〜∞]Bi,Bi=V(ai,bi)
になる。この形で証明しなければならない。
626132人目の素数さん:2007/04/28(土) 04:26:34
x^2+2y^2-4x+y+xy=0を満たす整数解の組を求めよ
627132人目の素数さん:2007/04/28(土) 05:45:05
(x,y)=(0,0),(4,0)
628132人目の素数さん:2007/04/29(日) 00:21:41
↓のスレにいっぱい問題投下されてるお
http://school7.2ch.net/test/read.cgi/jsaloon/1175709295/l50
629132人目の素数さん:2007/04/29(日) 00:22:59
ほとんどが学コンの過去問だからなあ・・・
630132人目の素数さん:2007/04/29(日) 00:34:25
20年くらい前の宿題の不動二次曲線(楕円、双曲線、放物線)をもつ
一次変換シリーズは面白かった。
631132人目の素数さん:2007/04/29(日) 01:13:36
>>630
面白そうですね。 さっさと うpれカス!
632132人目の素数さん:2007/04/29(日) 01:38:07
うpれなくてごめんねクズ
要は、標準基底をとったときに、条件を満たす行列の成分を求めろってことだゴミ
双曲線限定なら破格に簡単鴨、とだけ言っておいてやルンペン

それなりの、統一条件解よろしこ>>631
633132人目の素数さん:2007/04/29(日) 20:48:05
>>632
うpれカス! ニヤニヤ…
634132人目の素数さん:2007/04/30(月) 23:39:22
S_n=Σ[k=1→n]1/(n+k)とするとき、lim_[n→∞]n(log2-S_n)を求めよ。
635132人目の素数さん:2007/05/01(火) 17:18:37
>>630
うpれカス!
636132人目の素数さん:2007/05/01(火) 17:20:03
>634
 やっぱり I=∫[n,2n] (1/x)dx と比べるんだろうな。
 f(x)=1/x は下に凸なので、 割線 > 曲線 > 接線.

 (1/2)(1/n) + Σ[k=1,n] 1/(n+k) + (1/2)(1/2n) > I > c_1 + Σ[k=1,n-1] 1/(n+k) + c_2,
 S_n + 1/(4n) > I > S_n -(1/2n) + c_1 + c_2,
 f '(n) = -1/(n^2) より f(n +1/2) > f(n) -1/(2n^2),
 f '(2n) = -1/(4n^2) より f(2n -1/2) > f(2n) + (1/8n^2),
 c_1 = {高さが1/2, 底辺が f(n), f(n +1/2) の台形の面積} = (1/4){(2/n) - 1/(2n^2)},
 c_2 = {高さが1/2, 底辺が f(2n), f(2n -1/2) の台形の面積} = (1/4){(1/n) + (1/8n^2)},
 S_n + 1/(4n) > I > S_n + 1/(4n) - 3/(32n^2),
 1/4 > n(I-S_n) > 1/4 -3/(32n).
637636:2007/05/01(火) 17:32:05
>636 の訂正,すまそ.

 c_1 = {高さが1/2, 底辺が f(n)=1/n, f(n) -1/(2n^2) の台形の面積} = (1/4){(2/n) - 1/(2n^2)},
 c_2 = {高さが1/2, 底辺が f(2n)=1/(2n), f(2n) +1/(8n^2) の台形の面積} = (1/4){(1/n) + (1/8n^2)},

x=n, x=2n での接線を考えますた。
638132人目の素数さん:2007/05/01(火) 20:37:48
素数様の時代は華があった
639132人目の素数さん:2007/05/02(水) 15:56:26



連日1次方程式


5a+12b+13c+9d =13
7a+19b+19c+14d=19
14a+41b+41c+28d=41
15a+36b+40c+27d=38
 にCramerの公式を適用してbの値を求めよ。ただし公式を適用した
式を明記した上で、計算の方法や計算過程がわかるように、途中の計算式
を省略せずに書くこと。


640132人目の素数さん:2007/05/02(水) 17:14:49
何で外国人を雇わなきゃならないんだ
641132人目の素数さん:2007/05/02(水) 22:56:05
            ,,.、-‐…¬ー- 、
         ,、‐'"          ヽ,
     ,、、,/    ,,、        ゙`ヽ,
.   ,、-' ,.':...  ,、-‐'"       ,、  .,r'  ゙、     
.  (  l:/::::::;/      ,,、 -''" ,、 ' .',.  ',        
,、‐'"~>'ヽ:;::...,__   __,,,..、 -‐',、-'゙ l.  l   l    宿題は質問スレに行けよ〜
   / '  ゙〉、、二二、 -‐ '''"     |.  l i i l      
  ヽ、r''''ーイ            ,、‐.l ,  l ! l.|     馬鹿ですか?
   ..:,i゙   i|     ,   ヾ''''''''"´   |.l  |. l .l.!      
ヽ、ィ/ | i !il,  _,、-'゙        ,.r l |. | l :l.l'"""ヽ   
.,、-‐‐''ヾ、l::i゙ト、    ,,、   ー‐''"  | ! l:;'.:i !   / l   ,r''j
-'ー-‐   ヾ:l、、}.ゝ‐'''"    ,r、   l.|l .l:':::i/  / j ,r'''" /、,、-ッ、
三-   ヽ  {:rl: '、,      <、丿,、-|.li./:/! ./  ハ.{ 、   ニ,フ
-、゙_____,,. _,ノ゙〉.  l゙'''' ー-- r‐''i,7 ,.イl:;ン゙ ,| ./  ノ }  `゙''  ‐'シ
     ゙i::::::: ゙i''ト, ゙、.l    ゙、. / ./ /´   V     V、,ヽ,,..、 ‐'"
     l:::::::  'ij.゙、、, !     /  /       l  ,r /  .::::j、
    /l:::::::./ jヽ'! ', .   / ./      i  ,..{. ヽ .:::::/ ヽ
.   i゙  l:/   ' ゙!  ', ゙'''ト-i'、.,,,,_    ,イ /゙、. ゙、:::/  ノj
642132人目の素数さん:2007/05/03(木) 04:01:19
a_n>0,a_1=1,a_(n+1)^3 - 3a_(n+1) - a_n = 0のときlim[n→∞]9^n*(2-a_n)を求めよ
643132人目の素数さん:2007/05/03(木) 21:04:34
これは興味深い
644132人目の素数さん:2007/05/03(木) 22:39:49
>>642
答えを教えてください、さっさと うpれカス!
645132人目の素数さん:2007/05/04(金) 00:14:24
個人的には三倍角の公式辺りを使いそうだって思ったりしたわけだが、使わないのかな?
まぁいいや。
646132人目の素数さん:2007/05/04(金) 02:42:42
647132人目の素数さん:2007/05/04(金) 22:36:52
>>645
それで合ってるんじゃね?
648132人目の素数さん:2007/05/05(土) 22:03:39
よろしければ、解答を うpれカス! でございます。
649132人目の素数さん:2007/05/05(土) 22:54:40
>648
 つ a_n = 2cos(π/{3^n}),
 π^2 ぐらいか…
650132人目の素数さん:2007/05/05(土) 23:58:48
なんつーこたない
651132人目の素数さん:2007/05/06(日) 14:28:41
>>640 関孝和様が間違っていたからではなかろうか?
652132人目の素数さん:2007/05/06(日) 16:20:28
>>17の11とけたかな?

f(x)+g(x)+h(x)=ax^3+bx^2+cx+d=F(x)とおく
F(1)=a+b+c+d=6    ・・・@
F(2)=8a+4b+2c+d=6 ・・・・A
F(3)=27a+9b+3c+d=6 ・・・・B
A−@:7a+3b+c=0・・・C
B−@:26a+8b+2c=0・・・D
B-A:19a+5b+c=0・・・・E
D-C:19a+5b+c=0・・・F
D−E:7a+3b+c=0・・・・G
12a+2b=0
b=-6a
c=11a
6a=6-d
よってd=6以外のときF(x)は三次関数。よって最小値最大値は存在しない
d=6のときF(x)=6となり、これも最大値最小値が無いのは明らか。
証明終
653132人目の素数さん:2007/05/06(日) 16:25:36
最大値=最小値=6 だと思うが。
654132人目の素数さん:2007/05/06(日) 16:42:13
こんな感じでどうだ
k(x)=(x-1)(x-2)(x-3)とおく
f(1)=g(2)=h(3)=1
f(2)=g(3)=h(1)=2
f(3)=g(1)=h(2)=3
を満たす2次以下のf,g,hをf0,g0,h0とすると、(できる、ここはさぼり)
定数a,b,cによってf=f0+ak,g=g0+bk,h=h0+ckとかける。このとき
f+g+h=f0+g0+h0+(a+b+c)k
l=f0+g0+h0 とおけば、lは2次以下でl(1)=l(2)=l(3)=6
よってl≡6となり、f+g+h=6+(a+b+c)k
k=0のとき、6が最大かつ最小というのが普通の数学用語かな
655132人目の素数さん:2007/05/06(日) 16:44:28
>>653
リロードすべきだった、最後a+b+c=0のとき、ね
656652:2007/05/06(日) 16:51:46
また更にd=6のときはありえないことを示さないといけないって事?
面倒だな、やっぱ偏差値50じゃ解けないか
657132人目の素数さん:2007/05/06(日) 16:56:00
>>656
ありえる。出題ミスでしょう。
654もこの問題ではやり過ぎ
f+g+h-6=a(x-1)(x-2)(x-3)で十分だな
658132人目の素数さん:2007/05/07(月) 21:50:49
このスレが質問スレと化している件について
659132人目の素数さん:2007/05/07(月) 22:20:04
学力テストのB問題(論理・記述式)使って、東大クラスの問題作ろうぜ
660132人目の素数さん:2007/05/07(月) 23:00:24
>>659
噛み砕いて言って
661132人目の素数さん:2007/05/08(火) 02:36:48
√x-√yと√(x-y)とlog(x-y)の大小を調べよ、ただしx,yは実数とする
662132人目の素数さん:2007/05/08(火) 07:10:04
>x,yは実数
663132人目の素数さん:2007/05/08(火) 13:01:58
>>660
ttp://www.yomiuri.co.jp/feature/gakuryoku/t3suub-exam.pdf
この問題のことじゃね?
要は、この設定を使って東大クラスの問題を作ろうってさw
664132人目の素数さん:2007/05/08(火) 13:02:43
× 作ろうってさ
○ 作ろうってことさ
665132人目の素数さん:2007/05/08(火) 20:21:41
>>663携帯でごめん(-_-;)
666132人目の素数さん:2007/05/08(火) 21:42:49

             ,- ´  ̄ ` − 、    ,-‐ 、 
           /   . . . . . . .     ヽ´彡´ヽ: ヽ
      / ̄二/: : : : : : : : : : : : : ヽ: : : : : :ヽヽ   ヽ:.ヽ
     /:.厂 /: : : : : : : : : : : : | : : : : | ヽ: : : : .│ヽ   ヽ:.ヽ
     /:./ │.:.:.:.:.:.|:.:.:.:.:.:.:.:.:.:.|_,ィ─−ヽ:.:.:.:.:.:.│.ヽ   ヽ:.ヽ   ∧
    /:./ │:.:.│:.| ヽ:.:.:.:.:.:.:.│ ヽ│ヽ |:.:.:.:.:.:.│.:.|    |:.:.|  ノ |
.    |:.:.|  │:.:.│.|/ ̄.:.:.:.:.:│ _V_ ヽ|:.:.:.:.:.:│.:.:|    |:.:.| (::::/
    |:.:.|  .│:.:.:.|│ V ヽ:.:..‖  ´ て:ヽ |.:.:.:.:.| :.:.::.|    .|:.:.|. .∩
.   |:.:.|  │:.:.:. .ゝ r'て.|\|    i:.::ゝ`|:.:.:.│.:.:.:.:...|    |:.:.|. | |
   |:.:.|    │:.:.:.:.ヽ│ |:.:.ゝ     `´ │:./ノ:.:.:.:.:.:.:|    |:.:.|.| |
   |:.:.|     \:.:.:.:.ヽ, `´  ` _ρ   レ イ:.:.:.:.:.:.:.:.:.:..|.   |.::.|| |∩
   |/´ ̄勹  `ヘ.、_`ゝ   `     ,イ:.:.:.:.:.:.:.:.:.:.:.:.:.:.|.   ∩‖|| |
   | | | / / ヽ /:.:.:.:.:.:.:.ヽ 、_ _ _ , - ´ ,| ̄\ :.:.:.:.:.:.:.:.:.|  .ノ .| | |. |
| ̄ ̄ ヾゞ` ̄ ̄ ̄| :.:.:.:.:.:.| /|` _ -‐'    \:.:.:.:.:.:.:.|. / . , ヘ |
|     五  .お |  ,─フ  レ ' ´    /´ ̄`ヽ:.:.:.:.::|. ; ( ,ヘ, _ ヽ
|  負  月  ま |/: : : ヽヘ─' ´ ̄ >/  ___|.:.:.:.:.|. | ( へ _ |
|  .け  .病  ..い |: : : : :/ |: : : : : : : / :´   |:::鬥:::|:..:.:.:.:|. | ( へ  ノ
| . ん . な  .ら |_ , /. .|: : : : : : / :    |::ミ彡:|:.:.:.:. /   .| |./
|  な . ん     .|: : / `´\: : :: /  :    |   |:.:.:.:./    .| |
|  ヨ  か     .|.: |   │\/    |    ><-/    /| |
|  ! . に     .|__|   .│: : : ヽ   |_ ,- ´   /     /. -
───────  : : : : └─'´    |´  , -/     /
667132人目の素数さん:2007/05/09(水) 02:41:25
>>663
感動した。絵と写真が多くて。
668 ◆oGVH7uagAU :2007/05/09(水) 03:25:45
てす
669 ◆sbaPf1Rw1s :2007/05/09(水) 03:26:36
やた
670132人目の素数さん:2007/05/09(水) 11:24:50
>>639
毎日そんな事をやっているのですか?
671132人目の素数さん:2007/05/09(水) 19:21:29
>>670
東大生は解ける?
672132人目の素数さん:2007/05/09(水) 19:24:36
>>671
DQN大生でも解ける
673DQN大生:2007/05/10(木) 00:25:27
>>639

5a + 12b + 13c + 9d = 13 …… (1)
7a + 19b + 19c + 14d = 19 …… (2)
14a + 41b + 41c + 28d = 41 …… (3)
15a + 36b + 40c + 27d = 38 …… (4)

(4) - (1)*3    より c = -1,
{(3)-(2)*2}/3   より b + c = 1,
{(2)*41-(3)*19}/21 より a + 2d = 0,
{(1)*40-(2)*13}  より 5a + 12b + 9d = 26,
674132人目の素数さん:2007/05/10(木) 15:47:49
>>673
途中の式はないんですか( ̄ー+ ̄)
675132人目の素数さん:2007/05/11(金) 01:29:19
>674
>673 より b,c が出るので、未知数は a,d の2つだけ。
  a + 2d = 0,
5a + 9d = 26 - 12b = 2,
よって
 (a,b,c,d) = (4,2,-1,-2)
676132人目の素数さん:2007/05/11(金) 04:50:49
>>675
答えは、
5a + 12b + 13c + 9d = 13 …… (1)
7a + 19b + 19c + 14d = 19 …… (2)
14a + 41b + 41c + 28d = 41 …… (3)
15a + 36b + 40c + 27d = 38 …… (4)

(4) - (1)*3    より c = -1,
{(3)-(2)*2}/3   より b + c = 1,
{(2)*41-(3)*19}/21 より a + 2d = 0,
{(1)*40-(2)*13}  より 5a + 12b + 9d = 26,
より b,c が出るので、未知数は a,d の2つだけ。
  a + 2d = 0,
5a + 9d = 26 - 12b = 2,
よって
 (a,b,c,d) = (4,2,-1,-2)


って答えを書けば、正解なんですか!

凄すぎです!
これって難しい問題だったんですね。
ありがとうございました!
677132人目の素数さん:2007/05/11(金) 05:16:20
釣りかもしれんが・・・

それは中学生でもとける問題だ
多分Cramerの公式を使う練習に出されてるんだろう
それ丸写ししても大学のレポートだったら再提出だな
678132人目の素数さん:2007/05/11(金) 14:45:14
>>677
何で大学のレポートだったら再提出なんですか?
679132人目の素数さん:2007/05/11(金) 15:00:18
試しに出してみたら。
680132人目の素数さん:2007/05/11(金) 15:22:42
>>678
学校辞めて働け! 反吐が出る! (゚Д゚)≡゚д゚)、カァー ペッ!! >678
681132人目の素数さん:2007/05/11(金) 15:27:01
出さなければどうなるかは知らんが
出したら授業出て無い/聞いてないのバレバレ
682132人目の素数さん:2007/05/11(金) 15:33:48
というか、一般論として、大学で出された課題を初等的に解くと
合格したことに浮かれて勉強サボってますと告白するだけなので
教官に対する印象はものすごく悪いわな。
683132人目の素数さん:2007/05/11(金) 15:46:05
>Cramerの公式を適用してbの値を求めよ。ただし公式を適用した
>式を明記した上で、計算の方法や計算過程がわかるように、途中の計算式
>を省略せずに書くこと。

これで指示無視だもんな
こんな学生をバカにしたレポートを出すほうも出すほうだが
684132人目の素数さん:2007/05/11(金) 16:21:33
>>683
いやあ、こういう風に書かないと学生が勉強しないでしょ。
トップ層の大学ならこんな課題は出さないだろうけど。
685132人目の素数さん:2007/05/11(金) 22:38:20
>>683
再提出にならない書き方はどうすればいいですか?
686132人目の素数さん:2007/05/11(金) 22:58:17
>>685
(゚Д゚)≡゚д゚)、カァー ペッ!!
687132人目の素数さん:2007/05/12(土) 00:03:55
方程式x^2+y^2=61^4の解のうち、x,yがどちらも整数であるような組を全て求めよ。
688132人目の素数さん:2007/05/12(土) 00:51:17
範囲が有限だから、しらみつぶせば終わるな。


まぁ、もっとも試験時間でそんなことはできんだろうが
689132人目の素数さん:2007/05/12(土) 03:15:40
x,y>0の解を調べる
yを偶数とする
x^2=(61^2-y)(61^2+y)
61^2-y, 61^2+y の最大公約数gは
2・61^2の約数でしかも奇数だから、g=1, 61
 g=61のとき、yは61の倍数でxもそうなり、
x=61s, y=61tとして、s^2+t^2=61^2をとけばよい
tは偶数であり、s^2=(61-t)(61+t)
61-tと61+tは互いに素で、ともに平方数になって
t=60となり、このとき、(x,y)=(61・11,61・60)
 g=1のとき、61^2-y,61^2+yは互いに素な平方数で
s^2=61^2-y, t^2=61^2+yとおけば、s^2+t^2=2・61^2
(s^2-1)+(t^2-1)=2^4・3・5・31
これよりs,t≡±1(8,3,5)
s<tとすれば、61<t<61√2で、(s,t)=(49,71)
このとき、x=49・71,y=61^2-49^2
以下略
690132人目の素数さん:2007/05/12(土) 19:54:34
こんなん考えた!!

百人の村があります。
この村の習慣として年賀状を1番近い家の人におくります。
では、1番多く年賀状をもらえる可能性の人は何枚ですか?
ただし、それぞれの家から同じ距離の家はないとする。


691132人目の素数さん:2007/05/12(土) 20:38:25
>>690
救いようがねーな。
ちょっと考えたら気づくだろうが!
まず自分で解いてから、書き込むんだな。

国に帰れ!
692132人目の素数さん:2007/05/12(土) 22:50:47
>>690
わからん(>_<)
693ちょっと変更:2007/05/12(土) 23:02:12
百人の村があります。
この村の習慣として年賀状を1番近い家の人におくります。
では、1番多く年賀状をもらえる人の枚数として可能なのは最大何枚ですか?
ただし、それぞれの家から同じ距離の家はないとする。
694132人目の素数さん:2007/05/12(土) 23:04:57
>>693
おまえ、自分で解いたんだろうな?
695132人目の素数さん:2007/05/12(土) 23:09:53
常識?として同居人には送らないと考えてる
皆一人暮しとしても2軒ずつくっついてれば、みんな1枚しかもらえない
誰かがたくさんもらえるような家の配置を考えて最大は?ってこと
696132人目の素数さん:2007/05/12(土) 23:12:19
>>694
693=695≠690
見落としがなければ解けてる、面白い問題と思ったから勝手に訂正しました
697132人目の素数さん:2007/05/13(日) 00:06:19
算数オリンピックで似たような問題があったような。
ちなみに、小学生が「1軒ものすごく大きな家があって、結局残りの
99軒について直近の家がその大きな家だったら、99通来る」って答えが
あったそうな。
たしかに「家」=「大きさ0の点」と書かれてないしなw
698132人目の素数さん:2007/05/13(日) 00:09:14
>>697
頭柔らかいな、その可能性は気付かんかった
699132人目の素数さん:2007/05/13(日) 00:53:24

>>697
それが答えじゃないのか?
700132人目の素数さん:2007/05/13(日) 01:11:06
(1) 数列0,1,3,6,10・・・・の一般項を求めよ
(2) f(x)=1/〔x*{log(x)}^n〕とおく。f(x)をxで微分せよ。ただしnは定数とする。
(3) ∫1/log(x)dxを求めよ

東大レベルなら(3)だけでもスラスラ解くだろうな
701132人目の素数さん:2007/05/13(日) 01:49:29
>>693
答え 5枚
702132人目の素数さん:2007/05/13(日) 01:52:43
>>700
何を言っているんだ?
703132人目の素数さん:2007/05/13(日) 01:52:43
>>700
釣りなら他所でやれ
704132人目の素数さん:2007/05/13(日) 02:02:07
>>701
693だけど、俺もそのつもりだった
でも99まいってのには負けた・・・
705132人目の素数さん:2007/05/13(日) 02:03:05
sin x + sin 3x + sin 5x の周期を求めよ。
さらに、sin nx + sin (n+2)x + sin (n+4)x の周期を求めよ。
706132人目の素数さん:2007/05/13(日) 02:11:12
関数f(x)がx=0で連続、かつf(0)>0であるとする。このとき、次を満たすc>0が存在することを示せ。
「|x|<cならばf(x)>0」
ただし、f(x)がx=aで連続であるとは、aに収束する任意の実数列{an}に対してlim[n→∞]f(an)=f(a)が
成り立つときをいう。
707132人目の素数さん:2007/05/13(日) 02:15:30
n=3.1
708132人目の素数さん:2007/05/13(日) 02:41:22
>>700の答え

(1)n(n+1)/2
(2)-n/{x*〔log(x)〕^2n}
(3)
∫1/log(x)dx=x/log(x)+∫1/{x*〔log(x)〕^2}dx
        =x/log(x)+1/〔log(x)〕^2+2/〔log(x)〕^4+8∫1/{x*〔log(x)〕^8
        =x/log(x)+1/〔log(x)〕^2+2/〔log(x)〕^4+8/〔log(x)〕^8+64/〔log(x)〕^16+1024/〔log(x)〕^32+・・・・・・・・・・・・
        =x/log(x)+Σ[k=1,∞]2^{k(k+1)/2}/log(x)^2k

>>702,>>703他に(3)の解法あるの?
709132人目の素数さん:2007/05/13(日) 02:48:07
それは解けたとはいわねーよ。
710132人目の素数さん:2007/05/13(日) 03:00:50
独特の計算だな・・・
711132人目の素数さん:2007/05/13(日) 07:30:23
間違いだらけ
712132人目の素数さん:2007/05/13(日) 08:13:19
四面体の任意の三面の面積の和は残りの一面の面積より大きい事を示せ。
713132人目の素数さん:2007/05/13(日) 08:42:22
700の積分を活かしてこんな問題はどう?

lim[t→1+0](log(logt))^(-1)∫[t,e]1/log(x)dxを求めよ
714132人目の素数さん:2007/05/13(日) 09:06:37
>712
 各面を「残りの一面」に(垂直に)投影してみる…
715132人目の素数さん:2007/05/13(日) 09:08:44
>>714
投影して面からはみ出す位置に頂点がある場合は?
716132人目の素数さん:2007/05/13(日) 09:11:25
自明
717132人目の素数さん:2007/05/13(日) 09:15:04
あ、ほんとだ。ごめん。
718132人目の素数さん:2007/05/15(火) 12:25:43
96年頃の大数を立ち読みしてたら、SEGの広告にこんな問題が。

関数f(x)は次の2つの性質を満たす。
  (1) ∫_0^7 f(x) dx   (2) f(x+7)=f(x)
このとき、i=1,2,…,7に対して
  S_i = ∫_k^{k+i} f(x) dx
と定義するとき、
  S_1×S_2×S_3×S_4×S_5×S_6×S_7≧5000
  ( S_1からS_7までの積≧5000 )
となるようなkが存在することを示せ
719132人目の素数さん:2007/05/15(火) 12:41:23
(1)の条件がよく分からん。
720132人目の素数さん:2007/05/15(火) 13:32:41
orz
(1) ∫_0^7 f(x) dx = 7
です。
721KingOfUniverse ◆667la1PjK2 :2007/05/15(火) 14:19:58
nを自然数とする。関数fは∫_{0}^{n}f(x)dx=n, f(x+n)=f(x) を満たすとする。
S_{k,i}=∫_{k}^{k+i}f(x)dxと定義するとき、あるkが存在して
product_{i=1}^{n}S_{k,i}>=n! が成り立つことを示せ。
722132人目の素数さん:2007/05/15(火) 17:55:53
>>718
問題の内容よりも96年の大数を立ち読みできたことに驚きを隠せないのは俺だけでいい。
723132人目の素数さん:2007/05/15(火) 18:29:26
神保町に行けばあるよ。もちろん全号は置いてないが。
724132人目の素数さん:2007/05/15(火) 18:41:39
S_{k,i}

これ何?
725KingOfUniverse ◆667la1PjK2 :2007/05/15(火) 19:10:12
talk:>>724 お前は何を見ている?
726132人目の素数さん:2007/05/16(水) 12:44:35
>>721の二行目
727KingOfUniverse ◆667la1PjK2 :2007/05/16(水) 13:23:25
(m+1)sum_{k=0}^{n-1}product_{l=0}^{m-1}(k-l)=product_{l=0}^{m}(n-l) を証明せよ。
728132人目の素数さん:2007/05/16(水) 13:50:47
talk>>727
LHS=sum_{k=0}^{n-1}((k+1)-(k-m))product_{l=0}^{m-1}(k-l)
=sum_{k=0}^{n-1}(product_{l=0}^{m}(k+1-l)-product_{l=0}^{m-1}(k-l))
=product_{l=0}^m(n-1+1-l)-product_{l=0}^{m-1}(0-l))=RHS
729KingOfUniverse ◆667la1PjK2 :2007/05/16(水) 14:12:24
talk:>>728 やはり式を書きにくいか?
730132人目の素数さん:2007/05/16(水) 23:26:29
プロダクト?
731132人目の素数さん:2007/05/17(木) 00:58:20
誰か>>718教えて
732132人目の素数さん:2007/05/17(木) 01:42:28
Kingに聞け、一般化してるし
733KingOfUniverse ◆667la1PjK2 :2007/05/17(木) 04:37:54
[>>721]については、S_{k,i}>=i となるkが存在することを証明しよう。鳩ノ巣原理の変化形(?)を使うことになる。
734132人目の素数さん:2007/05/18(金) 02:19:53
>S_{k,i}>=i となるkが存在することを証明しよう
各iについて、S_{k,i}>=i を満たすk=k(i)が存在することは 鳩ノ巣原理から
言えるけど、これだとダメで、kがiに依存せずに取れないと意味が無い。どうやるの?
735KingOfUniverse ◆667la1PjK2 :2007/05/18(金) 07:47:34
talk:>>734 それではn=4の場合で考えてみろ。
736132人目の素数さん:2007/05/18(金) 09:03:22
重量のわからない水酸化ナトリウムを水100mlに溶かし
1mol/Lの塩酸を用いて中和したところ25mlで当量点に到達した。
水酸化ナトリウムの重量は何g?
詳しく解説してください
737132人目の素数さん:2007/05/18(金) 09:44:33
2008/2007<k/n<2009/2008
となる整数kが存在しないような正の整数nの個数を求めよ
738132人目の素数さん:2007/05/18(金) 13:40:09
>>737
2009/2008<2008/2007なので、
2009/2008<k/n<2008/2007と脳内補完して回答する。

答え 2015028
739738:2007/05/18(金) 14:52:03
>>738
境界線の処理を間違えました。
正しくは2013021でした。
740738:2007/05/18(金) 15:15:10
>>738 (略解)a=2007とおく。
n>a(a+1)のときは、このようなkは常に存在する。
以下1≦n≦a(a+1)のときを考える。
題意をみたす(n,k)は、座標平面上の領域 
1≦x≦a(a+1) かつ y>(a+2/a+1)x かつ y<(a+1/a)x
にある格子点と一対一に対応する。
この格子点の数を工夫して数えると a(a-1)/2
各xに対してこのような格子点は高々一個しかないので
1≦n≦a(a+1)で与不等式を満たすkが存在するnの個数もa(a-1)/2
だから 求めるnの個数はa(a+1)-a(a-1)/2=a(a+3)/2
(答え) 2017035
741132人目の素数さん:2007/05/18(金) 15:25:41
>>740
お見事
1/2008<(k-n)/n<1/2007だから、
1/2008<y/n<1/2007
という問題に直してしまえば楽
742132人目の素数さん:2007/05/18(金) 17:24:51
数学セミナーからの引用類題
743132人目の素数さん:2007/05/19(土) 10:29:15
優秀な方達が大苦戦してるお
http://school7.2ch.net/test/read.cgi/jsaloon/1175709295/513
744132人目の素数さん:2007/05/19(土) 11:02:55
何番だ
745132人目の素数さん:2007/05/19(土) 11:05:55
513 :名無しなのに合格:2007/05/17(木) 18:41:17 ID:dL6IOJ+2O
http://imepita.jp/20070517/668080
動点Pは点Aを出発し1秒ごとに点にひっついている辺を等確率で選んで隣の点に移動する。
例えばC→Dの確率は2/3である。
n秒後に点Aにいる確率を求めよ。
トリップにしにくいので答を書き込んで下さい。
746132人目の素数さん:2007/05/19(土) 11:17:19
場合分け必死の糞問だろ。
747132人目の素数さん:2007/05/19(土) 11:27:20
答えでてるじゃん
748132人目の素数さん:2007/05/19(土) 12:15:36
>743,745

遷移確率行列P =
 | 0 1 0 0 |
 |1/3 0 1/3 1/3 |
 | 0 1/3 0 2/3 |
 | 0 1/3 2/3 0 |,

 det|xI-P| = (x-1)(x -1/3)(x +2/3)^2,
より、Pの固有値は 1, 1/3, -2/3(重根)
749132人目の素数さん:2007/05/19(土) 20:40:30
>743,745
 固有(縦)ベクトルを横に並べたものを

Q =
 | 1/2, (1/2)√3, 3/√14, 3/√14 |
 | 1/2, (1/6)√3, -2/√14, -2/√14 |
 | 1/2, (-1/6)√3, 1/√14, 0 |
 | 1/2, (-1/6)√3, 0, 1/√14 |

とすると

Q^(-1) =
 | 1/5, 3/5, 3/5, 3/5 |
 | 1/√3, 1/√3, -1/√3, -1/√3 |
 | (1/15)√14, -(2/15)√14, (8/15)√14, -(7/15)√14 |
 | (1/15)√14, -(2/15)√14, -(7/15)√14, (8/15)√14 |

D =
 | 1  0 0 0 |
 | 0 1/3 0 0 |
 | 0 0 -(2/3) 0 |
 | 0 0 0 -(2/3) |

PQ = QD,
P = QDQ^(-1),
P^n = Q(D^n)Q^(-1).
750132人目の素数さん:2007/05/20(日) 14:02:06
>743,745

{n秒間で A→A に遷移する確率} = (P^n)_{1,1}
 = {Q(P^n)Q^(-1)}_{1,1}
 = Σ[i=1,4] Σ[j=1,4] Q_{1,i} (D^n)_{i,j} (Q^(-1))_{j,1}
 = Σ[i=1,4] Q_{1,i} (Q^(-1))_{i,1} (x_i)^n
 = (1/2)(1/5)1^n + {(1/2)√3}(1/√3)(1/3)^n + 2*(3/√14){(1/15)√14}(-2/3)^n
 = (1/10) + (1/2)(1/3)^n + (2/5)(-2/3)^n.
751132人目の素数さん:2007/05/24(木) 12:06:55
誰か>>718解いて
752132人目の素数さん:2007/05/24(木) 14:03:07
書くのが面倒。とりあえず
a_i=∫_i^{i+1} f(x) dx とおいてみる
さらに簡単にするためb_i=a_i-1とすれば
b_{i+7}=b_i, b_0+b_1+....+b_6=0
うまく番号kをとると、b_k,b_{k+1},...,b_{k+6}≧0となる事を言えばよい
この状況で考えてみてくれ
753132人目の素数さん:2007/05/24(木) 14:06:05
うまく番号kをとると、
b_k≧0
b_k+b_{k+1}≧0
...
b_k+b_{k+1}+...+b_{k+6}≧0
をいえばよい、だ
754132人目の素数さん:2007/05/25(金) 00:56:21
(1) ∫_0^7 f(x) dx =7=F(7)-F(0)
(2) f(x+7)=f(x) ->f=sin(2pix/7)+c
F(x)=-cos(2pix/7)(7/2pi)+cx
F(7)-F(0)=-1+c7+1=c7=7->c=1
755132人目の素数さん:2007/05/25(金) 01:05:18
si=F(k+i)-F(k)=(-cos(2pi(k+i)/7)+cos(2pik/7))(7/2pi)+i
756132人目の素数さん:2007/05/25(金) 01:52:25
n次性関数f(x)のx1,x2…x_nにおける法線の傾きの和p1+p2+…p_nをもとめよ
757132人目の素数さん:2007/05/25(金) 02:32:45
n次多項式f(x)があり、f(x)=0はn個の異なる実数解x1,x2…x_nを持つとする
このとき、y=f(x)のx1,x2…x_nにおける法線の傾きの和p1+p2+…p_nを求めよ

折角なんでちゃんと書いたぞ
758132人目の素数さん:2007/05/25(金) 08:17:57
ごめん寝ぼけてたm(__)mありがと!
それ自力で導いたけどオイラーが見つけてた
759132人目の素数さん:2007/05/25(金) 14:59:20
事実を発見したのが凄い
複素解析使えば正しいのは見るりゃわかるんだけどな
760 ◆5Cg9B1.Ywg :2007/05/30(水) 01:48:32

761 ◆BhMath2chk :2007/06/01(金) 00:00:00
a≦xのとき[a,x)での和をg(x)とし
x=bのときgが最小になるとすると
b≦xのとき[b,x)での和は0以上。
762132人目の素数さん:2007/06/03(日) 19:21:33
おーい! 我が家のおりこうさん達が、ここでも糞垂れやがったぞ!
763132人目の素数さん:2007/06/04(月) 00:38:59
>757
 f(x) = a(x-x_1)(x-x_2)…(x-x_k)…(x-x_n),  a≠0
と書ける。 題意より
 1/(-p_i) = f '(x_i) = a(x_i-x_1)……(x_i-x_{i-1})(x_i-x_{i+1})……(x_i-x_n).
n-1次多項式
 g_i(x) = (-p_i)a(x-x_1)……(x-x_{i-1})(x-x_{i+1})……(x-x_n)
とおくと、g_i(x_j) = δ_(i,j)     (クロネッカーのδ記号)

φ(x)を任意の函数とするとき、Σ[i=1,n] g_i(x)φ(x_i) とφ(x)は、n個の点 x=x_1,…,x_n で一致する。(ラグランジュ補間式)
さらに φ(x) が高々n-1次の多項式ならば 両者は恒等的に等しい。

φ(x)=x のとき、Σ[i=1,n] g_i(x)φ(x_i) = x,
x=0 とおいて、 Σ[i=1,n] p_i * (-x_1)(-x_2)…(-x_n) = 0,
(-x_1)(-x_2)…(-x_n)≠0 ならば Σ[i=1,n] p_i =0.

http://mathworld.wolfram.com/LagrangeInterpolatingPolynomial.html
数セミ増刊 「数学の問題 = 第A集」 日本評論社 (1978/05) No.64
764132人目の素数さん:2007/06/04(月) 01:55:28
'`ァ'`ァ
765132人目の素数さん:2007/06/04(月) 02:54:41
各頂点の角が等しいn角形が円に内接しているとする
(1)nが奇数であるとき正n角形であることを示せ
(2)nが偶数であるとき必ずしも正n角形でないことを反例をあげて示せ
766132人目の素数さん:2007/06/04(月) 03:14:42
(2)は底上げ厨房級として、(1)は見かけによらずへビィ?
767765:2007/06/04(月) 04:50:07
>>766
ヘビっちゃあヘビだが文系でも解けるように作ってるから
768132人目の素数さん:2007/06/04(月) 07:38:41
パクリを自作と言うなカス
769765:2007/06/04(月) 09:29:44
>>768
なんだと!
パクリというなら、元ネタを上げてみろ!
鉄入り作業靴で蹴り殺されたいのか?
770132人目の素数さん:2007/06/04(月) 21:13:51
>>765
入試のレベルには相応の問題ですね。
771132人目の素数さん:2007/06/05(火) 03:13:30
>>523

x=2cos[1/2]tとおいて
ほにゃららか
772132人目の素数さん:2007/06/05(火) 03:14:47
ミス

x=2cos[t]だった
773132人目の素数さん:2007/06/05(火) 06:13:09
>>765
(1)正n角形の状態から1つの角を動かし、
隣接する角をその角度に合わせると
隣接する角のその次の角のいずれかが
正n角形のときよりも大きくならなければならないから
でおk?
774132人目の素数さん:2007/06/05(火) 08:28:55
>>773
反対の角のいずれかのまちがいだた
775132人目の素数さん:2007/06/05(火) 18:29:38
>>765
(1) 頂点を一つおきに結ぶと星型 n 角形ができる。
辺の長さがすべて等しいので...
という感じかな?
776132人目の素数さん:2007/06/05(火) 18:38:47
原始的に考えれば楽勝
777132人目の素数さん:2007/06/05(火) 18:41:34
そんなことわざわざ書き込む必要があるのか。
778132人目の素数さん:2007/06/05(火) 18:50:21
すんません
779132人目の素数さん:2007/06/05(火) 18:58:50
解法をpliz
780132人目の素数さん:2007/06/05(火) 19:01:16
>>779
だから、原始的にやれば簡単だって、書いたら面白くない
781132人目の素数さん:2007/06/05(火) 19:04:32
n=5ぐらいでとりあえず考えてみろ。
782ヒント:2007/06/05(火) 19:10:26
           ________
         /|\          /|\
       /  |  \       /  |  \
      /   |   \    /   |   \
    /     |     \ /     |     \
   /      |     / \     |      \
 /_____|__/    \__|_____\
 |\       |   |\   /|   |        /|
 |  \      |__|_\/_|__|      /  |
 |   \   /\  |  /\  |  /\   /    |
 |     \/   \|/   \|/   \/     |
 |     /\   /|\   /|\   /\     |
 |   /   \/  |  \/  |  \/   \    |
 |  /      | ̄ ̄| ̄/\ ̄| ̄ ̄|      \  |
 |/       |   |/   \|   |        \|
 \ ̄ ̄ ̄ ̄ ̄| ̄ ̄\    / ̄ ̄| ̄ ̄ ̄ ̄ ̄/
   \      |     \ /     |      /
    \     |     / \     |     /
      \   |   /    \   |   /
       \  |  /       \  |  /
         \|/          \|/
            ̄ ̄ ̄ ̄ ̄ ̄ ̄ ̄

4次元(超立方体)
783132人目の素数さん:2007/06/05(火) 21:07:10
10cmx10cmの正方形からランダムウオークで外に出る確率をステップ数で
あらわせばどうなるか。
784132人目の素数さん:2007/06/05(火) 21:09:10
中心から出発で1歩1センチ、動きは辺に平行ってことでおK?
785132人目の素数さん:2007/06/05(火) 21:11:59
ステップ数で表せが意味不明。
786132人目の素数さん:2007/06/05(火) 21:37:38
おk、ステップ数は10歩目に外に出るとかってことで。
787132人目の素数さん:2007/06/05(火) 21:59:05
n歩目で外に出る確率P(n)をnで表せってことね。
788132人目の素数さん:2007/06/05(火) 23:11:38
ところで773でいいんだよな
789132人目の素数さん:2007/06/05(火) 23:39:45
だめ。
790132人目の素数さん:2007/06/06(水) 00:03:29
>765
(1)自明(2)長方形
791790:2007/06/06(水) 00:14:14
>773
うんうん言いたいことは痛いほどわかるよ
792132人目の素数さん:2007/06/06(水) 10:59:49
765のは面白いとは思うが、高校入試向けだと思う。
793132人目の素数さん:2007/06/06(水) 12:22:19
円周角の定理とその逆は数学 A に移ってしまっているので、
高校の入試問題にはできない。
794132人目の素数さん:2007/06/06(水) 12:30:11
(2)のほうが面白いな
795132人目の素数さん:2007/06/07(木) 06:14:55
Sai=2pi
(pi-ai)/2+(pi-ai+1)/2=(pi-ai+1)/2+(pi-ai+2)/2
ai=ai+2
n=2m+1=odd->a1=a3=...=a2m+1=a2=a4=...=a2m qed
796132人目の素数さん:2007/06/07(木) 06:41:35
(e+e^-1+e^i+e^-i)^n
e^p+qi 5>p,q>-5
797132人目の素数さん:2007/06/07(木) 11:56:59
3辺の長さがn n+1 n+2 n>=3となる三角形の面積が整数になる三角形の面積の集合をS
とする。
(1)Sのうちで三番目に小さいものの値を求めよ。
(2)Sの3辺のうち2番目に長い辺の長さを、小さいものから順に並べた場合
数列になることを証明せよ。また、数列の一般項を求めよ。
798KingOfUniverse ◆667la1PjK2 :2007/06/07(木) 12:34:26
talk:>>797 日本語を書け。
799132人目の素数さん:2007/06/07(木) 12:34:54
微妙に日本語が面白い
800132人目の素数さん:2007/06/07(木) 13:43:20
(1)8(2)?
801132人目の素数さん:2007/06/07(木) 18:34:33
>>797
(2)は順に並べてるんだから明らかに数列
802132人目の素数さん:2007/06/07(木) 18:42:01
いやいや日本語おかしいから。
803132人目の素数さん:2007/06/07(木) 19:11:52
ху平面上に2つの円C:х^2+у^2=2,Ca:(х-a)^2+(у+3)^2=1(aは定数)と、2円の外部の点PがありPからCに引かれた接線とCとの接点をQ,PからCaへ引いた接線とCaとの接点をRとする。
(1)Pの座標を(Х,Υ)とするとき、線分PQの長さをХ,Υを用いて表せ。
(2)点PがPQ=PRをみたしながら移動するとき、Pはある直線la 上にある。laの方程式を求めよ。
(3)у軸の定点(0,κ)(ただしκ≧0)と(2)で求めたlaとの距離をhとおくとaを変化させるときhの最小値を求めよ
804132人目の素数さん:2007/06/07(木) 19:13:06
計算が面倒なだけの問題。
805132人目の素数さん:2007/06/07(木) 19:14:25
>>804
東大にはありがち
806132人目の素数さん:2007/06/07(木) 19:20:17
>803
高校生のための数学の質問スレ
とマルチ
807132人目の素数さん:2007/06/07(木) 19:37:00
だなでも向こうのスレで誰も相手にしてないから解決してやろうぜ(;-_-)=3 フゥ
808132人目の素数さん:2007/06/07(木) 19:52:52
>>.807
お前がしてやれ
809132人目の素数さん:2007/06/07(木) 19:59:27
マルチには解答しない。
810132人目の素数さん:2007/06/08(金) 05:30:19 BE:303858634-2BP(1000)
こんなの簡単すぎて解答するやついないだろ
811132人目の素数さん:2007/06/09(土) 19:00:39
自然数a,bを用いてp=a^2+5b^2と表すことのできる最小の素数pを求めよ
812132人目の素数さん:2007/06/09(土) 19:13:31
>>811
29 

20で割って余りが1または9の素数がその形に表せるらしい。
813132人目の素数さん:2007/06/09(土) 20:24:21
>>811
つまらん
814132人目の素数さん:2007/06/09(土) 20:57:09
>>812
不正解
>>813
815132人目の素数さん:2007/06/09(土) 21:19:24
不正解って・・・
816132人目の素数さん:2007/06/09(土) 22:08:28
正解だろ。
817132人目の素数さん:2007/06/09(土) 22:14:59
自然数a,bを用いてp=a^2+5b^2と表すことのできる最大の素数pを求めよ

818132人目の素数さん:2007/06/09(土) 22:28:04
不正解も何もa=b=1のときa^2+5b^2=26だから自明だろ
てか
5*(b^2) (5b)^2
どっちなんだ?
819132人目の素数さん:2007/06/09(土) 22:37:02
>>818
5b^2はどう見ても5*(b^2)
820132人目の素数さん:2007/06/10(日) 01:17:10
3桁の非正則素数の総和を求めよ。
821jh:2007/06/10(日) 01:21:44
ソフトバンク主催NCJオンラインゲーム「リネージュ」の詐欺について
1キャンペーンと唄いネカフェに呼んで切断・もしくわPC容量によってEND
2上記についてネカフェPCのスペックを確認せずイベント時END
3上記2についてネカフェ指導したと嘘のメールをよこす
4公認ネカフェ公認PCと唄っていながらスペックが低スペックの店があり、そのPCの公認シールによって騙している(詐欺)
5もうすでに、初心者がこれからやってもレアアイテム何も取れない糞ゲームになっている。なぜなら、仕様変更、高LV対象のイベントしかやらず、さらにエピソードという段階ストーリーでさらに6/24から貧富の差がはげしくなる。無論高LVの人しかレア取れないでしょう。
6NCJ(ソフトバンク)には行政監査はいるべきでしょう。(詐欺行為してますから)
ソフトバンク主催NCJオンラインゲーム「リネージュ」の詐欺について
1キャンペーンと唄いネカフェに呼んで切断・もしくわPC容量によってEND
2上記についてネカフェPCのスペックを確認せずイベント時END
3上記2についてネカフェ指導したと嘘のメールをよこす
4公認ネカフェ公認PCと唄っていながらスペックが低スペックの店があり、そのPCの公認シールによって騙している(詐欺)
5もうすでに、初心者がこれからやってもレアアイテム何も取れない糞ゲームになっている。なぜなら、仕様変更、高LV対象のイベントしかやらず、さらにエピソードという段階ストーリーでさらに6/24から貧富の差がはげしくなる。無論高LVの人しかレア取れないでしょう。
6NCJ(ソフトバンク)には行政監査はいるべきでしょう。(詐欺行為してますから)
822132人目の素数さん:2007/06/10(日) 06:26:12
素数=1とそれ以外では割り切れない自然数
Σp^-2=aπ/6
823132人目の素数さん:2007/06/10(日) 06:35:45
Σp^-1=Σ2^-n<2
824132人目の素数さん:2007/06/10(日) 21:17:34
{2n/(2n-1)}^n≦2を示せ
825132人目の素数さん:2007/06/10(日) 22:11:58
(x+y)の三乗を解ける人はいるか?
826132人目の素数さん:2007/06/10(日) 22:23:28
>>825
方程式じゃないので解けない
827132人目の素数さん:2007/06/10(日) 22:48:28
展開ならできるよ
828132人目の素数さん:2007/06/10(日) 23:24:13
X^3―6X―1=0を解け。
829132人目の素数さん:2007/06/10(日) 23:30:12
解いた。
830132人目の素数さん:2007/06/10(日) 23:46:17
x=u+v
x^3=u^3+v^3+3uv(u+v)
uv=2,u^3v^3=8
u^3+v^3=1
(u^3-v^3)^2=1-4*8=-31
u^3-v^3=√31*i
u^3=(√31*i+1)/2
v^3=(√31*i-1)/2
1<x=u+v={(√31*i+1)/2}^(1/3)+{(√31*i-1)/2}^(1/3)<2
831132人目の素数さん:2007/06/10(日) 23:55:06
ちがくね?
832132人目の素数さん:2007/06/11(月) 23:28:36
ごめんあってた
833132人目の素数さん:2007/06/12(火) 02:47:48
【審議中】
    ∧,,∧  ∧,,∧
 ∧ (´・ω・) (・ω・`) ∧∧
( ´・ω) U) ( つと ノ(ω・` )
| U (  ´・) (・`  ) と ノ
 u-u (l    ) (   ノu-u
     `u-u'. `u-u'
834132人目の素数さん:2007/06/12(火) 03:35:54
【審議中】
    ∧,,∧   ∧,,∧
 ∧ < `∀´ ><`∀´ >∧∧
< `∀>  U) ( つと ノ( ∀´ >
| U <   `><´   > と ノ
 u-u (l    ) (   ノu-u
     `u-u'. `u-u'

【判決】:kingとのガチ対談を命ずる
    ∧,,∧   ∧,,∧
 ∧ < `∀´ > <`∀´>∧,,∧
<`∀´>  U) ( つと ノ<`∀´>
| U <`∀´> <`∀´>  と ノ
 u-u (l    ) (   ノu-u
     `u-u'. `u-u'
835132人目の素数さん:2007/06/12(火) 22:49:41
      ∩___∩    
      | ノ  _,  ,_ ヽ
     /  ●   ● |
     |    ( _●_)  ミ 
    彡、   |∪|  ノ
⊂⌒ヽ /    ヽノ  ヽ /⌒つ
  \ ヽ  /         ヽ /
   \_,,ノ      |、_ノ
836132人目の素数さん:2007/06/12(火) 22:50:50
>824
0≦k<2n のとき
{(2n-1)/2n}^k ≧ 1-(k/2n),
(kについての帰納法による)。
よって
 {2n/(2n-1)}^k ≦ 2n/(2n-k).
837132人目の素数さん:2007/06/13(水) 00:35:13
{2n/(2n-1)}^n
(1-1/2n)^-2n/2
(1-1/m)^m/-2->e^-1/-2=e^1/2<2
838132人目の素数さん:2007/06/13(水) 01:20:59
{2n/(2n-1)}^n
={1+1/(2n-1)}^n < {1+1/2(n-1)}^n
lim[n→∞][{1+1/2(n-1)}^n] = e^(1/2)<2
n=1のとき、{2n/(2n-1)}^n = 2
∴{2n/(2n-1)}^n≦2
839132人目の素数さん:2007/06/13(水) 16:42:00
納k=1,n]sin(100゚*k)=0
となるような最小の自然数nを求めよ
840132人目の素数さん:2007/06/13(水) 16:51:06
なめてんのか。
841132人目の素数さん:2007/06/13(水) 16:59:22
>>839
         ◤◥◣  コーヒー噴いた
  ▂  ◢◤▀〓▲▂▐         ▂ ▪ ▂▄▅▆▇■▀▀〓◣▬ ▪ ■ … .
 ▍ ▼     ◥◣▼        .▂▅■▀ ▪ ■ ▂¨ ∵▃ ▪ ・
  ▀▍ ◢◤     ▅ ▐◣   ◢◤ ◢▇█▀ ¨▂▄▅▆▇██■■〓◥◣▄▂
    ▍  ▅ ◢■     ▍ ■ ▂▅██▅▆▇██■〓▀▀ ◥◣ ∴ ▪ .
  ▐   ▂   ▐◣ ▐▅▇███████▀ ▪ ∴ ….▅ ■  ◥◣
   ▀◣▂  ▀◥▅▆▇████████▆▃▂  ▪ ■▂▄▃▄▂
     ◥◣▄▂▄▅▀   ■  ¨ ▀▀▀■▀▀▀ ▪ ■ ∴‥
842132人目の素数さん:2007/06/13(水) 21:17:08
正方形ABCDの辺ADの中点を点Mとし、直線MC上で正方形の外側に点Pを
∠PBC=15゚となるようにとる。
(1)線分BMと線分BPの長さを比較せよ。
(2)∠BMCは50゚より大きいか小さいか判定せよ。
843132人目の素数さん:2007/06/13(水) 22:00:21
BC4580年3月10日は何曜日?
844132人目の素数さん:2007/06/13(水) 22:29:56
覇^ika=0i+p
(e^ia(k+1)-1)/(e^ia-1)=0i+p
e^ia(k+1)-1=p(e^ia-1)
ia(k+1)=log(p(e^ia-1)+1)
k=(log(p(e^ia-1)+1))/ia-1
a=100,p=any
845132人目の素数さん:2007/06/13(水) 22:46:11
>839

(与式) = Σ[k=1,n] {cos(100゚*k -50゚) - cos(100゚*k +50゚)} / {2sin(50゚)}
    = {cos(50゚) - cos(100゚*n +50゚) } / {2sin(50゚)}
    = sin(50゚*(n+1))・sin(50゚*n)/sin(50゚),
n=17.
846132人目の素数さん:2007/06/13(水) 23:06:22
e^ia(k+1)-1=p(e^ia-1)
cosa(k+1)-1=pcosa-p
sina(k+1)=psina
p^2sina^2+(pcosa-p+1)^2=1
p^2+2p(cosa-1)+1=1
p=0,2(1-cosa)
sina(k+1)=2(1-cosa)sina

847132人目の素数さん:2007/06/13(水) 23:22:00
(p(cosa-1)+1)^2
p^2cosa^2-2pcosa+p+2p(cosa-1)+1
p^2cosa^2-p+1
p^2-p+1=1
p=0,1
p=0
a(k+1)=90+180m
k=90(1+2m)/a-1
p=1
a(k+1)=180-a+360k
k=(180-2a+360m)
848132人目の素数さん:2007/06/13(水) 23:49:11
p=0
k=(180-2a+360m)/a
a(k+1)=180m
ak=180m-a
k=180m/100-1
=9-1=8
p=1
a(k+1)=180-a+360m->k=(-20+360m)/100=7
a(k+1)=a+360m->k=360m/100=18
k=7
849132人目の素数さん:2007/06/15(金) 20:20:05
一辺の長さが1の正n角形の面積をSnとして、

lim Sn/(n^2) を求めよ。
n→∞
850132人目の素数さん:2007/06/15(金) 20:35:38
>>849

Sn=n*{cot(pi/n)} /4
lim Sn/(n^2)= lim {cos(pi/n)}*pi /4*pi*n*sin(pi/n)=1/4*pi
n→∞ n→∞

脊髄レスなんで間違ってるかも
851132人目の素数さん:2007/06/15(金) 20:53:59
>>850
1/(4pi)のことだよね。正解。
852132人目の素数さん:2007/06/15(金) 23:50:11
>>842
(1)
正方形の一辺の長さを1、∠AMB=θとおくと
∠BPC=θ−15 ∠BCP=180−θ
BPの長さをxとおくと、
sinθ=2/√5 、cosθ=1/√5であるから、
 x/sin∠BCP = 1/sin∠BPC
⇔x=sin(180−θ)/sin(θ−15)
=sinθ/(sinθcos15−cosθsin15)
=(6√2 - 2√6)/3
これはBM=√5 /2よりも大きいからBP>BM(答)

※(1)の(6√2 - 2√6)/3と√5 /2の大小の説明は省略。
853132人目の素数さん:2007/06/15(金) 23:56:12
(2)(1)より、
 180−2θ<θ−15
⇔θ<65
∠BMC=180−2θ>180−2*65=50
よって∠BMCは50°より大きい。(答)
854132人目の素数さん:2007/06/16(土) 09:15:38
>>849
出そうな問題なので、受験生にも分かるver
三角形の面積を考えることで、
Sn=1/{4sin(π/n)tan(π/n)}
よって、
Sn/n~2=(1/4)*{(π/n)/sin(π/n)}*{(π/n)/tan(π/n)}*(1/π~2)
    →1/(4π~2) (n→∞)
855849:2007/06/16(土) 19:04:08
>>854
何故Snがそう書けるんだ?
856132人目の素数さん:2007/06/16(土) 21:32:38
nを3以上の自然数とする。X + Y + Z = nをみたす自然数(X,Y,Z)の組はC[n-1,2]通り存在する(←二項係数ね)。このC[n-1,2]個の組それぞれについてXYZおよびXY+YZ+ZXを考え、その平均値をそれぞれ求めよ。
857132人目の素数さん:2007/06/17(日) 00:41:39
>>856
総和を求めれば平均値が求まるので、総和だけを考える。XYZに関してはこの場合、「自然数」ではなく「非負整数」としてもかわりがない。
まず横に1つ縦にn個動く方法はn+1通りあることは自明。これを用いて、横に5個、縦にn‐3個動く方法を考えてみよう。
横に2回目に動くまでに縦にX‐1個、横に4回目に動くまでに縦に更にY‐1個動いていたとする。
前述の自明から、第一部で縦にX‐1個よりX通り、第二部で縦にY‐1個よりY通り、最終部で縦に(n‐3)‐(X‐1+Y−1)、つまりZ‐1個動くのでZ通りの動き方があるので、全部でXYZ通りとなる。
X、Y、Zがありえるすべての自然数を動けばすなわち、横に5個、縦にn‐3個動く方法すべてを網羅したことになるので、その場合の数を求めればよく、これはC[n+2,5]通り。
よってXYZの総和もC[n+2,5]となる。
858132人目の素数さん:2007/06/17(日) 00:44:13
次にXY+YZ+ZXを考える。最初に言っておいたように、非負整数でもXYZの総和は変わらないので
和がnである自然数X,Y,Zに対して、(X-1)(Y-1)(Z-1)の総和は、和がn-3であるときの自然数X,Y,Zの積の総和と一致する。
つまり和がnである自然数X,Y,ZでXYZ-(XY+YZ+ZX)+(X+Y+Z-1)の総和はC[n-1,5]となる。n-1が1以上5以下でも成り立つことに注意してほしい。
XY+YZ+ZXの総和をSとおけば、X+Y+Z-1=n-1で全部でC[n-1,2]通りあるので、以下の式が成り立つ。
C[n-1,5]=C[n+2,5]-S+(n-1)C[n-1,2]
これを整理すればS=3*C[n+1,4]が求まる。
あとは求めた2つの値をC[n-1,2]で割ればよい。
859856 ◆wqlZAUTQF. :2007/06/17(日) 02:40:49
>>857-858
正解です。
ちなみに私が予備校の東大対策模試で出題した問題です。
860132人目の素数さん:2007/06/17(日) 08:00:28
東進?
861132人目の素数さん:2007/06/17(日) 08:07:31
x+y+z=n
AVxyz=xy(n-x-y)=nAVxy-AVxy(x+y)
AVxy+yz+zx=xy+(y+x)(n-(x+y))=AVxy+nAV(x+y)-AV(x+y)^2
AV(x+y)=AVx+AVy=n(n+1)/n=(n+1)
AVxy=AVyAVx=AVy(y+1)=AVy^2+(n+1)
...
862132人目の素数さん:2007/06/17(日) 08:46:08
<xyz>
<xy(n-(x+y))>
n<xy>-2<x^2y>

<xy>
<<x(m-x)>>
<m<x>-<x^2>>

<x^2y>
<<x^2(m-x)>>
<m<x^2>-<x^3>>

n<xy>-2<x^2y>
n<m<x>-<x^2>>-2<m<x^2>-<x^3>>
863132人目の素数さん:2007/06/17(日) 08:50:09
<xy+yz+zx>
<xy+(n-(x+y))(x+y)>
<xy>+n<x+y>-<(x+y)^2>
<xy>+2n<x>-2<x^2>-2<xy>
864132人目の素数さん:2007/06/17(日) 08:56:33
>>857
母関数を考えて、{x/(1-x)^2}^3 の係数を求めるのが機械的で簡単。
>>868
こちらは、3{x/(1-x)^2}^2(1/(1-x)) の係数。
865132人目の素数さん:2007/06/17(日) 09:00:27
<xy+yz+zx>
3<xy>
3*(2n^2+3n)/36
866132人目の素数さん:2007/06/17(日) 09:04:07
<xyz>
n<xy>-2<x^2y>
n(2n^2+3n)/12-2<x^2y>
867132人目の素数さん:2007/06/17(日) 09:05:00
nを3以上の自然数とする。X + Y + Z = nをみたす自然数(X,Y,Z)の組はC[n-1,2]通り存在する(←二項係数ね)。このC[n-1,2]個の組それぞれについてX^aY^bZ^cおよびX^aY^b+Y^bZ^c+Z^cX^aを考え、その平均値をそれぞれ求めよ。
868132人目の素数さん:2007/06/17(日) 09:15:30
<m<x^2>-<x^3>>
<mm(m+1)/2(m+1)-<x^3>>
<m^2/2-<x^3>>
n(2-1)/12-<<x^3>>
869132人目の素数さん:2007/06/17(日) 09:17:29
n(2n-1)/12-<<x^3>>
870856 ◆wqlZAUTQF. :2007/06/17(日) 11:17:28
任意のnについて以下の不等式が成り立つことを示せ。必要ならば自然対数の底の値e=2.718…を用いてもかまわない。

10Σ(1/k)(1/2)^k<7
(Σのkは1〜n)

※実際の模試では誘導二題つけてたけどここではいらないよね
871132人目の素数さん:2007/06/17(日) 11:30:35
1/2+1/8+1/24+1/4*1/8=67/96=0.697...
872132人目の素数さん:2007/06/17(日) 12:37:08
nを自然数とする。(0.5)^nは小数第何位まで数が並ぶか。nを用いて表せ。
873132人目の素数さん:2007/06/17(日) 12:44:38
>>872
4項目は1/4*1/16だべ

>>873
nじゃないの?
874872:2007/06/17(日) 12:51:26
>>873
答えはnだけど、むしろ証明をしてほしい。簡単かな…。
875132人目の素数さん:2007/06/17(日) 13:04:17
(0.5)^n=5^n*(1/10)^n
∴小数第n位まで数が並ぶ。

たぶん
「小数第1〜m位まで0が並ぶとき、mをnで表せ。」
とか加えたら面白くなるかも
876132人目の素数さん:2007/06/17(日) 13:06:15
>>873
>>872は上から評価した式の値ということでは?
877132人目の素数さん:2007/06/17(日) 20:51:49
http://blog.livedoor.jp/t_yonetani/
このホームページどう思いますか?
ここにレアな過去問題が!
http://image.blog.livedoor.jp/t_yonetani/imgs/c/2/c22ee8ec.gif
878132人目の素数さん:2007/06/17(日) 21:18:20
3つの角が,30゚,60゚,90゚である直角三角形の面積が14で
ある時,内接する正三角形の面積の最小値を求めよ.
879132人目の素数さん:2007/06/17(日) 21:23:29
>>864
少し解説していただけるとありがたいんですけど、、

母関数を1−xで割ると数列の和になるんでしたっけ?
880132人目の素数さん:2007/06/17(日) 21:35:02
>>878
東大にしては簡単すぎるんじゃないか
881132人目の素数さん:2007/06/17(日) 23:00:09
一次試験だよ
882132人目の素数さん:2007/06/17(日) 23:17:44
>>879
> 母関数を1−xで割ると数列の和になるんでしたっけ?
そう。
f(x)=a_0+a_1x+a_2x^2+...
s_n=a_0+a_1+...+a_n のとき、
f(x)/(1-x)=s_0+s_1x+s_2x^2+...

1/(1-x)^k= C[k-1,0]+C[k,1]x+C[k+1,2]x^2+...
883132人目の素数さん:2007/06/18(月) 00:54:09
>>878
確か東京工業大学の問題の誘導外したやつね
884132人目の素数さん:2007/06/18(月) 15:57:00
>>870
最終的にlog2<0.7って不等式が残った。これをe=2.718…を使って…面倒くせ
885132人目の素数さん:2007/06/18(月) 17:30:13
>871 で証明できてるでしょ。
886132人目の素数さん:2007/06/18(月) 19:05:31
先生はA君とB君に次のように言った。
「1以上13以下の自然数x,y(x≦y)を選び、A君にはxとyの積を、B君にはxとyの和を教えます。」
そして積、和をそれぞれ教えた。

 教えられた積を見てA君は言った。
A「この情報だけではx,yは特定できません。」…発言(ア)

 教えられた和を見てB君は言った。
B「この情報だけではx,yは特定できません。ただ、A君がx,yを絶対に特定できない、ということは分かります。」…発言(イ)

 発言(イ)を聞いたA君は言った。
A「それならx,yは特定できました。」…発言(ウ)

 発言(ウ)を聞いたB君は言った。
B「それならx,yは特定できました。」

以上の会話からx,yを特定せよ。
887132人目の素数さん:2007/06/18(月) 20:23:28
<m<x^2>-<x^3>>
<mm(m+1)(2m-1)/6(m+1)-<x^3>>
<m^3>/3-<m^2>/6-<<x^3>>
<<x^3>>=<(m^4+2m^3+m^2)/4(m+1)>
<m^3+m^2>/4
<m^3>/12-<m^2>5/12
(n^3+n^2)/48-n(2n-1)5/72


888132人目の素数さん:2007/06/18(月) 20:46:47
>>887
何をやっているのだ?
889132人目の素数さん:2007/06/18(月) 20:59:12
<x^2y>=(ΣΣx^2y)/(m+1)(n+1),x+y=m,m=0->n
890132人目の素数さん:2007/06/18(月) 22:38:25
>>886
算数オリンピックじゃないんだからさw

x=1 y=4
891132人目の素数さん:2007/06/18(月) 22:46:18
nを整数として、
5^nの最高位の数が8となる最小のnを求めよ。
log10底の2=0.3010
log10底の3=0.4771とする。
892132人目の素数さん:2007/06/18(月) 22:59:30
>>891
だから宿題貼るな
893132人目の素数さん:2007/06/19(火) 00:34:47
wx=yzを解け。
894132人目の素数さん:2007/06/19(火) 00:38:18
だが断る!
895132人目の素数さん:2007/06/19(火) 01:04:16
半径1の円Cの周上にどのように点P,Q,R,Sをとっても、点XをCの周上に適当にとることによってPX*QX*RX*SX≧をみたすようにできる。kの値の最大値を求めよ
896895:2007/06/19(火) 01:05:49
すまん>895だが
PX*QX*RX*SX≧k
だ。
897132人目の素数さん:2007/06/19(火) 06:55:46
Σxy=ΣΣx(m-x)
Σm^2(m-1)/2-(m-1)m(2m-1)/6
Σ3m^3-3m^2-2m^3+3m^2-m/6
Σ(m^3-m)/6
(n^4+2n^3+n^2-2n(n+1))/24
(n^4+2n^3-n^2-2n)/24
3Σxy/Σ(m-1)
<xy+yz+zx>=(n^4+2n^3-n^2-2n)/4(n-2)(n-1)
(n^2+3n+2)n/4(n-2)
<xyz>
<x(m-x)(n-m)>/m(m-1)/2
<(n-m)(mx-x^2)>/((n-1)(n-2))/2)
(n-m)(m^3-m)/6
n(m^3-m)/6-(m^4-m^2)/6,m=2->n-1
{15n((n-1)^3+2(n-1)^2-(n-1)-2)-2(6(n-1)^4+15(n-1)^3+10(n-1)^2-1)
+10(2(n-1)^2+3(n-1)+1)}/180(n-2)




898132人目の素数さん:2007/06/19(火) 10:20:07
>>897
ここを計算用紙に使うな
899132人目の素数さん:2007/06/19(火) 17:48:38
900132人目の素数さん:2007/06/19(火) 22:40:41
>>898
構って欲しいんだろw

オリジナルじゃなくて、どこかの予備校が出していた対策本にあった問題。
図が描けないので頂点などの記号を多用して説明。

正三角形ABCにおいて、Aから対辺BCに引いた垂線を軸に持つ放物線を、
ちょうど点B, 点Cで辺AB, 辺ACに接するように引く。
これを残る2頂点についても同様に引く。

全部で3本の放物線によって囲まれた部分の面積と、正三角形の面積との比を求めよ。
901132人目の素数さん:2007/06/20(水) 00:26:54
>>900
ちょっと易しい気もするけどなかなか良問だね
902132人目の素数さん:2007/06/20(水) 00:43:10
>>900
5/27かな?積分の計算は有名なあの公式を使うだけで
ほとんど初等幾何的に解ける点で良問って感じか。
903132人目の素数さん:2007/06/20(水) 03:53:34
aは0でない実数とする
関数f(x)=(3x^2-4)(x-a +1/a)の極大と極小の値の差が最小となるaを求めよ
904132人目の素数さん:2007/06/20(水) 04:08:37
98年1番か。
905132人目の素数さん:2007/06/20(水) 04:12:08
いつから、こぴぺスレに成り下がったんだい?
906132人目の素数さん:2007/06/20(水) 04:25:27
1からnまでの番号のついたカード、箱がありカード1枚につき一つの箱に入れる
カードの番号と箱の番号が一組も一致しない確率をp_nとおくとき
lim[n→∞]p_nを求めよ
907132人目の素数さん:2007/06/20(水) 04:26:48
またかよ
(゚Д゚)≡゚д゚)、カァー ペッ!! >>906
908132人目の素数さん:2007/06/20(水) 08:47:26
簡単目な問題を。前期の二問目くらい?答え自体より論証重視な感じで。

nを4以上の整数の定数とする。X[i] (1≦i≦n+3)は正の実数で、1≦k≦nを満たす整数kに対して
X[k]^3=X[k+1]*X[k+2]+6*X[k+3]
X[n+1]=X[1] X[n+2]=X[2] X[n+3]=X[3]
を満たす。このとき以下の問いに答えよ。
(1)X[1]〜X[n]の条件の対称性から、X[1]がこのn個の正の実数の中で最大のものの1つであるとしても一般性を失わない。
この条件の下でX[1]^3=X[2]*X[3]+6*X[4] を用いてX[1]の取りうる値の範囲を求めよ。
(2)X[i] (1≦i≦n)の値を求めよ。
909132人目の素数さん:2007/06/20(水) 20:17:05
一辺の長さが1の立方体がある。また、等辺の長さが1の直角二等辺三角形のシールが12枚あり、すべて異なる色である。
立方体のすべての辺とシールのすべての等辺が一致するようにシールを貼るとき(それぞれの面にふたつずつ貼る)、貼り方は何通りあるか。ただし、回転させて一致するものは同じものとする。


ちょっと簡単かもしんない
910132人目の素数さん:2007/06/21(木) 01:05:58
こういう問題好きじゃねえや。東大っぽくもないし。
911132人目の素数さん:2007/06/22(金) 08:16:07
問1
6点 P1=(a,b,c),P2=(b,c,a),P3=(c,b,,a)P4=(a,c,b),P5=(c,b,a),P6=(b,a,c)
は同一平面上にあることを示せ。

問2 上の6点が作る六角形の面積を求めよ。


うーん、もう少しいい問題の出来そうな題材な気がするので
解けたら改題を希望します。
912132人目の素数さん:2007/06/22(金) 10:07:19
>>911
問1で平面の式がきれいな形なのだから
問2は原点と結んだ六角すいの体積のほうがよい気が。
どっかの大学が既に出してそうな問題だが。
913132人目の素数さん:2007/06/22(金) 12:53:41
>>911
いい問題だと思う。解きたくなる問題。『a+b+c=1』などの制限を加えて面積の最大値・最小値を求めさせるとか。
914132人目の素数さん:2007/06/22(金) 18:50:05
xyz座標空間に原点Oを重心にもつ正四面体があり,4頂点のx座標はそれぞれ1,3,5,7である。
 このとき,この正四面体の一辺の長さを求めよ。
915132人目の素数さん:2007/06/22(金) 19:17:08
>>914
ちゃんと解いてから出せや
数値適当に書いた文章さらすんじゃなくて
916914:2007/06/22(金) 19:38:40
>>915
その数値なら綺麗になる。煩雑な数値になるんなら解き方がまずいか遠回りしてることになるよ
917915:2007/06/22(金) 20:02:14
>>914
喧嘩腰の言い方になってすみません
この条件だと正四面対の重心のx座標が原点に来ない気がして。
918132人目の素数さん:2007/06/22(金) 20:12:59
俺もそう思う
-1,3,5,-7
の間違いじゃなくて?
919914:2007/06/22(金) 20:29:25
>>917
>>918
いや、平行移動すりゃすむ話なんだけど…
920132人目の素数さん:2007/06/22(金) 20:36:29
>>919
重心動かさずに平行移動は無理でしょ
921915:2007/06/22(金) 20:37:58
>>920
重心ごとだってばw
922132人目の素数さん:2007/06/22(金) 20:42:19
>>920
なまえの数字も移動してるよw
原点云々を無視しても一つに定まらないよね
ありえる範囲を求めるの?
923914:2007/06/22(金) 20:45:47
ああすまん、俺は914だった。
全頂点の座標は一つに定まらない。でも一辺の長さは求まる。そういう問題だ
924132人目の素数さん:2007/06/22(金) 20:52:59
結局よくわからんが、x座標だけで考えれば2√10とかか?
なんか中三の学校の宿題程度な感じだが
925914:2007/06/22(金) 20:58:06
正解。
ちょっと平行移動してひねっただけでなんでここまで議論せにゃならんのだ?
926132人目の素数さん:2007/06/22(金) 20:59:19
>>914
とりあえず、翻訳してみた。

xyz座標空間に正四面体がある。
この正四面体の重心はx軸上にあり、4頂点のx座標はそれぞれ1,3,5,7である。
このとき,この正四面体の一辺の長さを求めよ。
927132人目の素数さん:2007/06/22(金) 21:00:36
>>921
重心ごと移動したら、重心の位置が変わるだろ
928132人目の素数さん:2007/06/22(金) 21:08:38
>926
うまく平行移動すりゃ重心はx軸上になくても解けるぞ。x軸に平行に-4移動してからyz平面に平行に移動すりゃいい。
>927
重心変わっても一辺の長さが変わらないから問題ないって>914は言いたいんだろ。
929920:2007/06/22(金) 21:15:24
重心は原点O(0,0,0)にあって、頂点のx座標が全部正っていうのが想像できないのだが
930132人目の素数さん:2007/06/22(金) 21:24:01
あのなあw
ここまでのレス見てないのか?x座標も重心も全部平行移動だよ
931914:2007/06/22(金) 21:26:58
うわっ、レスのびすぎ・・・
とりあえず>>928>>930代弁サンクス
932132人目の素数さん:2007/06/22(金) 21:40:52
いや、そうじゃなくて>>914が日本語としておかしいって言ってるだけだろ。
933132人目の素数さん:2007/06/22(金) 21:42:07
もういいよ。次の問題誰か頼む
934MASUDA ◆wqlZAUTQF. :2007/06/22(金) 21:57:17
じゃあわたくしが

a = 2^(4/3)、b = 2^(3/2)として、a^bとb^aの大小比較をせよ。
935132人目の素数さん:2007/06/22(金) 22:19:36
(゚Д゚)≡゚д゚)、カァー ペッ!! >>914
936132人目の素数さん:2007/06/22(金) 22:27:34
文科向け?
次の等式を満たす整数x,y,z(x≦y)の組をそれぞれ全て求めなさい。
(1) 2^x+2^y=2^z
(2) 3^x+3^y=3^z
(3) 2^x+2^y=3^z
937132人目の素数さん:2007/06/22(金) 22:28:25
>935
解けなかったからってひがむなよw
938132人目の素数さん:2007/06/22(金) 22:31:33
>>936
東大が連続して小問ださないと思うけど。(1)は無数にあってもいいの?
939132人目の素数さん:2007/06/22(金) 22:36:05
【問題】
アルファベットの大文字『H』に直線を3本書くと三角形が最大で【?】個出来きる、三角形が出来る最大数【?】個出来ると必ず【?】の形が現れるので答えよ。

※直、三角形の中心に直線を書くと三角形が二つになるが、枠の三角形も入れて三つとはしない。

答えの例:三角形の数=【3】個、形=【家】の場合、鍵は【3イエ】となる。
940132人目の素数さん:2007/06/22(金) 22:47:13
>>939
日本語おかしいえ
941132人目の素数さん:2007/06/22(金) 22:51:59
>>939
文章力ないだろ?
これがゆとり世代か…
寒い時代になったものだな
942132人目の素数さん:2007/06/22(金) 23:16:47
>936
おいらの勘違いならごめん、先に謝っとく。
(3)で1+2^y=3^zをみたす自然数解がy=3,z=2だけだと証明するのは高校生にはできるの?
943132人目の素数さん:2007/06/23(土) 02:29:17
>>942できるとおもうよ
944132人目の素数さん:2007/06/23(土) 05:23:41
>>942
y=1,z=1もあるよ
945132人目の素数さん:2007/06/23(土) 06:42:31
xとyが(x^2+y^2)^2=x^2-y^2を満たすときx^2+(1-y)^2の最大値を求めよ
946132人目の素数さん:2007/06/23(土) 09:00:00
(3^(z/2)+1)(3^(z/2)−1)=2^y。
947132人目の素数さん:2007/06/23(土) 14:23:51
>>936
文系
948132人目の素数さん:2007/06/23(土) 14:32:34
>934
微妙なひねり方をw
ありがちな問題と思ったらそういうことかw
ちなみに某サイトの益田さんですか?
949132人目の素数さん:2007/06/23(土) 14:52:48
eを自然対数、πを円周率として
(1)e^πとπ^eとではどちらが大きいか?
(2) (1)で小さい方をE、大きい方をΠとするとe^Πとπ^Eとではどちらが大きいか?
950132人目の素数さん:2007/06/23(土) 15:51:12
>>934>>948
ほんとだw
a<e<bになってるw
951132人目の素数さん:2007/06/23(土) 16:06:40
logx

log(logx)
952MASUDA ◆wqlZAUTQF. :2007/06/23(土) 17:07:12
>>948
ええ、その益田ですが。934はちょっと遊びで。予備校で出したら馬鹿の1つ覚えよろしくみんなlogx/xを持ち出したので。パターンはよくないよと。
953132人目の素数さん:2007/06/23(土) 17:16:33
益田氏って856の人か?
954MASUDA ◆wqlZAUTQF. :2007/06/23(土) 17:53:23
>>953
はい、わたくしですが。
955132人目の素数さん:2007/06/23(土) 20:06:29
実際に塾で出題されてる方から見たら
このスレの問題の大半はチラ裏程度なのかな
956MASUDA ◆wqlZAUTQF. :2007/06/23(土) 20:45:07
いえいえ、参考になるものもありますよー。私自身はどちらかというと京大系の問題を作ることが多いですけどね
957132人目の素数さん:2007/06/23(土) 20:50:30
(・∀・) ニヤニヤ…
958132人目の素数さん:2007/06/23(土) 21:29:09
>934
 a = 2^(4/3), b = 2^(3/2) より
 log_[2]( a^b ) = b*log_[2]( a ) = (4/3)b = (1/3)2^(7/2) = √(128/9),
 log_[2]( b^a ) = a*log_[2]( b ) = (3/2)a = 3・2^(1/3) = 54^(1/3),
 ここで 3^12 = 531441 > 524288 = 2^19 を使う。
a^b < b^a
959132人目の素数さん:2007/06/23(土) 22:17:39
>945
 √(x^2 +y^2) = r とおいて、与式をrだけで表わす。
 題意より r^4 = x^2 -y^2,
 x^2 = (r^2)(1+r^2)/2, y^2 = (r^2)(1-r^2)/2.
 (与式) = r^2 -2y +1 = r^2 ±2r√{(1-r^2)/2} +1
 = ((1+√3)/2){r ±((√3 -1)/√2)√(1-r^2)}^2 + (3-√3)/2,
複号は +がy<0の枝、-がy>0の枝に対応する。
点(0,1)からの距離の最大値の2乗を求めるから、y<0 の枝(+)をとる。
これは r^2 = (3+√3)/6 で極大値 (3+√3)/2 = 2.3660… をとる。
このとき、x^2 = (5+2√3)/12, y=-1/(2√3).

レムニスケート
高木: 「解析概論」 改訂第3版, 岩波書店 (1961) 第3章 積分法, §40, p.136 [例2]
http://mathworld.wolfram.com/Lemniscate.html
960132人目の素数さん:2007/06/23(土) 22:33:53
>>959
正答です
961132人目の素数さん:2007/06/23(土) 23:04:08
>926
 A(1,±1,±√5)
 B(3,干3,干√5)
 C(5,±3,干√5)
 D(7,干1,±√5)    (y座標、z座標それぞれで複号同順)
あるいは、x軸のまわりにGoogle回したもの。
辺の長さは >924
962132人目の素数さん:2007/06/23(土) 23:34:03
x,y,zを正の整数とする。
x+y+zがx,y,zの公倍数になるとき、x,y,zを求めよ。
963MASUDA ◆wqlZAUTQF. :2007/06/24(日) 00:25:04
>958
ご名答

>924
一般化できますね。原点に重心をもつ正四面体のx座標をa,b,c,dとすれば、正四面体の一辺の長さは
√{2(a^2+b^2+c^2+d^2)}
である。
964132人目の素数さん:2007/06/24(日) 00:34:02
>>956
実際のところ、このスレでも京大系の問題が多いですよね。
あなたのに限らず。
東大っぽい問題って作るのムズイんですかね。(というかメンドイ?)

下の問題は僕の自作なんですけど、こういうのが「東大っぽい」と個人的には思うんです。
こういうのあれば、出してくださいな。

座標平面上の3点O(0, 0), A(0, 1), B(√3, 1)を頂点とする三角形OABがある。
この三角形を、Oを固定して原点のまわりに毎秒1ラジアンの角速度で反時計回りに回転させる。
一方、点Pは三角形OABが回転を始めるのと同時に原点を出発し、
時刻θ(秒)におけるy座標がasinθとなるようにy軸上を運動するものとする。
ただし、aは正の定数である。このとき、以下の問いに答えよ。
(1) 0≦θ≦π/3において、Pが三角形OABの外部に脱出することがあるようなaの値の範囲を求めよ。
(2) θが0からπ/3まで変化するとき、Pが三角形OABの外部に脱出して、内部に戻ってこないようなaの値の最小値を求めよ。
(3) aを(2)で求めた値とする。θが0からπ/3まで変化するとき、
三角形OABの内部でPが通過した軌道がえがく曲線は三角形OABを2つの部分に分ける。
このうち、頂点Aのある側の面積を求めよ。
965132人目の素数さん:2007/06/24(日) 00:44:38
>>908に誰も反応してない件
966MASUDA ◆wqlZAUTQF. :2007/06/24(日) 01:25:22
>>964
いえ、私の場合は単なる好みです。東大より京大の問題の方が解いてて個人的に面白いので、自然と京大っぽい問題を作りたくなったんですよ。東大の場合は必要以上に「教育的配慮」を考えないといけないので、そういう意味では作問に面白さを見いだせず…
967132人目の素数さん:2007/06/24(日) 02:14:00
>>956
やっぱ参考になるのは素数様の問題ですよね?
968132人目の素数さん:2007/06/24(日) 03:29:10
>>966
>東大の場合は必要以上に「教育的配慮」

ああ、何となく分かります。
京大の問題は数学的に意味のある問題、自然な発想から生まれてくる問題、という感じで、
東大の問題は高校数学の全範囲をちゃんと理解しているかを総合的に試される問題、という感じですかね。
僕はどっちかというと後者の方が好きですね。
うまいこと作為的に仕組まれたパズルみたいな感じが面白い。
969132人目の素数さん:2007/06/24(日) 03:34:50
∫[0,1](xlogx)^ndx を求めよ。

どうよ?ちょっと考えるでしょ?大学生ならβ,γ関数使うけど高校生なら
ちょっと考えないといけない。
970132人目の素数さん:2007/06/24(日) 03:43:22
kougisekibun.
971132人目の素数さん:2007/06/24(日) 03:46:52
広義積分でなくともいけるぜ。一発解いてみてよ?
972132人目の素数さん:2007/06/24(日) 03:48:13
∫[0,1]x^n*(logx)^k dx = I[k]とおけばいいのかな
と大学生がふと思ってみる
973132人目の素数さん:2007/06/24(日) 04:03:46
>広義積分でなくともいける
何をわけのわからない事を。
974132人目の素数さん:2007/06/24(日) 04:06:59
>>973
馬鹿はレスするな。ちょっとした大学の研究小僧や助手程度では解けない。
解けないから非難するのは明らかに論理破綻しておる。
975132人目の素数さん:2007/06/24(日) 04:11:01
>>969
nは一応自然数ってことでいいんだよね?
976132人目の素数さん:2007/06/24(日) 04:16:44
>>975
ごめん。そう。問題に不備があった。nは自然数にしておく。
977132人目の素数さん:2007/06/24(日) 04:18:51
そろそろ次スレ?
978132人目の素数さん:2007/06/24(日) 04:19:34
>>980 でいいんじゃないかな。
979132人目の素数さん:2007/06/24(日) 04:26:01
↓よろ
980132人目の素数さん:2007/06/24(日) 04:27:48
>>972
とき方は自由だ。しかしその解き方では混乱してしまうかもしれない。
あなたの能力次第だが。自分にはその発想がない。

ヒントは後ほど出す。解はさらに後に出す。

981132人目の素数さん:2007/06/24(日) 04:59:55
[>>980] がたてないようなのでたててもいいかな?
982132人目の素数さん:2007/06/24(日) 05:07:15
次スレ。
★東大入試作問者になったつもりのスレ★ 第九問
http://science6.2ch.net/test/read.cgi/math/1182629190/
983132人目の素数さん:2007/06/24(日) 13:23:35
東大 理三 前 76
東北大 医 医 前 73
九大 医 医 前 73
東京医歯大 医 医 前 72
名大 医 医 前 72
京大 医 医 前 72
阪大 医 医 前 72
京都府医大 医 医 前 72
千葉大 医 医 前 71
神戸大 医 医 前 71
岡山大 医 医 前 70
北大 医 医学系 前 69
名古屋市大 医 医 前 69
横浜市大 医 医 前 68
大阪市大 医 医 前 68
熊本大 医 医 前 67
筑波大 医 医 前 66
金沢大 医 医 前 66
三重大 医 医 前 66
広島大 医 医 前 66
奈良県医大 医 医 前 66
東大 理一 前 65
群馬大 医 医 前 65
新潟大 医 医 前 65
滋賀医大 医 医 前 65
札幌医大 医 医 前 65
984132人目の素数さん:2007/06/24(日) 13:55:33
>983
やはり関東と関西で偏差値基準は違うんだな。関西では
東大 理三
京大 医
阪大 医
東北大 医
東医歯大
名大 医
九大 医
京府医大
金沢大 医
北大 医
の順
985132人目の素数さん:2007/06/24(日) 16:42:36
ume
986132人目の素数さん:2007/06/25(月) 05:00:00
百八十三日。
987132人目の素数さん:2007/06/26(火) 05:00:00
百八十四日。
988132人目の素数さん:2007/06/27(水) 05:10:00
百八十五日十分。
989132人目の素数さん:2007/06/27(水) 18:34:01
ume
990132人目の素数さん:2007/06/27(水) 23:40:27
うめるなら、うめきれよ
991132人目の素数さん:2007/06/28(木) 00:07:18
一般に、現在の第nスレが埋まったときの、第(n+1)スレのレス数をp_nとする。
ここは第8スレであり、1000まで埋まった時の第9スレのレス数をp_9、
及びそれ以前の p_1, p_2, ... p_8 から、lim[n→∞]p_n を考察せよ。
992132人目の素数さん:2007/06/28(木) 00:11:33
 
993132人目の素数さん:2007/06/28(木) 07:27:39
994132人目の素数さん
ume